Как стать автором
Обновить

Комментарии 561

НЛО прилетело и опубликовало эту надпись здесь
Рождение baby-universes из «bulk» — это отдельная гипотеза.

>в чём же находилась первородная бесконечная Вселенная?
Такой тип мышления череват ошибками, так как исходит от «здравого смысла» — типа, где то это должно было находиться… Здравый смысл — плохой помошних в таких теориях.
Однако, что касается моделей, которые пытаются объяснить сам big bang, то я знаю как минимум пять, но они дают разные ответы на ваш вопрос.

>действительно появилась из точки, только эта точка находилась не просто в пространстве, а в 4D пространстве-времени
Это возможно только для замкнутой вселенной. Мы не знаем, замкнутая наша вселенная или нет (скорее всего нет)

>И поэтому же бессмысленнен вопрос — где у Вселенной в пространстве тот центр, откуда она начала расширяться?
Даже для замкнутой вселенной этот центр находится не в пространстве, а в прошлом. Так что указать направление на центр (в 3D пространстве) невозможно

Впрочем, в одной из моделей с bulk, как ни странно, есть центр у каждой baby universe. Однако это может нас слишком сильно увести в сторону
НЛО прилетело и опубликовало эту надпись здесь
НЛО прилетело и опубликовало эту надпись здесь
НЛО прилетело и опубликовало эту надпись здесь
Ну да, я и говорю, так же как и двумерный житель сферы не сможет указать на центр шара.

Это при условии, что плоскость обитания двумерного жителя не является сечением сферы, проходящим через ее центр.
Двумерный житель сферы обитает, конечно же, на сфере. Это не плоскость. И сфера не проходит через свой центр.
Мне кажется, термин «обитатель сферы» не исключает возможность обитания внутри пространства, образованного поверхностью сферы.
Обычно именно для этого термин «обитатель сферы» и используют. Что-бы указать где именно он обитает. Пространство внутри сферы не принадлежит сфере.
НЛО прилетело и опубликовало эту надпись здесь
Вот прочитайте статью)
Большой взрыв не произошел в какой то определенной точке, а сразу везде. Пожтому ему не надо было куда то там расространяться со скоростью света. И границы между взрывом и невзрывом никакой нет. Опять эта жуткая аналогия со взрывом…
Ограничена скорость движения тел (частиц, света) относительно друг друга, когда они прилетают рядом друг с другом (это важно). Скорость растягивания пространства ничем не ограничена. При этом если в двух разных точках такого пространства покоятся два тела, то они могут удаляться друг от друга со скоростью сколь угодно превышающей скорость света, именно потому, что они не в одной точке, а находятся на расстоянии.

Это мы и наблюдаем во Вселенной. Где-то там существуют галактики, которые удаляются от нас со скоростью больше света. Именно поэтому мы их не видим, они удаляются слишком быстро, что свет от них никогда не сможет достичь нас. Эти галактики находятся за нашим космологическим горизонтом событий.
Именно поэтому мы их не видим, они удаляются слишком быстро, что свет от них никогда не сможет достичь нас.

Если это правда, то как-то это даже обречённо-грустно. В этом есть какая-то тщетность «покорения» космоса.
Так же тщетно, как выходить из комнаты, зная, что Земля не бесконечна :)
Скопления галактик, вплоть до сверхгалактики вполне хорошо связаны гравитацией ТМ. Но наука пока не придумала, как улететь на 1 млн. световых лет до 4000 г. н.э.
Есть разные теории, например что вселенная вывалилась из другого-пространства времени, и реликтовое излучение есть остаточное колебание этого явления.
В любом случае, нет той точки откуда она «взорвалась» (как например взрывается некий заряд в нашем понимании), под взрывом скорее понимается прорыв нашего пространства-времени с образованием элементарных частиц, потому потом собрались в облака, образовали первое поколение звезд, а из остатков первого поколения звезд уже образовались планеты.

Есть теория, что исходя из 10-12 мерности пространства (теория струн) то, что мы в своем мире видим как материю (фермионы) в других измерениях существует как энергия, оттуда и странные квантовые эффекты в нашем измерении.

В общем теорий куча, но это теории, проследить историю Вселенной до моменты Большого Взрыва невозможно.
В общем теорий куча, но это теории, проследить историю Вселенной до моменты Большого Взрыва невозможно.
Я правильно понимаю, что невозможно на наших современных технологиях? То есть, к примеру, если мы каким-то образом сможем взять информацию из других 6-8 измерений, то, возможно, там информация будет больше?
Если теория верна, то возможно в других измерения информации о возникновении нашей вселенной будет больше.

Но согласно тем же теоретическим выкладкам, эти пространства для нашего мира чрезвычайно «маленькие», и чтобы проникнуть в эти пространства нужно невероятное количество энергии, кто-то из ученых говорил про «коллайдер величиной с Галактику».
Проследить в момент БВ не выходит, т.к. практически вся информация ограничена реликтовым излучением (эпоха рекомбинации, около 380 тысяч лет после БВ).
Есть попытки заметить сам факт инфляции (кроме теории о том, что начальные флуктуации плотности материи вызваны квантовыми флуктуациями в «инфлатонном поле»). Но факт наличия такого поля тоже может оказаться погрешностью наблюдений.
вселенная вывалилась из другого-пространства времени

Идея заманчивая, но для доказательства этого нужны «железные» мотивы произошедшего. Что могло повлиять на свёртывание (10-12)и-мерного в 3х-мерное пространство. Повышение энтропии? Отбор энергии и упорядочивание подобно механизму кристаллизации? или креационизм? Неизвестно.
Что могло повлиять на свёртывание (10-12)и-мерного в 3х-мерное пространство

И может ли наше 3-мерное так же свернуться в 2-мерное?
Конечно. Как-то так
Заголовок спойлера
Корабль полз назад к двумерному участку космоса — сначала медленно, затем постепенно ускоряясь. Двигатели «Омеги» всё так же давали «полный вперед». Это чуть притормозит падение корабля и отсрочит неизбежное, но очень ненадолго.

В двух тысячах километров отсюда сияние, испускаемое сплющившимся катером и его экипажем, погасло. В отличие от коллапса четырехмерного пространства в трехмерное, переход трех измерений в два сопровождался намного меньшим высвобождением энергии. Две плоские фигуры четко виднелись в звездном свете. На сплющившемся катере просматривались как детали конструкции — кабина, термоядерный реактор и прочее — так и развернувшаяся в плоскость фигура наблюдателя в кабине. В плоском теле другого астронавта четко просматривались кости, кровеносные сосуды и другие органы.

В процессе выпадания в два измерения каждая точка трехмерного объекта проецировалась на плоскость в прямом соответствии с геометрическими законами, и потому эти две фигуры оказались наиболее полными и точными отображениями первоначально трехмерных катера и людей. Все внутренние структуры лежали теперь бок о бок в двух измерениях, ни одна деталь не перекрывала другую. Процесс проецирования, однако, сильно отличался от того, что используется в технических чертежах, поэтому визуально восстановить первоначальную трехмерную структуру было трудно.

Самое большое отличие от чертежа заключалось в том, что двумерное развертывание произошло на всех уровнях: все исходные трехмерные структуры и детали легли параллельно в двух измерениях, и результат до некоторой степени повторял эффект, получавшийся при просмотре трехмерного мира из четырехмерного пространства. Это очень напоминало изображения фракталов: сколько ни увеличивай часть изображения, менее сложным оно не станет. Однако фракталы — это теоретические концепции, потому что их действительные изображения неизбежно ограничиваются разрешением: после увеличения в несколько раз изображения теряют свою фрактальную природу. С другой стороны, сложность трехмерных объектов, развернутых в двух измерениях, была реальной: разрешение их лежало на уровне элементарных частиц. Наблюдательный терминал «Омеги» имел ограниченное разрешение, но от сложности и количества подробностей у зрителей голова шла кругом. Это был самый сложный образ Вселенной; если непрерывно смотреть на него слишком долго, можно потерять рассудок.

И катер, и его экипаж больше не имели толщины.
Мне кажется, что двумерному миру придется создавать свою жизнь, наша никак не подойдет.
Так а там не говорится, что наша подойдет.
Они, очевидно, погибли.
Просто отпечаток остался.
Если быть совсем точным, то 10-12 мерного в 4х-мерное. Время в эти измерения точно входит и по идее обособленно. Хотя для НФ про путешествия во времени можно ввести «2-мерное время».
Ну подождите, разве сингулярность в ЧД в нашем 4х-мерном пространстве — это не 3-х мерное пространство со свёрнутым временем, где место времени занимает радиус? Т.е. я всегда считал, что наша вселенная — это ЧД в 5-мерной вселенной и так далее.
Обмен ролями между радиусом и временем, в силу изменения их знаков в метрике, это свойство шварцшильдовских координат при применении их к пространству времени внутри ЧД — все, что под горизонтом. Там число измерений то же самое, что снаружи. Более того, это именно свойство координат, в других координатах такого изменения не происходит. Сингулярность же это просто точка или, в случае вращающейся ЧД, тонкое кольцо, там ряд параметров в наших формулах обращается в бесконечность, что тоже больше говорит про наши формулы, чем про природу. Но путать сингулярность со всем объемом ЧД не стоит.
Ну это в общей теории относительности сингулярность — просто точка. Однако в теории гравитации Эйнштейна — Картана гравитационный коллапс образует так называемый мост Эйнштейна — Розена. Материя достигает огромной, но конечной плотности и «отскакивает», образуя другую сторону моста Эйнштейна — Розена, которая растёт в качестве новой вселенной. И вот размерность этой новой вселенной как раз и будет на единицу меньше.
Эта теория мне нравится тем, что увязывает все парадоксы: и то, что радиус Хаббла наблюдаемой Вселенной равняется её радиусу Шварцшильда. И свёрнутые измерения. И объясняет тёмную энергию.
У нас нет инструментов чтобы даже предположительно понять, что было до БВ. После БВ остались следы, и развитие Вселенной более-менее не противоречиво описано математически, а вот что было «до» — полная загадка.

Что касается 10 мерного пространства и вообще теории струн. Понятно, что все это теории, проверить это экспериментально пока нет возможности, но на сегодняшний день «теория струн» — единственная теория, которая не противоречит ни ОТО ни СТО, объединяет все 4 взаимодействия, то есть претендует на место «теории всего».
Если вы планируете дожить до «окончательной версии», у меня для вас плохие новости…
Про приливные силы сами ерунду написали. Откуда взялось 1 секунда и 300000 км? Например, для ЧД в 10 солнечных человека порвёт на расстоянии порядка 1000 км. При этом радиус её будет около 30 км.
Мне врезалось в память с physicsforums. Похоже вы правы, и это расстояние значительно меньше
Я не знаю модуль Юнга человека, но можете найти для стального прута.
ЧД в 10 солнечных масс будет иметь уск. свободного падения 2.74 км/с2 на расстоянии около 700 тысяч км, на расстоянии 70 тысяч вырастет до 2.74*105м/с2.
То есть факт способности разорвать падающий объект тут важен в сравнении с нейтронной звездой (уникальной будет НЗ массой 2 массы Солнца) или обычным голубым гигантом (он имеет слишком большой радиус при массе 10 масс Солнца и выше).
>Я не знаю модуль Юнга человека
Ищу добровольца для эксперимента)
НЛО прилетело и опубликовало эту надпись здесь
НЛО прилетело и опубликовало эту надпись здесь
Речь не про ускорение свободного падения, а про его градиент. То есть надо посчитать, насколько у вас ноги будут разгоняться быстрее головы. Так вот, если я правильно прикинул, для ЧД в 10 солнечных на расстоянии в 1000 км приливная сила составит около 350 Н. Это аналогично, если вас подвесить за руки и к ногам привязать груз 35кг. Согласитесь, что это точно не разорвёт

Если учесть, что пространство тоже искажено, то собственно линейная деформация человеческого тела (если привязать нулевую точку отсчета к голове или ногам) остается под вопросом. А учитывая свободное падение, я думаю, что испытатель будет испытывать натуральную невесомость до момента замедления (столкновения с гипотетической поверхностью).
И вот тогда — пшик. Очень большой пшик.

Почему невесомость, он же будет ускорено двигаться? То есть двигаясь, например ногами вперёд, ему тяжело будет опустить руки. А если головой вперёд, то поднять.
Нет. В падающих самолетах создают самую настоящую невесомость
В самолете ускорение константа при невесомости, а при падении в дыру нет.
Нет никакой разницы, падать в дыру или на Землю. Ускорение растет и там и там, просто в случае Земли рост слишком мал, чтоб его заметить.
Именно потому что ускорение ускорения, при падении на землю мало его не заметно. А разница в том, что при падении на землю вас не порвёт ещё до момента касания земли. А при падении на дыру малой массы порвёт именно из-за ускорения ускорения.
Постойте. Мы же можем «падать» в ЧД так, чтобы оставаться при этом на её орбите, верно? Мне кажется нужно было бы сделать замечание о таком специальном значении слова «падать» в этом контексте. То есть, находясь на некотрой орбите вокруг ЧД наш космческий аппарат будет испытывать приливное воздейтсвие, скорее весего он окажется в приливном захвате, а человек на борту будет комфортно испытвать невесомость пока его тело находится вдоль поверхности гравитирующей массы. Однако развернувшись поперёк он испытает разницу в ускорениях между ногами и головой и будет растянут за означенные части тела с некоторой силой.
Это возможно только далеко от ЧД
Ближе к ЧД, ниже 'орбиты света', невозможны никакие замкнутые орбиты для пассивных тел. Однако, вы можете вращаться вокруг ЧД, если использовать двигатели
Ну вот. А говорили что пересечение горизонта можно даже не заметить. Согласитесь, потеря возможности вращаться по орбите не теряя «высоту» это очень даже заметный эффект!
Может потом опишите, что за «альтиметр» будет использовать гипотетический Купер«гном в скафандре», находясь под горизонтом.
А еще можно падать в горизонтальном положении.
Можно легко показать, что скажем равнобедренный треугольник момент силы тяжести разворачивает основанием вниз, если речь не про движение по круговой орбите с первой космической.
В случае с круговой момент центробежной силы наоборот, развернет основание в ту сторону, где выше «потенциал центробежной силы. Именно такая особенность обоих сил приводит к проблеме стпутников. Главная затрата топлива в них не на торможение (если говорить о геостационарной), а на то, чтобы каждые сутки совершить 1 оборот вокруг оси.
Сударь, мне кажется в данном случае правильнее привязывать человека за ноги и крутить. Вообще, как полагаете правильнее падать в ЧД, головой или ногами вперёд?
Разница невелика и инструментально её установить практически невозможно
Но и без Юнга можно оценить, что когда ускорение свободного падения будет больше 20 земных наблюдателю уже сильно поплохеет.

Какая ему печаль до ускорения с которым он свободно падает ?

Ему нет дела до скорости, а до ускорения ему как раз-таки дело есть
Не расскажете, какое именно?

Какая разница телу, свободно ускоряющемуся в [условно] однородном гравитационном поле, ускоряется оно 10 м/с² или 200?
Так в том то и дело что оно не однородно
Дело в том, что свободное падение в гравитационном поле, это не совсем ускорение. По ОТО вы не ускоряетесь, а движетесь по геодезической, т.е. равномерно прямолинейно, но в искревленном пространстве-времени. А разорвет вас из-за того, что геодезические, по которым движутся ваши ноги и голова, в этом самом пространстве-времени расходятся, и как раз вторая производная от функции расстояния между геодезическими в трех пространственных измерениях по функции расстояния между ними же во времени и будет тем самым «ускорением», которое вас разорвет. Причем рассчет влоб даст лишь приблизительную оценку, т.к. модуль Юнга тоже будет зависеть от постоянно изменяющегося градиента координат точек вашего тела по оси времени.

Как результат, точный рассчет момента разрыва можно давать как очень годную тему курсовой работы студентам физ. факультета.
геодезические, по которым движутся ваши ноги и голова, в этом самом пространстве-времени расходятся
Что по рабоче-крестьянски звучит как 'гравитационное поле неоднородно'. Эта история отлична от
когда ускорение свободного падения будет больше 20 земных наблюдателю уже сильно поплохеет
хуже того, при (средней) напряженности гравитационного поля в 20g получить градиент, опасный для механической прочности тушки, довольно затруднительно

Со вторым согласен. А по поводу первого — все атомы Вашего тела движутся по какой-то геодезической, просто в уравнении геодезической ещё нужно учесть «все прочие силы» в уравнении, в котором фигурирует 4-ускорение и 4-скорость такого вида:
Картинка.
Наверно, ещё нужно учесть скорость передачи гравитационного воздействия от ног к голове
давайте привяжем вам к ногам гирю и выкинем из самолета.
Каждый его атом свободно падает.
Но пространство в целом искривляется.
Поэтому просто изменяется «расстояние» между атомами в разных частях тела.
Но это — изменение, которое как бы видно наблюдателю. А не подопытному.
Стоя рядом с ЧД, мы, как сторонний наблюдатель, увидим падающего в неё человека искаженным. Прям как на картинке выше.
А он сам будет чувствовать себя хорошо, пока электрические импульсы будут доходить по его нервам от точки, в которой находятся его ноги до точки, в которой находится его голова…
Искривлённое пространство => разное ускорение разных частей тела. Если ноги подопытного ускоряются быстрее, чем голова (скажем, голова ещё ускоряется с 10g, а ноги уже с 15g, разрыв в 5g), то ему может вполне неиллюзорно поплохеть, причём гораздо раньше, чем нервные импульсы перестанут доходить. В конечном итоге разница ускорений станет невыносимой и его разорвёт приливными силами.
А может до разрыва костей сила гравитации не пропустит «ток» ионов кальция до столба мозга (или как там называется отдел, контактирующим со спинным м.?).

Наблюдатель не точечный. Если ноги падают быстрее головы, это уже печально.

'ноги быстрее головы' это про [существенно] неоднородное гравитационное поле.
исходный же посыл был, что 20g достаточно самих по себе
Tzimie по последним данным вроде скорость вращения чд около половины скорости света nplus1, может ли материя на горизонте событий разгоняться до световых скоростей?
Момент вращения ЧД может быть разным, зависит от исходного момента коллапсирующего тела. Есть некий «максимальный» момент, который дыра может поглотить. Такая дыра называется «критической». Если черная дыра поглотит еще больше момента, то непонятно что будет (проблемы с теорией и оголяется сингулярность). Считается что большинство реальных черных дыр близки к критическим.
Честно говоря не очень понятно в чём именно там проблема. В сверхкритической чёрной дыре материя, чтобы она в такой форме не представляла окажется над горизонтом, т.е. её линейная скорость будет выше скорости убегания. Следовательно она должна покинуть окрестности чёрной дыры. Или проще говоря — чёрную дыру разорвёт центробежной силой.
в сверхкритической черной дыре нет горизонта.
по этой причине да, голая сингулярность видна и может даже излучать
я бы не рекомендовал думать о таких критических объектах в сугубо классических терминах «центробежной силы».
Чисто интуитивно голые сингулярности мне не кажутся страшными объектами, их существование было бы интересным.

Компьютерные симуляции дают противоречивый результат: у некоторых получалось такие объекты формировать, у других исследователей критическая черная дыра отказывалась принимать дополнительный момент вращения, буквально отплевываясь от вращающихся тел, которые могли сделать ее сверхкритической.
Вот интересно, если ей туда не орбитальный момент напихать, от которого, я вполне допускаю, что можно отплеваться, а спин. :) Жаль, что без квантовой гравитации этого не понять.
именно, непонятно как отплевываться от поляризованных фотонов или электронов, которые туда впрыскиваются с полюсов
А с другой точки зрения наоборот должен быть такой эффект. Любая обычная звезда для коллапса в НЗ или ЧД должна куда-то скинут спин. Либо он уйдет с внешними оболочками, либо будет выкинут фотонами и гравитонами. Конечно гравитон пока штука гипотетическая, но на Вики (ага, самый авторитетный и свежий источник) написано, что любая теория поля со спином 2 дает физические проявления, как и ОТО.
Так что должна выйти разница между потоком вылетающих из ЧД (при формировании, слиянии и т.д.) фотонов и гравитонов с проекциями спина «вверх» и «вниз» относительно общей оси, заданной начальным суммарным вектором орбитального момента вещества.
Но так как «с другой стороны от источника ГВ» у нас не своего LIGO, то вроде как проверить этот вопрос не выйдет.
Там надо посчитать, не вырастет ли мощность хокинговского излучения с такой дыры. А то может она просто начнёт сильнее светится по экватору, и орбитальный момент будет уноситься излучением.
Ответ на ваш вопрос ДА — но это относительно далеких, невращающихся объектов.
НЛО прилетело и опубликовало эту надпись здесь
Вот отличная отмазка. Как кто-то сказал выше — «на ней заклятье». Ага.
НЛО прилетело и опубликовало эту надпись здесь
Пойдем по пунктам.
>При ускорении массивного тела, масса этого тела растёт при приближении к околосветовым скоростям
Это миф (ныне не используемое понятие *релятивисткой массы*) — пока не спорьте, я обещаю рассказать про это в выпуске 2

>что и препятствует нормальному ускорению, посредством реактивной тяги, например
В СТО тело не может двигаться быстрее света (локально относительно близких объектов), и никакая сила не может ускорить тело быстрее света
В ОТО сверсветовые движения возможны, но относительно далеких объектов
По честному нужно записать уравнение геодезической для 4-скорости пробного тела в скажем метрике Шварцшильда.
Да, в ОТО (ЕМНИП) нарушится обычное правило
uiui = 1
>В то время как остальная Вселенная уже давным давно разлетится на фотоны.
Как я уже говорил и показано рассчетами, падая в черную дыру внешний мир вы будете наблюдать ЗАМЕДЛЕННО а не УСКОРЕНО

Что касается «В то время как» вы здесь попали в ловушку «здравого смысла», мы считате что есть некоре общее глобальное время, и что может иметь смысл например такой вопрос: когда я достиг сингулярности, что произошло на земле?" Такой вопрос не имеет смысла. Но имеет смысл другой: пусть с наблюдаю за часами на земле. Какой будет последний отсчет, который я увижу перед сингулярностью? Как уже говорилось, на земле пройдет немного
НЛО прилетело и опубликовало эту надпись здесь
Вообще-то, у меня есть подозрение, что «замедление времени» конечно более сложный процесс, но можно упростить его понимание до банальной механики.

Если гравитационные силы настолько велики, что вмешиваются в микропроцессы замедляя движение молекул, атомов и частиц, то наблюдать там уже ничего не выйдет. Ведь время — это по сути скорость протекания процессов.

Ни техника ни живое существо существовать в таких условиях не сможет. И подозреваю, что это случится задолго то семикратного замедления. Поэтому даже мысленный эксперимент про «наблюдателя» некорректен.

Если замедляется абсолютно всё, даже скорость света, то ни техника, ни живые существа этого замедления никак заметить не смогут.

В том-то и дело, что не может все «просто замедляться».
На микроуровне происходят процессы, которые влияют на скорость прохождения реакций на более высоких уровнях.

Например, вы говорите замедляется скорость света. Это косвенно означает, что замедляется движение всех частиц, включая, например, электроны?
Что происходит с атомами, на орбите которых летали эти электроны? Атомы еще остаются атомами или структура веществе необратимо меняется?
По ОТО ничего не происходит, так как пространство так же искажается. Более того, замедление времени относительно — для пролетающего мимо Земли звездолета, время замедлиться на Земле, а на Земле — наборот. Поэтому не может замедляться, что-то одно иначе мы все бы развалились на части. Вас же не удивляет, что относительно релитового излучения мы летим с бешеными скоростями, но при этом сами этого никак не замечаем. Точно так же с временем, с точки зрения локального наблюдателя течение времени остается совершенно обычным, это скорость времи всех остальной Вселенной становится другой.
Более того, замедление времени относительно — для пролетающего мимо Земли звездолета, время замедлиться на Земле, а на Земле — наборот.

То, что в чайнике рядом со стаканом вода вскипает, не означает, что для воды в стакане в кипящей воде ускорилось время и наоборот. IMHO это все же спекуляция терминологией. Просто для живого наблюдателя время критично в силу его ограниченного существования.

Поэтому не может замедляться, что-то одно иначе мы все бы развалились на части.

Конечно не может. Но я имел ввиду, что происходящее на макроуровне — следствие того, что происходит на микроуровне.

Мне не нравится сам термин «Скорость времени». Мне кажется, что более логичным является термин «скорость протекания процессов».
Но тогда всех процессов. Как процессов с точки зрения диф. уравнений движения, так и с точки зрения всех 3 взаимодействий.
P.S. Ещё желающие могут узнать, описывается ли распад бозона Хиггса «скоростью» процессов поля Хиггса.
Фактически, скорость света не может замедляться или ускоряться, потому что её величина условна, это всего лишь соотношение между двумя измерительными линейками: линейкой метров и линейкой секунд. Можно спокойно переделать все линейки, приняв c=1 (планковская система величин) — и вся физика останется неизменной. Релятивистское «замедление времени» эквивалентно повороту в 4-пространстве, которое изменяет видимые пропорции линеек, но никак иначе не влияет на материю и локальные процессы. Атомы остаются атомами, просто со стороны стороннего наблюдателя они двигаются медленнее, а внутренний наблюдатель ничего не замечает. Только если наблюдатель сильно распределённый, разница протекания процессов в разных частях системы начнёт на него ощутимо влиять.

ЗЫ. Вообще, нужно помнить, что величина любой физической константы, имеющей размерность, конвенциальна, и может быть безболезненно изменена переопределением «метра», «секунды», «килограмма» и т.п. Истинные константы — безразмерны (постоянная тонкой структуры, например).
В СТО все так. В ОТО еще нужно учитывать эффекты геометрии. Так же, как длина одного градуса долготы различна на экваторе и в Питере, хоть в метрах ее считай, хоть в милях, в ОТО в разных точках пространства могут отличаться временные отрезки и длины, в каких единицах ни считай. Т.е. двое часов, пролетевшие из А в Б одним маршрутом и другим, могут показать разное время, в чем бы они ни измеряли время.
Это относится к случаю протяжённого наблюдателя, про который я упомянул отдельно.
Есть у кого-то теория, что замедляется скорость процессов, но только электромагнитных.
Конечно ни одно живое существо не переживет, если между его мозгом и сердцем скачек («плавный», так сказать) метрики обеспечит разницу течения времени в 137 раз.
А так, я не знаю, когда разорвет стальной прут возле ЧД массой 3 массы Солнца.
Есть у кого-то теория, что попав в ЧД вы окажетесь в мире, где летающие пони какают радугой. [/sarsacm]

ОТО вполне четко говорит, что замедлятся все процессы и все эксперименты это подтверждают, более того множество выкладок показывают, что изменение только части процессов из-за относительно скорости — сломает вообще все. Земля ведь тоже движеться со скоростью близкой к световой относительно определеных точек отсчета, то есть даже у нас на Земле мы должны это наблюдать.

А вы пишите где-то у кого-то есть какая-то теория, ну дайте тогда ссылки на эту теорию, причем желательно с математическими выкладками.
А «альтернативщиков» редко бывают математические выкладки. Такие теории происходят наверное от «эфирщиков», но точно не помню.
А так понятно, что скажем в СТО замедлялась скорость распада мюонов, а это слабое взаимодействие. Доказательство тому — детектор LHCb в процессах с sqrt(s) = 8 ТэВ фиксировал разпад W-бозона на мюон и антинейтрино (заточен именно под мюоны тригер «запиши событие»).
Или скажем происходит рождение пары Z-бозонов (тот самый сеанс LHC, но детектор CMS) с распадом на пары «электрон-позитрон» или «мюон-антимюон». Эти 4 частицы рождаются с огромной инвариантной массой в диапазоне от 110 до 1100 ГэВ. Например поперечный импульс пары свыше 45 ГэВ.
Так как ОТО — общий случай для СТО, она не будет так уж сильно меняться закон, дающий частице с массой покоя m иметь кин. энергию T >> mc2
Я вот не понимаю, почему находясь на Земле, я наблюдаю немного ускоренную версию вселенной, находясь на Юпитере, я буду наблюдать ещё более ускоренную вселенную. Но вот в ЧД внешний мир вдруг замедлится. Почему?
Потому что на Земле вы НАХОДИТЕСЬ а в черную дыру вы ПАДАЕТЕ
Если вы будете ВИСЕТЬ недалеко над горизонтом, то вселенная будет выглядеть ускоренной
>Разве что-то помешает ему набрать скорость выше световой — относительно удалённого неподвижного наблюдателя — после пересечения горизонта событий?

Тут сразу несколько чисто математических проблем. Скорость это вектор, а в используемой в ОТО римановой геометрии осмысленно сравнивать вектора мы можем лишь в одной точке. Если у нас через одну точку пролетают одновременно два тела, мы можем сравнить их скорости. Если же они далеко друг от друга, то объективного ответа «какова скорость второго тела в системе отсчета первого» уже нет, т.к. результат зависит от того, как мы проводим в пространстве-времени множество одновременных событий, и даже по какому пути мы переносим вектор скорости из одной точки в другую для сравнения векторов.

Мы можем использовать метрику Шварцшильда в привычном виде, она описывает как все выглдядит для удаленного статичного наблюдателя. Но для такого наблюдателя, в такой системе отсчета, падающий в ЧД объект никогда не пересечет горизонт событий, так и будет вечно к нему приближаться все медленнее. (Считаем ЧД тоже статичной, с испарающейся все сложнее) Получается, что осмысленно говорить о скорости кого-то упавшего под горизонт в такой удаленной системе отсчета не получится. Даже если думать не о том, кто падает в ЧД снаружи, а уже там внутри как-то оказался, в привычных координатах Шварцшильда внутри ЧД знаки у времени и радиуса меняются местами в метрике, радиус начинает работать как время, и внутри ЧД у всех объектов скорость как бы быстрее света (для внешнего наблюдателя), но это не настоящая скорость, а то, что внутри ЧД можно назвать скоростью, для стороннего наблюдателя выглядит не как метры радиуса в секунду времени, а наоборот, секунды времени в метры радиуса, т.к. роли этих измерений поменялись. Но это уже спекуляции, проще и корректнее считать, что все, что под горизонтом, уже за пределами пространства-времени внешнего наблюдателя, так что его координатами там пользоваться смысла нет.
Как уже говорилось, перейдите из Швардшильдовой метрики в Eddington или Kruskal и проблемы исчезнут. Бесконечность на event horizon чисто техническая, связанная с неудачным выбором метрики.
Во-первых, это та же самая метрика просто в других координатах.
Во-вторых, такая смена координат отражает смену «точки зрения», для статичного удаленного наблюдателя шварцшильдские координаты ближе всего к натуральным, координаты Крускала же ближе тому, кто падает.
В-третьих, проблема выбора общего «сейчас» и сравнения векторов никуда не девается. Есть два корабля у ЧД, какой момент истории одного корабля одновременен текущему моменту истории другого? Ответ не так однозначен же.
>Во-первых, это та же самая метрика просто в других координатах.
Да, вместо «координат» написал «метрика». Но главное, бесконечности там нет

>Во-вторых, такая смена координат
Да

>В-третьих, проблема выбора общего «сейчас» и сравнения векторов никуда не девается
А в ОТО в общем случае НЕТ глобального «сейчас»
Вот об этом и речь. Выше человек спрашивал про скорость относительно внешнего наблюдателя. Ее фиг определишь однозначно.
Для того, чтобы замедление времени было бы таким, планета должна была быть на расстоянии чуть более 0.003% от радиуса Шварцшильда. Между тем, судя по виду на черную дыру
Самый популярный миф, на который Кип Торн отвечал в интервью:

Время там идёт замедленно — один час на ее поверхности равен семи земным годам.

ПРЕТЕНЗИЯ: Такое возможно только вблизи объектов, обладающих огромной массой, например, на орбите черной дыры. Но нужно находиться совсем рядом с дырой, практически над ее поверхностью. А стабильная орбита вокруг черной дыры должна превышать диаметр Гаргантюа как минимум трижды. Иначе планету Миллер давно бы засосало внутрь. С учетом показанных в фильме кадров время на поверхности планеты должно течь медленнее, чем на Земле, всего процентов на двадцать.

Это верно в отношении невращающихся черных дыр, но с Гаргантюа все обстоит по-другому. Гаргантюа — сверхмассивная вращающаяся черная дыра, что несколько меняет ее воздействие на окружающее пространство. При определенных условиях, скажем, если она будет вращаться очень быстро, а планета Миллер — располагаться достаточно близко к циркулярной орбите Гаргантюа, такое замедление времени возможно.

Правда, у вращающихся черных дыр есть предел скорости вращения, причем максимума они, как правило, не достигают. Чтобы на планете Миллер было такое замедление времени, Гаргантюа должна вращаться лишь чуточку меньше максимума. Это реально, хотя и маловероятно.


Это отмазка с его стороны
Такое замедление возможно только у эргосферы, которая находится недалеко от горизонта событий, где то в райное 1.5радиуса в случае максимального вразения черной дыры. И то только при «ретроградном» вращении, что почти невозможно для планеты. Кроме того, те же эффекты увлечения около эргосферы сразу бы разорвали планету при малейшем ее вращении.

По этой причине я предпочитаю fantasy, сказали бы про что на планету наложено заклятье, и никто бы и не вякал.
С одной стороны «отмазка» профессора и лауреата нобелевской премии, с другой стороны ваши допущения. Даже не знаю кому верить.
Про свойства вращающихся ЧД ничего толком не знаю.
По поводу планет, на поверхности которой скорость течения времени в разы меньше, чем на орбите. В самом грубом приближении скорость течения времени — это функция потенциала GM/r. Отношение разности потенциалов между 2 точками на линии к центру ЧД к расстоянию между точками — это будет средняя сила гравитации на этом участке. Тут вспомнил, что нужно добавить — если на участке «планета-орбита» средняя гравитация 100 «же», значит на планете она как минимум больше.
Без фантастической «большой ЧД» (грав радиус хотя бы 0.1 а.е.) мы не получим большой разности течения времени, если только «на орбите вокруг планеты» — это не точка много дальше точки Лагранжа (номера не помню, той что дальше по лучу «звезда-планета»).
НЛО прилетело и опубликовало эту надпись здесь
Будет!
И аватарку поставь, чтобы легче узнавать! =)
Шелдона — разоблачителя!)
Ещё стартрек претендует на научность, можно оттуда поразбирать, что близко к реальной науке
В последнем сериале Стартреку они мгновенно путешествуют по миру вставляя провода в животное. Меня «порадовало».
Не, ну я конечно имел в виду классику: оригинал, следующее поколение, DS 9
В TNG/DS9/VOY уже все весьма далеко от научности. Технический прогресс достиг небывалых высот.
Просто смотря TOS видеш такой примитивный уровень технологий, придуманный в начале 60х годов для 2270 года. Потом смотришь ENT и удивляешься, почему в 2150 все весьма развитое. А потом вспомнинаешь, что скажем у Стругацких в 2150 уже весьма развитая цивилизация.
Та я так, вспомнил грустную историю.
Зачем вы назвали астромиколога лейтенанта Пола Стеймица животным??
А вы видели, как они с тихоходкой обращаются? Настоящие животные!
У меня такой вопрос:
— есть парадокс, согласно которому, если два корабля связанные тросом синхронно ускоряются до скорости света, то трос разорвёт
— я правильно, понимаю, что этот эффект при приближении к скорости света разорвёт и сам корабль тоже?
О, да, паравозик Эйнштейна. О него было сломано много копий.

Действительно, чтобы длинный жесткий объект ускорить, разные его части должны двигаться с разным ускорением (хвост быстрее). Если ускорение небольшое (реактивный двигатель) то ничего не разорвет, но ускоряться будете долго — жесткость корпуса все компенсирует (корабли немного сожмет что анрушает идеальность ускорения).

Однако здесь есть более глубокая тема: если объект ОЧЕНЬ ДЛИННЫЙ, то есть максимальное ускорение, которое он может выдержать, оставшись целым. Потому что хвост не может двигаться быстрее скорости света. Поэтому если вы ускоряетесь и обернетесь назад, то где то очень далеко будет область, которая вас никогда не сможет догнать. Там образуется (только для вас!) видимый горизонт (как у черной дыры!) от которого идет излучение Хокинга. От него вам будут лететь фотоны (которые видите только вы!). Это пример того, что в квантовой механике ускоренные наблюдатели не всегда будут согласны в том, существуют какие либо частицы и сколько их.

Про все это безумие можно начать читать тут: en.wikipedia.org/wiki/Unruh_effect
А посчитать на какой скорости при заданном ускорении (к примеру 1g) разорвёт, к примеру километровый корабль, как?
Не скорости а ускорении!
Если ваш реактивный двигатель магическим образом ускоряет вас 1g, то мы можете ускаряться хоть год и чувствовать все тот же 1g. Для других мфы будете укорачиваться и ускоряться все меньше и меньше, приближаясь к скорости света.
То есть, при 1g можно ускоряться приближаясь к скорости света, не боясь, того что корабль разорвёт?
Вы зря прицепились к приближению к скорости света. Скорость относительна, поэтому вы даже сами можете поставить эксперимент и узнать разорвёт ли вас при приближении к скорости света!

Представьте, что мимо Земли пролетает инопланетный космический корабль с скоростью 0,99с. Для него именно вы движетесь со скорость 0,99с. Теперь вы садитесь в автомобиль, начинаете ускоряться (пусть со ускорением 1g, неважно)… и вас разрывает. Так что ли?
Только если вы летите в идеальном вакууме. В реальном космосе энергия столкновений с космическими лучами, свободными атомами и прочим мусором будет расти с вашей скоростью и в какой-то момент вас просто поджарит ими.
Это не меньшее безумие, чем рождение внутри протона пары частиц с массой 8-9 масс покоя протона. Там «ускоренный наблюдатель» не согласен с составом протона будет.
— есть парадокс, согласно которому, если два корабля связанные тросом синхронно ускоряются до скорости света, то трос разорвёт
Вы уверены, что есть такой парадокс? Разве трос вообще заметит изменения в точке отсчета кораблей?
Дамс, все так сложно с этой физикой.
Нет, не правильно.

В парадоксе с тросом, как почти во всех парадоксах теории относительности, не принимается в расчёт относительность одновременности в разных системах отсчёта. При одновременном ускорение в сопутствующей системе отсчёта связанных кораблей трос не порвётся.
Да, ключевое слово — сопутствующей
А при одновременном ускорении в «земной» — порвется.

Вообще интересно что даже знаменитый Бэлл известный своей теоремой и не только запутался в этом
Ну я не вдаюсь в тонкости. Понятно, что имеется ввиду общая инерциальная СО кораблей. Излишне вам говорить, что ускоренная СО не может быть глобально инерциальной, а только локально. Поэтому такой СО нельзя «хорошо» накрыть и передний, и задний корабль, сколько-нибудь удалённые друг от друга.
сколько-нибудь удалённые друг от друга.
А в цифрах это сколько? Километры? Метры?
Зависит от ускорения. Из размерности задачи (есть только два параметра ускорение А и скорость света С) можно составить только одну величину размерности длины: С^2/А. Это и будет характерное расстояние. При 1g оно составит примерно 10^16 метров. Это где-то 1 световой год.
Чуть меньше 1 светового года, да. Или чуть больше 1 месяца на «12-кратных», но я не помню, читал ли эту книгу Стругацких. Хотя понятно, что если разгоняться за 12 месяцев внешней СО, то на последнем месяце будут огромные перегрузки.
Гравитационный радиус Земли чуть меньше сантиметра.
Разве это так? Ведь в центре земли невесомость, так как вещество в центре земли тянется во все стороны одинаково. Разве там есть такая плотность что бы был Гравитационный радиус? Наверно самая высокая плотность лежит чуть дальше от центра земли. Дальше — может на сантиметры, а может на метры?
Гравитационный радиус — это размер до которого нужно сжать тело, чтобы оно приобрело свойства чёрной дыры.
Ага, «Гравитационный радиус Земли чуть меньше сантиметра» — это наверно следовало читать как «если бы землю сплющили до 1го кубического см»
Да
Грав. радиус удобно вводить для любого тела со сферической симметрией, т.к. через него можно записать вокруг тела метрику Шварцшильда. Просто при грав. радиусе 1 см получится, что расстояние 9 км от уровня моря не очень сильно отличается от точки с расстоянием 10^9 км, хотя точные эксперименты с атомными часами дали выявить эффекты ОТО даже на масштабе орбиты высотой несколько десятков тысяч км (требуется учитывать в работе спутников GPS).
Есть еще много мифов, если статья понравится, я продолжу.
На самом деле хотелось бы не разбор и критику мифов, а ясный не предвзятый обзор того, что является проверенными на экспериментах моделями, а что — лишь теориями, будоражащими воображение неискушённой публики своей грандиозностью, неизведанностью, космической фундаментальностью.
Иначе всё это превратится в очередную типичную дискуссию участников собрания свидетелей Иеговы большого взрыва.
Здесь я пытаюсь ограничится только научными теориями, неправильно подаваемые Discovery и ище с ним. Про Unruh effect в комментах разве что ступил в область гипотетических построений

Из научно-популярного посмотрите на Ютубе лекции Сергея Попова по астрофизике. Он всегда очень аккуратно обращается с такими понятиями как "доказано", "гипотеза", "стандартная гипотеза" и максимально ссылается на публикации. При этом интересно рассказывает.

Простите, а как можно экспериментально проверить модель чёрной дыры или большого взрыва?
Тут наука выходит на границы возможности применения принципа Falsifiability.
И это проблема
Ну почему же? Модели черных дыр можно проверять по косвенным наблюдениям — динамике окружения или излучения, и по прямым — наблюдая гравитационные волны.
Большой взрыв до какой-то степени можно проверять по реликтовому излучению, балансу частиц и т.п. По крайней мере какие-то классы моделей это поможет отделить.
Ну это как раз никак не поможет проверить модель черной дыры. Вообще, кроме любопытного эффекта никакой пользы в сравнении аналоговых моделей с астрономическими нет.
А что с ненулевой плотностью межзвёздного газа?
На какой скорости он начнёт оказывать воздействие на корабль?
Там все плохо. Там не только газ, там любая пылинка приводит к фатальным последствиям. Собственно, любая пылинка имеет столько энергии, что ее достаточно чтобы расплавить космический корабль. Так что никаких полетов до mind upload.
И на какой скорости, такая пылинка будет иметь такую энергию? (как оценочно примерно посчитать и прикинуть?)
никаких полетов до mind upload

Почему же? Старый добрый корабль поколений — точно безопасен.
Вы классику читали? Где только 1 человек на корабле знал, как подать команду «Приземляемся в этой системе».
Пример для пылинок на 0.1c — Иван Александрович Корзников «Реальности межзвездных полетов», 2006 — go2starss.narod.ru/pub/E009_RMP.html (оригинал — zhurnal.lib.ru/k/korznikow_i_a/realxnostimezhzwezdnyhpoletow.shtml)

> Такие частицы, сильно рассеивающие и поглощающие свет, имеют размеры 0.1-1 микрон и массу порядка 10^-13 г
> При v=0.1c… Легко найти, что пройдя путь в 1 световой год, экран звездолета встретит n=rs=10^-12 · 9.46 · 10^17 =10^6 пылинок на каждый см^2, и каждые 500 пылинок сроют слой 0.448 мм экрана. Значит, после 1 светового года пути экран будет стерт на толщину 90 см…
>… не менее 5 частиц массивнее 0.1 г на все поперечное сечение звездолета. А каждая такая частица при v=0.1c имеет энергию более 4.53·10^10 Дж, что эквивалентно кумулятивному взрыву 11 тонн тротила.
> Поэтому межзвездные перелеты могут осуществляться лишь с существенно меньшими скоростями, порядка 0.01с и менее.

Немного еще есть в Pros and Cons of relativistic interstellar flight,
Oleg G. Semyonov, arxiv.org/ftp/arxiv/papers/1807/1807.08608.pdf
The interaction of relativistic spacecrafts with the interstellar medium, arxiv.org/abs/1608.05284
и из en.wikipedia.org/wiki/Interstellar_travel#Interstellar_medium

Карта местности и проекты на 300 лет — marsmeta.narod.ru/univ/unistep.html
> Заключение. При выбранных условиях: скорость звездолета 0.1·с и время перелета не больше 50 лет человечество не сможет распространить волну цивилизации дальше системы Центавра и звезды Барнарда. Теоретически возможны единичные исследовательские полеты к Сириусу, Лаланаду 21185 и Вольфу 359. Однако колонизация окрестностей этих звезд растягивается на столь большое время, что делает это предприятие сомнительным.
/trololomode
При достижении определённой скорости перестают действовать силы межмолекулярного и межатомного взаимодействия, и тело превращается в набор несущихся невзаимодействующих частиц. При это, локально, оно продолжает «ощущать» себя целым и невредимым. Далее. Поскольку этот набор несущихся частиц явно не будет обладать строго параллельными траекториями у каждой частицы и будет не сфокусирован, то тело будет расширяться и «утончатся». После определённого расстояния, даже планеты и звёзды не доставят каких-либо проблем, т.к. они будут проходить это истончившиеся тело гигантических размеров насквозь, не провзаимодействовав ни разу.
Снова Рен ТВ
Да, не. Телек не смотрю. У дураков мысли сходятся, просто.
А если серьёзно, то любое тело испариться уже на этапе разгона, а он возможен только в инерциальной системе.
Правда, я не знаю, на какой конкретно. Хотелось бы чего-нибудь по этой теме, но поиск не очень помогает.
Скорость относительна. Скорость может быть только относительно чего-то, само по себе тело не может иметь скорости и что-то там переставать действовать тоже не может.
что-то там переставать действовать тоже не может

может и даже обязано. Ни одно физическое тело невозможно разогнать до около световых скоростей из-за этого в принципе.
Разогнать не возможно. Потому что что-бы раньше лететь медленнее, а сейчас быстрее нужно тоже самое тело отсчета относительно которого летим. А так можете считать что вы прямо сейчас летите с околосветовой скоростью.
Зачем мне так считать?
Что-бы свести до абсурда изначальное утверждение
При достижении определённой скорости перестают действовать ...
Абсурда что-то не наблюдаю.
При достижении определённой скорости
относительно чего-нибудь, раз так угодно. Не думал, что тут есть такие, для кого это необходимо уточнять.
Более того, не понимаю, с чем спорите-то?
У вас там написано
При достижении определённой скорости перестают действовать силы межмолекулярного и межатомного взаимодействия
а потом вы написали
относительно чего-нибудь, раз так угодно
А фокус в том что относительно одного тела рассматриваемое тело достигло определенной скорости а относительно какого-то другого тела не достигло. Это означает что относительно одного тела силы межмолекулярного и межатомного взаимодействия перестают действовать, а относительно другого тела не перестают. Понимаете? Скорость может быть разной в разных системах отсчета. Силы же не могут быть разными.
Вроде как не просто можете, а все частицы всегда двигаются со скоростью света в четырехмерном пространстве времени.
Тогда уж лучше так, скорость — это гиперболический тангенс угла между касательными к траекториям в четырехмерном пространстве. Потому скорость может быть только между двумя. Угол не получится если прямая только одна.

И процессы между этими двумя телами зависят от скорости, камень на скорости 1 м/с об голову совсем не тоже что на скорости 100 м/с. Но вот процессы в одним и том же теле не могут зависеть от его скорости относительно какого-то произвольного другого.
Bloater Drive Гаррисона же. «При его работе расстояния между атомами корабля увеличиваются, корабль разбухает, достигая нужной точки пространства, а потом сжимается в нужном направлении.»
Совсем боян. По смыслу так же описано «подпространство» у Стругацких.
Вот серьезная статья на тему столкновения космических аппаратов с пылинками arxiv.org/pdf/1608.05284v2.pdf

Согласно ей, при скорости v=0.2c на пути длиной 4 световых года от ударов пылинками графитовый экран потеряет на испарение слой толщиной 0.7 до 2 миллиметров (в зависимости от плотности межзвездной среды в наших окрестностях). Кварцевый экран — от 0.5 до 1.5 миллиметров.

Из Рис.16 в статье видно, что при скорости v=0.1c потери на испарение для кварцевого щита будут 0.3 мм максимум.

Иначе говоря, Иван Александрович Корзников в своих оценках ошибся аж на 4 порядка вверх. Причины этой ошибки:
1) Выбор титана в качестве материала щита в его оценках (довольно нелепый выбор, так как при столкновениях на 0.1с для материала щита важна не прочность, а удельная теплота испарения).
2) Неверное понимание физики процессов при столкновении с пылинкой и, соответственно, использование для оценок потери материала соотношения, выведенного для скоростей много меньше 0.1с.

Вывод: для близких межзвездных перелетов проблема столкновения с пылинками не очень существенна. Даже для тонких аппаратов вроде Starchip — их после окончания разгона можно просто повернуть ребром по направлению движения.
На любой. Разница только в силе (грубо) воздействия.
Для того, чтобы замедление времени было бы таким, планета должна была быть на расстоянии чуть более 0.003% от радиуса Шварцшильда.

А вот П. Р. Амнуэль, ссылаясь на Кипа Торна (того самого консультанта фильма и по совместительству Лауреата Нобелевской премии по физике 2017 года), в статье Черная дыра в кинематографе, опубликованной в журнале «Химия и жизнь» (№3, 2015), утверждает, что
эффекта [...] действительно можно добиться, но лишь в случае, если черная дыра вращается так быстро, что ее горизонт событий, то есть то, что можно считать границей дыры, движется со скоростью, которая меньше скорости света всего на 0,01 миллиметра в секунду! Только тогда пространство-время вокруг нее искривляется и закручивается столь странным образом, что и время замедляется в сто раз, с точки зрения удаленного наблюдателя, и гравитационное поле уравновешивается центробежной силой, благодаря чему и тело астронавта, и корпус космического корабля не испытывают чудовищных перегрузок.
Возможно. Но также очевидно, что эта область КРАЙНЕ узкая. Сотни метров, километры. Планеты точно туда не поместится, и ее разные стороны будут в совершенно разных условиях
Ну так и у нас на Земле разные стороны в совершенно разных условиях) Это похоже на противопоставление «запрещено» против «не запрещено, но маловероятно» Торна. В этом конкретном вопросе я бы примкнул к лагерю Торна.
Только нашу планету не разрывает приливными силами, а ту — разорвёт.
Как раз добрался до книги. Приводится довольно очевидный факт: «Сила растяжений и сжатий обратно пропорциональна квадрату массы Гаргантюа(чёрной дыры)». Соответственно, всё очень просто, – для того, чтобы планету не разорвало, накладывается ограничение на минимально возможную массу чёрной дыры.
ИМХО, ЧД (родившиеся в естественных астрономических процессах) имеют определенный минимум массы. Звезда слишком маленькой массы создаст при коллапсе обычную НЗ. Можно попробовать конечно вариант, когда при столкновении 2 НЗ образуется ЧД при сильном выбросе массы. По идее, тогда ограничение по получившейся массе может оказаться ниже случая обычного коллапса массивной звезды.
Не, не. Обратно пропорциональна. Минимальное ограничение по массе, которое приводится в книге, — не менее 100 миллионов солнечных масс.
Я понял, о чем Вы. Массивная ЧД имеет большой гр. радиус.
Грубо говоря для разности течения времени между точками 1 и 2 нам нужна разница грав. потенциалов, а её отношение к разнице радиусов дает величину, равную напряженности грав. поля в какой-то точке между 1 и 2.
Само собой разумеется, что такая огромная разница течения времени между планетой и орбитой над ней невозможна. Даже если запихнуть космический корабль в точку L2 системы «ЧД-планета».
Минимальный предел массы для образования ЧД 3,6-3,8 масс солнца. Да и при слиянии двух НЗ выброса массы не будет.
Будь то чёрная дыра, или не чёрная, или не дыра вовсе, космонавтам пофиг структура и природа центрального тела, если они от него находятся на расстоянии в разы превышающем гравитационный радиус.

Вообще меня забавляет страшилка (вот про это следовало бы написать), что чёрная дыра опасна, что она всё затягивает. На самом деле она никак не отличается от ньютоновского тяготеющего центра такой же массы. Для примера сравним Солнце, как оно есть, и чёрную дыру солнечной массы (размером, ЕМНИП, 3 км). В такую ЧД ещё надо умудриться упасть, т.е. надо очень и очень старательно прицеливаться. Чуть мимо — и ваш избыточный момент импульса не даст вам упасть в такую ЧД. Получается, что пролетев в 5 км (ну ок не в 5 км, а в 100 км) от солнечной ЧД, у вас не будет никаких проблем. Если же вы нацелитесь в 100 км от центра Солнца, то вам капец. Пропадёте даже пролетая не в 100 км, и не в 10000 км, а при 1000000 км от него.

Вывод: чёрные дыры — самые безобидные объекты во вселенной. Ну по крайней мере ЧД звездных масштабов.
Только Вы можете её не заметить в плотной туманности.
В плотной туманности вокруг неё наверняка будет хорошо заметный аккреционный диск.
А разве её нельзя вычислить по притяжению? То есть определяли ведь сотни лет назад, как планеты должны двигаться вокруг Солнца, так даже крайние планеты системы нашли, то же самое можно вычислить и с черной дырой?
По притяжению мы не отличим ЧД от звезды массой 3-5 масс Солнца. Но я согласен с Tyusha в том, что не так уж опасно пролететь от такой ЧД на расстоянии 3-5 радиусов Солнца. Максимум — повышенный рентгеновский фон и гамма-кванты энергией 400-500 кэВ. Правда это я взял «мягкое» гамма-излучение. Если вокруг ЧД будет атмосфера из кислорода и более тяжелых элементов с плотностью атомов кислорода (и б.т.э.) хотя бы как в воде при н.у. — этот диапазон будет поглощаться сильнее, а скажем 800 кэВ — уже слабее.
Но ведь звезду мы будем видеть? В точке Х у нас есть что-то с массой 3-5 масс Солнца. Если это звезда — оно светится как звезда. Если черная дыра — заламывает свет. Так можно определить, что это. Я так понимаю, что мы не знаем еще туманностей в космосе, которые могут звезду закрыть, а все туманности, которые мы знаем — это один атом (к примеру) водорода на километры вокруг.
Так заламывает свет что звезда, что ЧД одинаково. Разница будет только в траектории тех фотонов, которые натолкнулись бы на плазму короны белого/голубого субгиганта/гиганта (лень искать инфу о сп. классе звезды взятой мною массы).
Может учет какой-то квантовой гравитации скажет, что на расстоянии радиуса звезды упомянутой массы грав. поле ЧД более сильно искривляет метрику, но с проверкой любого вида КТГ пока кажется проблемы.
оно светится как звезда

Вроде, нейтронная звезда практически не светится в видимом диапозоне, а некоторые виды таких звезд вообще практически ничего не излучают. Определить, где ЧД, а где нейтронная звезда можно лишь в теории по массе.
Обычные пульсары излучают радиоволны, а «во время еды» — рентген (как правило — тесные двойные системы). Но, механизм порождения рентгена, ИМХО, включает в себя ондуляторный. А ондулятор может выдавать практически любую длину волны, зависящую от периода магнитного поля, энергии частицы (gamma ~ 1 или gamma ~ (постоянная решетки железа)/(комптоновская длина электрона)) и взаимной ориентации импульса чатсицы с магнитным полем.
Это Вам и синхротронное излучение типа СВЧ, и «мягкое» рентгеновское излучение (типичное излучение ондулятора в слабых магнитных полях, при нулевом угле ориентации и gamma ~ 1 будет половиной периода, например — ферримагнитного домена). Может выйти весь диапазон между ними. Другое дело, что одно видит рентгеновский телескоп (скажем на большой высоте, где кислорода концентрация падает и длина «своб. пробега» рентгеновских фотонов растет хотя бы на 70%), а другое — радиоволновой в диапазоне СВЧ (скажем интерферометр с базой в 1000 раз больше радиуса зеркала самого крупного рефрактора).
Если бы мне сказали, что ваш комментарий написан нейронной сетью,. я бы не удивился. Хотя в том, что я ничего не понял, исключительно заслуга моего невежества. Впрочем, было очень интересно.
Аккреционный диск мы тоже будем видеть, он достаточно разогревается и производит тепловое излучение.
Там сначала не тепловое, а рентгеновское. «Тепловой» ультрафиолет будет от плазмы с температурой 7500 К, то есть немного теплее поверхности нашего Солнца. Конечно плазма с температурой kT = 5 эВ начнет больше излучать в ультрафиолете.
При чем тут ультрафиолет? Его вспоминают в связи с тепловым излучением исключительно в силу того, что такой пик теплового излучения соответствует комнатной температуре. Повысь температуру — будет пик на более высоких частотах. Тоже тепловое.
Конечно будет. До ренгена наверное сложно дойти в том смысле, что наверное не выйдет «равновесного» излучения АЧТ при такой температуре плазмы. Особенно — когда ионизация достигает «1 свободный электрон на 1 ядро» и нужно вводить отдельно «электронную» и «ионную» температуру.
Ну не такие уж и безобидные, они умеют внезапно делать пиу-пиу и все вокруг поджаривать, когда им удается зохавать что-нибудь массивное.
Более того, чем массивнее ЧД, тем она безопаснее: радиус горизонта событий пропорционален первой степени массы => градиент гравитационного поля у более массивной ЧД меньше, и существенный рост поля тяготения вблизи ГС не будет сюрпризом. Это если лететь не вслепую (в противоположном случае в более массивную ЧД легче случайно попасть).
Спасибо, было интересно почитать. Но «Интерстеллар» — научная фантастика (science fiction), искать там «научность» — дело неблагодарное, ибо см. описание жанра научной фантастики. ЗВ, опять же, научной фантастикой никогда и не являлись.
Что-то с этим утверждением не так.
Например?
Научная фантастика (НФ) — жанр в литературе, кино и других видах искусства, одна из разновидностей фантастики. Научная фантастика основывается на фантастических допущениях (вымысле, спекуляции) в области науки, включая как точные, так и естественные, и гуманитарные науки. Научная фантастика описывает вымышленные технологии и научные открытия, контакты с нечеловеческим разумом, возможное будущее или альтернативный ход истории, а также влияние этих допущений на человеческое общество и личность. (с) Википедия

Т.е. никакой корреляции с реальной наукой в НФ может и не быть. Придумали, что E = mc^3, выстроили вокруг этого придуманный мир с объяснениями и использованием этого допущения — это НФ, а просто нарисовали, что драконы летают в космосе — фэнтези.
«Более того, если Вселенная бесконечна, то она бесконечна с самого начала. Бесконечна и почти однородна. И эта картинка (и еще красивый рендер со взрывом, который любят показывать) никакого отношения к реальности не имеет. Нет никакой точки, где был взрыв, который распространялся в окружающую пустоту. Повторюсь, вселенная была почти однородной (и чем дальше в прошлое, тем более однородной).»
— точняк!!! а «дальше в прошлое»- относительно нашего видения далеких космособытий, она и была и остается однородной… бесконечно однородной, я как то мысль свою пытался выразить в плане понимания «начала» вселенной — с составляющей субчастицы атома она началась, начинается, и там же парадоксально она заканчивается, и это явление повсеместно и бесконечно однородно, как и фундаментальные законы квантовой механики.
теория же Б.В хоть и безобразна и глупа, но такова она есть, приверженцы ее не глупы, они тоже мыслители.
С этим вам на Рен ТВ )
спасибо)) может быть вы и правы не в свою тему влез))
НЛО прилетело и опубликовало эту надпись здесь
Я принципиально аппрувлю все комментарии)
Конечно. Этот человек ведь специально только что зарегистрировался на Хабре ради этого комментария — не стерпел лжи релятивистов.

UPD: Хотя может быть, Владимир зарегистрировался уже не в первый раз, кто знает.
Ну, хотя бы не долго мучался.
ну вот, зачем забанили человека, я теперь никогда не узнаю что он написал! Хоть бы заминусовали чтоб можно было прочитать, интересно же!
Не очень понял с горизонтом событий. Тело любой массы пересекая горизонт событий имеет бесконечную кинетическую энергию? Или нет? Если да, то это немного противоречит закону сохранения энергии системы ЧД, тело. Если нет, то оно может выскочить и обратно. Парадокс однако.
1. Не бесконечную
2. С понятием и сохранением энергии в ОТО все сложно… Об этом будет 2 часть
3. Пространство время внутри дыры устроено так, что выскочить невозможно вне зависимости от энергии. Под горизонтом сингулярность, к которой вы летите, оказывается не впереди, а в БУДУЩЕМ. То есть как вам надо двигаться, чтобы избежать ПОЛНОЧИ сегодня?
Про полночь понравилось. Никогда не слышала такой лаконичной и красивой аналогии.
проблемы ОТО разве не пытается решить Излучение Хокинга?
У ОТО нет проблем. :) Кроме проблем с сингулярностью. Но излучение Хокинга не про сингулярность. Да и само излучение Хокинга, если уж говорить в терминах проблем, скорее их добавляет, нежели решает. :) Информационный парадокс и всё вот это.
«пространство внутри устроено так»
С чего бы? Если черная дыра достаточно велика, то как многие утверждают, что наблюдатель даже не заметит разницы. То есть локально оно имеет те же свойства.
Локально всё пространство везде: в чёрной дыре, в белой, в «эпицентре большого взрыва» устроено одинаково, о чём нам и говорит принцип эквивалентности. Чёрная дыра порождает нетривиальные глобальные топологии, хотя в любом месте чёрной дыры (окромя сингулярности) с пространством-временем локально «всё в порядке». Наблюдатель падает в чёрную дыру, пролетел горизонт, летел себе дальше, как говорится: ничего не предвещало,… но сингулярность.
И это никак не противоречит тому, что я сказал!
Локально все выглядит нормально. А в будущем у вас сингулярность

Вот так это выглядит в координатной системе Крускала, она наиболее близка к тому, как воспринимает мир наблюдатель, падающий в черную дыру
en.wikipedia.org/wiki/Kruskal%E2%80%93Szekeres_coordinates
Две вселенных и две внутренние области одной дыры в этой диаграмме безусловно многое обьясняет. Правда белые дыры не обнаружены… Видимо прячутся в других вселенных.
Диаграмма конечно даёт вменяемую математическую модель траекторий, но что там есть на самом деле вроде никто не знает
две вселенных возникают в идиализированной картине вечно существующей черной дыры, у которой должна быть белая пара.

Реальные черные дыры, сформированные коллапсом, выглядят иначе и белой черной дыры и негативного пространства там нет: www.semanticscholar.org/paper/26.2-Schwarzschild%E2%80%99s-Spacetime-2/bbe516a300e3cf212ae6a08783509fb1942b2400/figure/5

Для вращающихся черных дыр все хуже, kerr metrics является «идеализированной» метрикой вечно существующей черной дыры, а реалистичного решения нет
Извиняюсь за надоедливость, но как оценить кинетическую энергию падающего пробного тела на горизонте? Очевидно она прямо пропорциональна массе пробного тела. А какой второй множитель? Если она конечна, то ее можно посчитать???
Это будет во второй части статьи)
Спойлер (я догадалась, догадалась, только тс-с-с-с...)
Энергия Вселенной не сохраняется.
Тише!!! )))
Спойлер про Вселенную.
Она 5 миллиардов лет расширяется с ускорением, если верить модели LambdaCDM. Странно было бы пробовать описать такую Вселенную сохранением энергии. Получаем явную неоднородность времени.

P.S. Немного не по теме
Про законы сохранения. Кто-то в курсе — на законы сохранения барионного и лептонного заряда действительно нет порождающей их симметрии?
Хотя глянул на Вики — для сильного и электрослабого можно ввести калибровочные поля.
на законы сохранения барионного и лептонного заряда действительно нет порождающей их симметрии?

SU(2)xSU(3)
Ну значит я плохой источник раньше нашел. Но люди все равно пытаются ограничить сверху вероятность невозможных процессов, вроде распада
Z -> p + e, mu (нарушение з.с. сразу лептонного и барионного заряда).
Да, ещё большее ограничение (то есть более слабое) на распады
Z -> mu + anti-tau
Z -> anti-mu + tau
SU(2)xSU(3) это про слабое и сильное взаимодействия как раз. Где тут сохранение лептонного заряда?
В стандартной модели есть локальные калибровочные и глобальные калибровочные симметрии. Из локальных получаются взаимодействия и сохранения, конкретно благодаря SU(2) сохраняется слабый изоспин, а благодаря SU(3) цвет. Из глобальных взаимодействия не получаются, но благодаря им сохраняется число лептонов. Это обычно U(1) (кажется только они), глобальные фазы.
Ээ… А есть чем подтвердить слова?

Насколько мне подсказывает склероз, калибровочные симметрии все локальные. Глобальная симметрия U(1) сидит в основаниях КМ, например, физическому состоянию соответствует не просто вектор Гильбертова пространства, а луч — умножение вектора на комплексное число единичной «длины» дает все то же состояние, т.е. в рамках U(1) сколько хошь умножай глобально, ничего не изменится. Вот если локально U(1) симметрию требовать, там уже есть эффект — получается электрический заряд и его сохранение.

Вы правы, я затупила. Локальная калибровочная инвариантность даёт только сохранение электрического заряда, но не лептонного или барионного.

Ага, там нужно читать
В стандартной модели есть локальные калибровочные и глобальные калибровочные симметрии.
Симметрии есть, например, ЕМНИП, сильное вз-ие никак не зависит от аромата кварка. Ботомоний имеет энергию выше чармония по 2 причинам:
1. Естественная энергия покоя кварков.
2. Наверное, как и в случае с мюоным водородом, энергия состояния растет с ростом приведенной массы частицы «на орбите». Может не прямо линейно, а скажем у пиона почти 95% массы идет на взаимодействие 2 кварков (правда это выражается через глюоны и вирт. пары «кварк-антикварк»).

P.S. Есть план мюонного коллайдера. Может где-то и был протон-пионный, но пион с временем жизни 26 нс будет сложно родить, выделить и разогнать.
Главный вопрос не «как» оценить, а в какой системе координат. Для удалённого наблюдателя падающая частица остановится на горизонте, и кинетическая энергия у неё будет почти нулевая. В то же время наблюдатель на горизонте воскликнет: о какая чёртова прорва кинетической энергии была у этой частицы, только что промчавшейся мимо!
На горизонте нет наблюдателей :(. Но если пробное тело достаточно крупное то будет видно по изменению массы черной дыры
Да а с точки зрения удаленного наблюдателя, если оно остановится на горизонте, то его кинетическая энергия исчезнет неизвестно куда
Понятно куда: падение частицы в ЧД — это неупругое столкновение. Часть кинетической энергии в виде отдачи получит ЧД, но это доля крайне мала, в виду несоизмеримости масс. Остальное (а по факту вся) кинетическая энергия частицы, которую она имела вдали от ЧД, добавится к Mc^2 чёрной дыры. Т.е. масса покоя ЧД увеличится больше, чем если просто приплюсовать массу упавшей частицы. Мораль: энергия сохраняется, никакой магии.
Вот как раз и был вопрос, а насколько именно больше. В два раза, в три?
Ну а если пробная масса достаточно велика, например ещё одна черная дыра, то откуда возьмётся этот константный фактор и куда он потом денется?
Можно конечно сказать, что эта энергия уже была в этой системе из двух тел, но тогда суммарная масса двух черных дыр больше суммы их масс…
Если частица падала из бесконечности с нулевой скоростью (без кинетической энергии), то конечная масса ЧД — прямая сумма масс.
В два раза, в три?

Во сколько раз кинетическая энергия частицы на бесконечности больше её массы покоя, во столько и будет.
Если речь о Bondi или ADM mass и первоначально оба тела были в покое, то суммарная масса равна сумме масс МИНУС масса излученная в виде гравитационных волн
Возьмём с Земли кирпич. И утащим его в бесконечность. При этом его масса увеличится, ведь мы вытащили его из потенциальной ямы? Примерно как энергия связи в атомном ядре. А если брать более глубокую потенциальную яму, то масса кирпича/пробной частицы выродится на горизонте. Я правильно понял?
Масса кирпича не изменится, кирпич — он и в чернодырной Африке кирпич. Увеличится суммарная масса системы «Земля-кирпич», т.к. в неё закачали энергию при вытаскивании кирпича на бесконечность. И ещё раз повторю. Кинетическая энергия зависит от системы отсчёта, поэтому энергия частицы на ГС для удалённого наблюдателя нулевая, т.к. она остановилась возле ГС; для местного (на ГС) наблюдателя — энергия частицы какая-то большая.
Суммарная масса системы имеет своих двух представителей: Земля и Кирпич. Увеличатся массы обоих и Земли и и Кирпича. Энергия системы где то должна находиться. Закачали в систему энергию — получите добавочную массу. Вытащили энергию — получите дефект массы. Посмотрите на ядерные реакции.
Энергия системы где то должна находиться.

Никогда не задумывалась над вопросом, куда падающий с крыши кирпич складывает свою нарастающую кинетическую энергию. Он же кирпич у него внутри нет места. :)

Энергия системы существует в виде искрящихся синих молний только в кино. Для нас энергия (масса) — это всего напросто число, характеристика, которую внешний наблюдатель приписывает системе при механических взаимодействиях. В этом смысле энергия существует лишь в воображении того, кто выполняет расчёты. Он её придумал для своего удобства, это же модель.

UPD: А ещё не понятно, как кирпич в процессе падения успевает проинтегрировать действие по всем возможным траекториям, отыскать минимум, и именно согласно ему падать. У него же даже мозгов для этого нет.
Пока он не стукнулся, с его массой все в порядке, она постоянная, его масса покоя как бы уменьшится на его кинетическую энергию, то есть сумма массы покоя и релятивистской массы останется как его полная энергия, но при ударе он ее потеряет в виде тепла/ излучения. Суммарная энергия при этом сохраняется, а его масса покоя уменьшится
У вас путаница в терминах. Масса покоя — это масса покоя, они ни о чего не зависит.
но при ударе он ее потеряет в виде тепла

Нет. Горячий кирпич (как замкнутая система) весит больше холодного.
Он остынет. И будет весить меньше. В данном случае гравитация обеспечивает отрицательную потенциальную яму. То есть его измеряемая масса будет его масса в бесконечности минус энергия связи. Вам ниже для аналогии атомные ядра.
Энергия системы где то должна находиться.

В гравитационном поле, например.

Интересная штука получается. Понятно, что энергия хранится в поле. Но тогда получается, что у этого поля есть масса, а отсюда (по аналогии с другими полями) вытекает наличие гравитонов как носителей этой массы.
Прочитайте вторую статью, продолжение
Энергия гравитационно поля неопределена
На Вики вообще написан обратный факт. Конечно с разработкой КТГ пока много проблем, но там утверждается, что поле с носителями спина 2 обязано давать эффекты ОТО. Это конечно без микроскопических эффектов квантовой теории поля, но утверждение выглядит весьма сильным.
P.S. У поля может не быть массы покоя, будет только импульс и кин. энергия. А с другой точки зрения (не смогу точно доказать это утверждение) есть ограничение на массу гравитона, как типичного носителя грав. поля.
Причина этого в том, что массивный переносчик поля (типа W-бозона) обязан вызывать экспоненциальное падение потенциала с расстоянием, см. подробнее на Вики на пример потенциала Юкавы.
Это где на Вики?
en.wikipedia.org/wiki/Gravitational_energy#General_relativity

In general relativity gravitational energy is extremely complex, and there is no single agreed upon definition of the concept.… Addition of the matter stress–energy–momentum tensor to the Landau–Lifshitz pseudotensor results in a combined matter plus gravitational energy pseudotensor that has a vanishing 4-divergence in all frames — ensuring the conservation law. Some people object to this derivation on the grounds that pseudotensors are inappropriate in general relativity, but the divergence of the combined matter plus gravitational energy pseudotensor is a tensor.
Может я ошибся веткой, по идее отвечал для Gutt.
Сейчас не нашел конкретно того утверждения, нашел пока такое.

А по поводу энергии на Вики есть такое:
Проблема энергии
В ОТО оказывается, что величина

$ t_{\nu }^{\mu }} $


не может быть тензором, а представляет собой псевдотензор — величину, преобразующуюся как тензор только лишь при линейных преобразованиях координат. Это означает, что в ОТО энергия гравитационного поля в принципе не может быть локализована (что следует из слабого принципа эквивалентности).
Подобное происходит с массой нейтронов в большинстве стабильных ядер (а нет, во всех стабильных). Как минимум — энергия конфигурации нуклонов с условием «нуклон №N = нейтрон» оказывается ниже, чем у систему «нуклон №N = протон + где-то болтается электрон + нужно компенсировать импульс антинейтрино».
Но части протонов в нестабильных ядрах вообще не везет — либо протон слишком высоко от «хорошей пот. ямы» и может породить нейтрон, позитрон и антинейтрино, либо не так высоко, но может столкнуться с 1s-электроном в реакции
p + e -> n +nue
Например это происходит с вероятностью выше 10% за 1.25 млрд. лет с атомом калия-40.
Мне всегда было интересно: если при пересечении горизонта событий пространство приобретает свойства времени (все пути внутри ведут в сингулярность, так же как все пути снаружи ведут в будущее), значит ли это в некотором смысле, что время становится похоже на пространство? Например, можно ли двигаться назад во времени (что бы это ни означало внутри горизонта)?
Смотрите на диаграмму Крускала. Конусы действительно ложатся набок, будущее оказывается не вверху, а вбок, к сингулярности, которая оказывается в будущем. Однако это только с точки зрения стороннего наблюдателя — для локального падающего наблюдателя все по прежнему, пространство трехмерно, время одномерно, «No drama».
То есть как вам надо двигаться, чтобы избежать ПОЛНОЧИ сегодня?
Так просто быстрее скорости света. Разве нет?
Для «большого взрыва» я для себя придумал такую аналогию: возмём пресс с большими (бесконечно большими) поршнем и дном. Поршень прижат ко дну, пространства нет. Откручиваем винт и — бабах — у нас по всему пространству под поршнем одновременно появляется эээ пространство. Осторожно, дальше аналогия перестаёт работать.
А вот у меня вопрос: сингулярность в чёрной дыре зачем придумана? Она точно нужна?
Она не придумана, она попала туда из математики, точка, в которой невозможно вычислить значение функции. В случае физики — результат бессмысленный. В школьной физике, где тела моделируются материальными точками вы можете получить сингулярность, вычисляя силу гравитационного взаимодействия между двумя точками сближающимися точками, когда расстояние между ними станет равным нулю.
Не вспомню где (к сожалению), но мне как-то попадалось утверждение, что сигнулярность нельзя пронаблюдать. Следовательно, физическим смыслом она обладать и не обязана, вполне может оставаться этаким математическим артефактом.
У чёрной дыры есть масса, материя составляющая эту массу должна находиться в какой-то форме. Она не может быть просто математическим артефактом.
Так никто и не спорит, что у дыры наблюдаемые физические параметры таки есть. Но у сингулярности, которая скрыта где-то внутри дыры, они теряют смысл.
Понятно. Мы на разных языках говорим. Перевожу: Сингулярность это не волшебный объект, Тот-Который-Нельзя-Наблюдать. Это свидетельство неполноты наших физических моделей. Поскольку чёрные дыры принято считать существующими, получается у нас есть (внутри чёрной дыры) некий реальный объект, описать который известными нам законами природы мы не можем. А те, которыми можем (вроде М-теории) находятся на стадии гипотезы и не понятно насколько физичны.
Сингулярность — результат решения уравнений Эйнштейна. Тут можно было бы поставить точку, но на мой взгляд не всё так очевидно.

Во-первых, решения Шварцшильда, Керра и других сделаны в предположении, что метрика стационарна (для Шварцшильда статична). Т.е. это решения для ЧД которые «существовали всегда». А в реальных ЧД это заведомо не так, есть фаза коллапса. Решение с коллапсом не найдено, и вряд ли будет когда-либо найдено. И нельзя быть до конца уверенной, что в таком решении обязательно будет сингулярность.

Ну и во-вторых, что там с единственностью решения уравнений Эйнштейна? Насколько я знаю, глобальная единственность решения задачи Коши для них не доказана.
Совершенно верно
Если для невращающейся коллапсирующей звезды есть численные решения, то для вращающейся их нет (изза возникающих временных циклов, которые не дают возможности применить даже численные методы!)

Наконец, есть еще один прикол. Помимо ОТО есть теория Эйнштейна-Картана, являющаяся расширением ОТО. Экспериментально они совпадают, экспермиентальная проверка разницы далеко за пределами наших возможностей. Так как вторая теория сложнее, то бритва Окамма вроде как ее вырезает. Но теория Картана решает столько теоретических проблем (и избегает сингулярностей!) что вполне возможно что верна как раз она

en.wikipedia.org/wiki/Einstein%E2%80%93Cartan_theory
Точно за пределами:
Эффекты неметричности в данной теории являются настолько малыми, что ими можно пренебречь даже в нейтронных звёздах.


Нужно хотя бы пронаблюдать слияние НЗ заранее и словить грав. волны от этого события.
Кстати, это одна из причин, почему невозможны статические решения Вселенной — она бы просто стала черной дырой


И только прочитав это, я понял, что ни хрена раньше не понимал в красоте Вселенной.
Вы перепутали планеты в Интерстеллар.

На фото в статье изображена ледяная планета доктора Манна, на которой не наблюдалось замедления времени.

А планета-океан Миллер, где 1 час шёл за 7 лет, находилась гораздо ближе к Гаргантюа:

Похоже вы правы. Сути правда это не меняет) Объяснение «на планету наложено заклятье» куда более правдоподобно
Сам Нолан не скрывал, что «в некоторых местах мы смухлевали» (цитата не точная). С одной стороны в фильме много несоответствий современной физике, гораздо больше, чем Вы упомянули. С другой стороны, история получилась захватывающей и лично я Нолану все вольности простил. Одна из моих любимейших кинолент последних лет.
А я просто ржал, когда смотрел. Уровень реалистичности еще ниже чем в «Гравитации»
Про реалистичность в «Интерстелларе», в принципе, говорить не приходится. Хотя расчёт внешнего вида чёрной дыры с гравитационным линзированием, по слухам, привёл Кипа Торна к научным открытиям.

Но фильм всё равно приобщает к знаниям, его заслуга в том, что неискушённый зритель получает яркое представление о таких понятиях как ОТО, сверхмассивная чёрная дыра, кротовая нора, экзопланеты, 4-мерное пространство-время, колония О'Нила и пр. Во-многих местах создатели фильма соврали, и, тем не менее, на данный момент это самое удачное кино, знакомящее рядового зрителя с вышеперечисленными концепциями.
По абсолютному значению — да. «Знаменатель» у них разный.
Интерстеллар — фантастические фильм о каких-то далеких черных дырах, где, к тому же, и воовсе вводится дополнительная сущность (5и-мерные люди), никак не описанные реальными известными законами физики, поэтому тут фантазия автора.

Гравитация — там, фактически, нет fiction, контекст абсолютно реальный. И тем больнее ударяет по здравому смыслу пренебрегавшие простейшими законами физики, и количество фейспалмов при просмотре куда больше. Маленький челнок, взлетающий с планеты в интерстеллере не так сильно заметен на общем фоне фикшена, как гребанный Джорж Клуни, висящий на тросе в невесомости под действием какой-то неведомой силы.
Вроде уже 100500 раз разобрали, что Клуни висел на тросе под действием центробежной силы, ибо в космосе практически невозможно не вращаться, вероятность иметь ω = 0 равна нулю, поэтому там всё разлетается при любой возможности.
Вы привели уравнение замедление времени для стационарной шварцильдовской черной дыры. Справедлива ли эта же формула для вращающихся черных дыр? Я слышал, что Кип Торн в книге " Интерстеллар. Наука за кадром" привел в обоснование свои расчеты. Сам не читал, врать не буду.
Эта картинка ни о чём не говорит. Не факт, что планета близко к ЧД, она просто находится почти на одной линии «с фотографом». Если бы планета была бы близко к ЧД, как кажется, то это значит, что у неё большое наклонение, а значит она пересекает аккреционный диск, где бы ей «очень поплохело».
В принципе да, если отлететь от планеты Манна подальше и поиграть с фокусным расстоянием, то и в этом случае можно получить фото в стиле «крошечная планетка на фоне огромной звезды».

Но то, что на планете Миллер плещется водичка, а на планете Манна даже облака замёрзли, уже свидетельствует, что первая поближе к чёрной дыре, чем вторая.
Про однозначное соответствие между разницей течения времени и силой гравитации я уже писал.
Ещё упомяну первую теорему о среднем. Только я её почему-то запомнил в форме функции F из раздела «Замечание».
То есть где-то на пути между 2 точками будет такая, в которой сила гравитации (градиент от грав. пот. со знаком минус) будет равна разнице грав. потенциалов, деленной на расстояние между точками на направлении градиента.
НЛО прилетело и опубликовало эту надпись здесь
Перечислю известные мне модели:
1. Eternal inflation — baby universes это островки обычного вакуума в поле сверхплотного. Ответ: пространство такое же как у нас.
2. Эволюция черных дыр — каждая вселенная возникает из черных дыр предыдущей (В теории Эйнштейна Картана) — примерно тоже
3. Циклический Big Rip — тоже наше пространство
4. Туннелирование «из ничего» — ответ: N/A, до Big Bang ничего не было
5. Симметрия относительно t: U(t)=U(-t), ответ: понятие «до Big Bang» не определено
НЛО прилетело и опубликовало эту надпись здесь
НЛО прилетело и опубликовало эту надпись здесь
en.wikipedia.org/wiki/G%C3%B6del_metric
Это точное решение уравнений ОТО
Но увы, нереалистичное
А как же версия, предложенная Ниайешем Афшорди, о том, что наша Вселенная является трёхмерной браной, получившейся в результате коллапса четырёхмерной звезды в четырёхмерную чёрную дыру?
Добавляем)
Если не брать в расчёт концепции мультивселенных, когда есть материнская вселенная, а мы — лишь грыжа на ней, то пространство и время возникли при БВ. Соответственно вопрос: «что было до», лишён смысла. Это всё равно что спрашивать: что находится к северу от Северного полюса?
НЛО прилетело и опубликовало эту надпись здесь
Мы вам не скажем: кто мы… (в сторону: Клюшку номер 9, пожалуйста)

– Я слыхивал о вещах похлеще, – сказал Форд. – Где-то я прочел об одной планете в седьмом измерении, которую использовали как бильярдный шар в каком-то межгалактическом баре. Так вот, этот шар загнали в лузу, а лузой служила черная дыра. Погибло десять миллиардов разумных существ.
– С ума сойти! – ужаснулась Мелла.
– Зато этот удар принес тридцать очков, – заметил Форд.


"Автостопом по Галактике"

могу ошибаться но мне кажется что вы подходите к этому вопросу эмпирически. проблема в том что наши чувства и мышление (включая логику) образовались на планете и в этой вселенной. а ваш вопрос находится за пределами системы. эмпирические его не взять, он вне привычного опыта. тут только математика спасает. но если кратко у нас нет никаких наблюдений момента До. мы не можем заглянуть в точку пространства-времени которая находиться вне нашего пространства-времени. условно можно сказать что нашего пространства-времени не существовало до взрыва. пока мы можем только проводить аналоги (как с чд) или делать выводы из математики. но все это лишь возможные варианты. доказать их внутри системы нельзя. тупо нет информации. мы можем найти возможные решения как могла образоваться вселенная идентичная нашей, но не можем выяснить точно как образовалась наша вселенная.

Что пространство и время нашей Вселенной возникли 14 млрд лет назад не отменяет вопроса — а что было до этого события? Просто нужно перейти к понятиям вечности и бесконечности, в которые наша однажды возникшая Вселенная не вписывается. Так и вопрос «что находится к северу от Северного полюса?» приобретает смысл, если поместить Землю во внешнее пространство, имеющее в своей координатной сетке своё «северное направление».
Тогда в вечном и бесконечном сверхпространстве вселенных, соразмерных нашей, должно возникнуть очень много, если мы не исповедуем вселенский геоцентризм в духе Горькавого. Эта идея плодотворна и, главное, проверяемая. Только логика и научные знания…
Так что черную дыру можно сделать из любого материала, не сжимая его — из воды

И как это будет выглядеть из воды? И каких размеров будет?

Огромных размеров. Но это масса меньше 1 миллиарда масса Солнца (сам не считал сейчас, поверил инфе на Вики). Гравитация на поверхности достаточно большая.
А разве из этого не следует, что наша вселенная должна быть чёрной дырой? Ну или стать ею в какой-то момент.
Любые идеи про Вселенную такие, что не понятно, ч то там придумали физики с сингулярностью.
Вселенная в период времени t <= tPlanck расширялась экспоненциально и выросла до размера явно больше грав. радиуса. Только 5 миллиардов лет назад ТЭ стала расширять Вселенную с ускорением, то есть это член уравнения можно откинуть до времени t = tрекомбинации.
Хотя Вы задали правильный вопрос. Но Вселенная очень разрежена. 1 масса Солнца в сфере радиусом 3 км — это не ЧД. Правда и белый карлик нельзя сжать в такой радиус естественным процессом. Масштабируем дальше. 1 миллиард масс Солнца в сфере радиусом 2 950 000 000 км — это ЧД. Это нужно загнать в сферу радиусом с большую полуось Урана миллиард масс Солнца, то есть грубо говоря — «кубик» из прижатых друг к другу Солнц с ребром 1000 диаметров Солнца.
Ещё в 1000 раз больше — уже больше массы нашей Галактики, а до соседней галактики огромное расстояние (около 50 кпк до ближайшей «не карликовой» галактики). Дальней шее увеличение масштабов Вселенной тоже весьма высоко. В радиусе 1 Мпк есть ещё 2 спиральные галактики, самый далекий объект Местной группы — 3.4 млн. световых лет.
Дальше масштабы опять растут. Соседнее скопление галактик — от 15 до 22 Мпк (как узнал — весьма большая штука, 1300-2000 галактик). Но все равно выходит не более «1/3 галактики» на кубический Мпк (оценка плотности местного сверхскопления), где галактики как правило очень маленькие, редкие достигают 10-20% массы Млечного пути.
Финальный уровень масштаба — гигантские войды, галактические «нити» и «стены» (порядка и более 1 миллиарда св. лет).
Может, вы сможете ответить на пару вопросов, на которые я не смог нагуглить ответы.
Почему радиус черной дыры не зависит от положения наблюдателя? На горизонте событий вторая космическая скорость равна скорости света, т.е. досветовой скорости недостаточно, чтобы улететь в бесконечность. Но чем ближе наблюдатель — тем меньшую разность гравитационных потенциалов нужно преодолеть, чтобы до него добраться. Получается, что некоторая точка пространства может быть внутри черной дыры с точки зрения далекого наблюдателя, но снаружи с точки зрения наблюдателя поближе.
И второй вопрос, как следствие первого. Получается, что информацию из черной дыры можно получить, используя цепочку ретрансляторов, по принципу связи с обратной стороной Луны.
Я догадываюсь, что это не так, но не понимаю, почему.
Если бы не было никакого ОТО, то частица материи могла бы летать по эллиптической орбите, которая в перигелии падала под горизонт, а в афелии могла бы отдаляться в 10 или 100 раз дальше от центра ЧД. Но ОТО говорит, что все такие траектории замкнуты внутри самой сингулярности.
Смысла придумывать для описания окрестностей ЧД комбинацию «СТО + теория грав. Ньютона» кажется нет, т.к. такая теория описывалась бы наверное «вредным понятием» релятивистской массы, которая есть «равной гравитационной и инертной».
Более точную теорию уже упоминал — объединение уравнения геодезических и метрики, удовлетворяющей уравнению Эйнштейна.
Кажется на GT уже была статья. На более глубоком горизонте перестает работать само ОТО и траектории частиц перестают быть однозначными.

О. Вопрос. Можно ведь не набрав орбитальной скорости по параболе выскочить за пределы орбиты и упасть назад. Горизон событий у нас ровно на сфере где орбитальная скорость равна световой, так ведь. Получается можно не достигая скорости света взлететь с черной дыры, набрать в пике высоту выше чем горизонт событий и послать оттуда сообщение. И потом упасть вовнутрь. Или я где- наврал?

По формальному определению грав. радиус — это когда вторая космическая равна скорости света. Но само понятие о возможности ввести понятие «вторая космическая» в ОТО чуть сложнее (см. мой коммент над Вашим, на 1 минуту раньше). А с «первой космической» в таком простом определении тоже проблемы. Если мы построим динамики СТО в поле гравитации Ньютона, то формально можно будет приравнять грав. силу (GMm/r2) к центробежной с учетом «поперечной массы»:
GMm/r2 = gamma*m*v2/R,
да, я тут написал полный бред, но хотя бы как очень грубое приближение можно взять.
Более точно — вот тут, первый член в формуле для силы.
Но этот фокус работает только в том случае, если сила гравитации не растет с ростом «инертной массы», а это нарушало бы принцип эквивалентности.
В Ньютоновской теории так и получается примерно, но вот уже 100 лет ясно, что она некорректна, а работает ОТО, где определение горизонта событий не имеет отношения к второй космической, и подняться с него нельзя даже на миллиметр. Там чисто геометрически выходит так, световые конусы все «загибаются внутрь» на горизонте и ниже.
Таким образом, увеличивая масштаб любого тела, мы всегда дойдем до состояния, когда гравитационный радиус «догонит» настоящий. Так что черную дыру можно сделать из любого материала, не сжимая его — из воды, ваты, газа.

А что удержит такой объект от гравитационного коллапса?
Собственно ЧД возникает не потому что материя плотно упаковывается, а материя плотно упаковывается потому что гравитация пересиливает все остальные силы.
А так интересная концепция — можно себе вообразить невероятно массивное скопление галактик, которые для внешнего наблюдателя выглядит как ЧД, а внутри состоит из самого обычного вещества, звёзд, газа, планет, они все вращаются вокруг друг друга…
Наблюдаемая вселенная как раз подходящий размер и плотность имеет…
Увеличим радиус Земли в 10 раз (оставив плотность материи такой же). Такая суперземля будет в 1000 раз тяжелее. Гравитационный радиус, соответственно, тоже увеличится в 1000 раз, а объем внутри гравитацонного радиуса увеличится в миллиард! раз. То есть в 1000 раз больше материи нам надо запихать в миллиард раз больший объем, то есть теперь сжимать материю надо в миллион раз меньше.

Земля КУБ???? Может в расчетах стоит использовать число «ПИ»?

Попробуйте на бумажке. При скейле и вычислении относитеньго роста объема и радиуса пи сократится

Но не только объем куба вырастет в 1000 раз, если радиус увеличить в 10 раз, объем шара тоже.

Даже если использовать, то это не играет никакого значения в данном контексте.

Про чёрные дыри и о том, что же происходят при падении на неё мне больше всего нравится объяснение Сусскинда.


НЛО прилетело и опубликовало эту надпись здесь

Так как гравитация конечна всюду, кроме сингулярности, то и ускорение там тоже конечно. Горизонт событий не связан с величиной гравитации/ускорения, он связан с [не]возможностью коммуникации.

Известно, что никаким экспериментом невозможно отличить ускорение от гравитации.

Но это только для материальных точек. А в реальности пользуемся тем фактом, что поле гравитации — сферическое, а поле ускорения — линейное: ходим из угла в угол кабины, в которой нас заперли, с отвесом — в поле гравитации отвес везде будет указывать на центр тяготеющей массы, а в ускоряющейся кабине — везде строго "вниз".

Отличить нельзя только локально. Если наблюдателю разрешить немного двигаться в стороны, то он легко отличить ускоряющийся в космосе лифт от стоящего неподвижно на земле. Во втором случае при сдвиге на метр вправо или влево угол отклонения отвеса будет меняться, т.к. он показывает точно на центр планеты.

Тут вопросо в том, использовали ли отвес при изготовлении пола лифта. Вдруг у него радиус кривизны поверхности 6371 км?
Тогда при замерах в ускоряющемся лифте будут получаться «интересные» результаты.
Тогда у кабины будут разные размеры пола и потолка, или наклонные стены, или какие-то ещё искажения геометрии
Конечно разные. Например — радиус кривизны поверхности пола будет на 3 метра меньше, чем у поверхности потолка. Стены ровные, то есть под углом скажем 1/6371000 радиан (это почти 0.0324 угловых секунд, сможете измерить без постройки интерферометра из стен лифта?).
Но я вообще-то не имел в виду такое. Мы используем литье в форме размером скажем 2 на 2 метра, чтобы «отлить» 2 листа жести (одинаковых). Ну то есть не жести, а чего-то более прочного, что не прогнется хотя бы под собственным весом.
Так углы отклонения отвеса будут того же значения, правильно? В обсуждении предполагается, что наблюдатель способен их измерить, иначе этот тред не имеет смысла.

В постановке вопроса о равноценности инерции и гравитации предполагается, что у запертого в лифте наблюдателя есть любые измерительные приборы с фантастической точностью измерения, а вот выйти/каким-либо образом выглянуть из лифта он принципиально не может.


Поэтому описание лифта Энштейна начинается с постулата, что лифт "висит над какой-то гравитирующей массой в однородном поле" — каковое, как идеальный мужчина, не существует.

Конечно способен, но с помощью отвеса. В смысле он может понять, что стены слегка расширяются, но это вроде как не очень важно.
Но вообще-то смысл замечания к принципу эквивалентности (сильному) у меня в том, что невозможно создать схему ускоренного движения НСО, которая была бы эквивалентна даже такому простому полю гравитации, как поле однородной (или хотя бы описанной плавной функцией ro = ro ( r )) сферы.
P.S. Под плавной функцией плотности в контексте ОТО будем иметь в виду такой тензор энергии-импульса, чтобы возможные разрывы при расчете тензора Римана устранялись.
Объясняю, о чем речь
Вот формула тензора.
Пуская у нас полностью равны друг другу ков. производные в первых 2 слагаемых для любых пар (ro, sigma) и (mu, nu) в точках, где разрыв функции давал бы бесконечность и аналогично — при суммировании последних 2 произведений по lambda все проблемы вычисления символов Кристоффеля устраняются для любых пар (ro, sigma) и (mu, nu).
Спасибо за статью, тема интересная, сам увлекаюсь и ищу материалы по этой теме. Тема огромная и неисчерпаемая, одних теорий возникновения Вселенной десятки.

Жду продолжения.
«Когда видимая Вселенная была не больше размера атома» — Была ли на момент взрыва вокруг этой сингулярности пустота, или бесконечное однородное пространство-время, было ли вокруг бесконечное количество подобных пузырьков или вся вселенная это статическая запись на неком абстрактном компьютере/хранилище информации — это вопрос скорее филосовский. У нас нет и скорее всего не будет возможности «заглянуть за грань», получить какую-либо информацию подтверждающую одну из теорий. По крайней мере пока не сможем выйти за пределы «нашей вселенной».
Я это к чему, эта картинка имеет отношение к нашей реальности. Да БВ не отменяет того что вокруг пузырька что-то существовало, что и как и существовало ли — науке это неизвестно. И вряд ли будет известно. Упираемся в скорость света и стрелу времени. Если рассматривать только нашу вселенную то это не «миф», а рассматривать ее как часть чего-то большего — имхо это уже за пределами теории большого взрыва, как и причины возникновения этого пузырька. Вы верно заметили, только t>0. Да может показаться софистикой, но это вопрос что рассматривать в качестве доступного обьекта изучения.

Чаще встречается другое заблуждение этой теории в «обывательском восприятии», что материя просто разлетается очень быстро в неком обстрактном и постоянном пространстве и абсолютном времени. Буквально как граната рванула и осколки разлетаются. Сложно представить раздувание пространства, если сложнее раздувание пространства-времени.
Насколько малой может быть черная дыра, если пространство все-таки неквантуемо? Какие есть ограничения, помимо необходимости огромного количества энергии для того, чтобы целенаправленным выстрелом на более мощном, чем существующий, коллайдере столкнуть частицы в одной точке и перешагнуть порог плотности в момент столкновения для рождения микроскопической черной дыры для лабораторного изучения?

Независимо от того, какой окажется физика пространства, спектр масс ЧД ограничен снизу минимальным значение порядка планковской массы (1 нанограмм) и дальше масса дискретно растёт как корень из N (некое квантовой число, описывающее состояние ЧД). Такие результаты получаются даже без теории квантовой гравитации, достаточно теории поля и ОТО.


ЧД не стабильна к переходу в состояния с меньшим N. Но без квантовой гравитации не может быть решён вопрос со стабильность наинизшего энергетического состояния. Другими слова, остаётся ли "огарок" от ЧД после хокинговском излучения, или же она полностью рассасыаается.


Ваш вопрос про образование ЧД при высокоэнергетиче ких столкновениях отвечу так: никаких фундаментальных запретов на это нет, и я внутренне уверена, что это возможно. Другое дело, что "нарастить мясо" на такой ЧД не получится из-за её быстрого распада до основного состояния (из-за излучения Хокинга). А сечение взаимодействия малых ЧД с веществом нереально исчезающе мало (нейтрино взаимодействуют с матерей на порядок порядка сильнее). Так что быстро накидать в новорождённую ЧД частиц, чтобы она подросла, вряд ли возможно.


Поэтому того, что в ЦЕРНе родят ЧД, которая разрастётся и поглотит Землю, бояться не стоит. Даже если там и родится микроЧД, то она просто провалится к центру Земли, где и будет летать по эллиптической орбите, т.к. вещество для неё прозрачно. Ну или улетит в неведомую даль космоса, если у неё окажется существенный импульс.

насколько я понимаю, в космосе есть излучение с куда большими энергиями, нежели способен дать коллайдер. и если такое и происходит, то явление не такое уж что и редкое.
Столкновение 1.22*109 частиц с энергией 10^10 ГэВ в сфере радиусом в планковскую длину — это по Вашему очень вероятное событие? Может Вы не понимаете масштаб величин.
Планковская масса — 2.176*10-5 грамм, то есть 1/46 кубического миллиметра воды.
На LHC пробуют найти сверхмалые ЧД, может они возможны только в многомерных теориях гравитации. В доступном диапазоне энергий (до 10 ТэВ) процесса «испарения» таких ЧД не замечено, может просто статистики не хватает. Это на 6 порядков меньше упомянутой мною выше энергии частицы космических лучей, участвующей в гипотетичесокм рождении ЧД.
Да, есть ещё 1 проблема. Из протонов Вы такую ЧД не соберете, их оттолкнет электростатический потенциал штуки «0.02 мг протонов» с расстояния «1 длина Планка». Можеет оценить необходимую энергию, это Вам не «слабый» барьер в термояде.
А нам не нужно столкновение 1.22*10^9 частиц с энергией 10^10 ГэВ, достаточно столкновения одной частицы с энергией 1.22*10^19 ГэВ с любой другой. Не думаю, что это редкое событие. Видимо если что то и образуется оно неустойчиво и не опасно.
1.22*10^19 ГэВ

Эммм, это на 11 порядков больше энергии частицы "О майн готт", которая по текущим теориям не должна была иметь возможность пролететь далеко по вакууму без потерь энергии. И что-то мне сдается, что процесс, который генерирует такие о-майн-готт частицы, сложно раскочегарить ещё на 11 порядков вверх.

Тут Вы немного ошиблись. Любая статья о физике частиц (на коллайдере в смысле) оперирует понятием sqrt(s), по крайней мере — при кинетической энергии в системе ЦМ в разы больше, чем среднее геометрическое энергий покоя частиц.
Вот типичный пример применения такого обозначения:
arxiv.org/abs/1609.05122
Но, протон с кин. энергией 1 ПэВ (а ещё лучше — антипротон) — это уже интересный объект, т.к. его столкновение с любым протоном, кроме летящего «назад» породит в направлении «вперед» огромную струю частиц, в основании которой могут рождаться даже t-кварки. И т.к. импульс у частицы огромный, а в системе ЦМ ипульс нулевой, значит в этой струе суммарный «продольный» импульс частиц должен быть 1 ПэВ/c (включая всякие бозоны, родившиеся в процессе). Именно столкновение таких частиц с легкими атомами (водород, углерод, азот и кислород) в атмосфере фиксируется детекторами площадью десятки гектар.

Что в центре черной дыры? Может-ли там оказаться комфортная для жизни невесомость?

Свободно падающий наблюдатель всегда ощущает невесомость. Она комфортна если приливные силы невелики. Для невращающейся черной дыры все плохо, как уже говорилось, как ни рыпайся, все кончишь в сингулярности.

Для вращающейся есть решения комфортных орбит — но для «идеального» решения Керра. Что внутри «реалистичных» — черных дыр — никто не знает.
Где-то читал, что пересечение горизонта событий практически эквивалентно полному разрушению любого объекта, так как «двигаться можно только к сингулярности» (неточная цитата) и взаимодействие между любыми частицами этого объекта, направленное «наружу», пропадет. Т.е. например, никаких нервных импульсов.
Что с этим мифом?
Как я уже писал, свободно падающий наблюдатель может даже не заметить пересечения горизонта
No drama!
В статье же вроде есть опровержение мифа:

стало ясно то, что позже назвали «No drama» — при пересечении горизонта событий ничего особенного для свободно падающего наблюдателя не происходит — никакой тревожной музыки, пиу-пиу и зеленоватых всполохов. Провал под горизонт можно даже не заметить!


С точки зрения падающего наблюдателя горизонта событий не существует, он просто притягивается к массивному телу, локально все точки пространства до сингулярности доступны, поэтому ходить по звездолету он сможет без проблем.
Я, вероятно, совершаю ошибку, пытаясь осознать то, что нужно просто принять :)
Но неужели визуально это будет незаметно? Правильно ли представлять горзионт «снаружи» как черный непрозрачный шар? И если да — то как он будет выглядеть по мере приближения к нему (и — пересечении)?
Но неужели визуально это будет незаметно? Правильно ли представлять горзионт «снаружи» как черный непрозрачный шар? И если да — то как он будет выглядеть по мере приближения к нему (и — пересечении)?

Правильно. В теории, как я понимаю, для наблюдателя он будет падать к черному шару, при этом визуально шар будет уменьшаться (он будет видеть ближащие внутрености ЧД перед собой, так свет вполне сможет пройти это расстояние). Момент перехода горизонта событий никак не будет заметен, как мы не замечаем того что большая часть наблюдаемой Вселенной уже для нас недостижима на скорости света. Наблюдатель будет видеть то что находится внутри ЧД по мере полета к сингулятрности (например, другой звездолет безуспещно пытающийся вылететь из ЧД). То что с ним будет при достижении сингулятрности — никто не знает.

Есть теория, что вся наша наблюдаемая Вселенная это и есть внутрености ЧД (в которой пространство расширяется), в этом случае наблюдатель проживет без проблем всю свою жизнь внутри ЧД.
В принципе Вы правы в таком смысле. Скажем обычное твердое тело существует за счет электромагнитных сил. ЭМ сила вызывается виртуальными фотонами. Каким бы виртуальным не был фотон, мы не сможем посчитать пропагатор в направлении, куда вообще не могут вести геодезические (я честно не учил КТП, так что предполагаю только). По этому, условно говоря, каждое упавшее под сингулярность ядро атома перестанет притягивать окружающие электроны, оставшиеся над горизонтом. Или скажем упадут в железе таким образом только ядро и внутренние электронные оболочки (конфигурация аргона). Электроны 3d6 4s2 останутся болтаьться в электронном газе.
Но это конечно прикол, так как до падения в сингулярность столкновения в плазме должны ионизировать оставшийся атом «аргона» полностью. Весьма сильная ионизация начнется уже при kT = 68.6 эВ, электроны 1s потребуют может в 9 раз большей энергии.
Каким бы виртуальным не был фотон, мы не сможем посчитать пропагатор в направлении, куда вообще не могут вести геодезические

Они, насколько понимаю, могут туда вести, просто у ГС загнутся обратно. Ничто не мешает фотонам двигаться прочь от сингулярности, просто они далеко улететь не смогут.
Насколько я понимаю, виртуальные могут вылететь изза горизонта, так как spacelike траектории для них не запрещены
Виртуальный фотон ничем не отличается от обычного, с точки зрения возможности лететь куда угодно, по моему скромному пониманию.
Но сначала я честно нашел определение.
Виртуа́льная части́ца — объект, который характеризуется почти всеми квантовыми числами, присущими одной из реальных элементарных частиц, но для которого нарушена свойственная последней связь между энергией и импульсом частицы.

Типичное свойство виртуальны частиц в КХД — они могут иметь отрицательный квадрат массы покоя.
Вот тут гляньте пример в конце, про обычную силу ЭМ притяжения.
И другие процессы с заряженными частицами и фотонами:
Эффект Комптона
Эффект Комптона объясняется поглощением реального фотона реальным электроном с образованием виртуального электрона и последующим распадом виртуального электрона на реальные электрон и фотон, имеющие другие направления движения и энергии

И, в последних. У большинства частиц есть конечное время жизни и их энергия характеризуется шириной распада.
Объяснение Игоря Иванова о виртуальности W-бозона
Для него нет понятия «совсем реального». Номинальная масса W 80,385 ГэВ — это формальная характеристика, описывающая положение пика. В каждом конкретном событии возникает W-бозон с типичной массой 77-84 ГэВ. И это еще не виртуальный W, это самый обычный «реальный» бозон. Просто он еще реальнее быть не может, из-за ширины. О виртуальном W говорят, когда инвариантная масса опускается, скажем ниже 70 или выше 90.
>Виртуальный фотон ничем не отличается от обычного, с точки зрения возможности лететь куда угодно, по моему скромному пониманию
Я тоже думаю что с точки зрения MWI виртуальные частицы почти эквивалетны настоящим, но интересно что у большинства работающих физиков это вызывает лютый баттхерт и они буквально начинают орать «виртуальные частицы не имеют никакой реальности а лишь математический аппарат для рассчетов!»

Однако я помню что у виртуального фотона есть амплитуда вероятности лететь и быстрее и медленнее скорости света, но по мере распространения его на макроскопические растояния эти две амплитуды гасят друг друга…
ЕМНИП.
Гасятся все амплитуды, которые не соответствуют классической траектории, то есть классическая тр-я является наиболее вероятной, что не мешает ей иметь вероятность скажем «1 триллионная» из 6*1023 траекторий. Но честно, это я знаю не для фотона, а для массивной частицы и наверное даже нерелятивистской (я знаю метод вывода нер-го уравнения Гам.-Як. из ур-ия Шр-ра, но с рел. уравнением Г.Я. и Дирака или Клейна-Гордона такого перехода может не выйдет).
Про лететь быстрее или медленнее — вполне возможно, если их придется описать мнимой массой.

И раз статья о популяризации, то советую глянуть сюда:
"эта задача очень простая с точки зрения вычислений" © кандидат физико-математических наук
К вопросу о расширяемости:
Нашей вселенной это не грозит, так как она расширяется.
А как вы относитесь к этой теории habr.com/en/post/371363
Начало про уменьшение массы верное. Дальше — мне кажется малоправдоподобно, но конкретная критика пусть исходит от профессиональных ученых.
Нет ли теории, что наша вселенная это «обратная сторона» сингулярности чёрной дыры?
Ну т.е. если провалиться в сингулярность, то попадёшь в новую вселенную, для которой эта сингулярность — большой взрыв.


Увидел комментарий ТСа
да, насколько я помнб это теория Leonard Susskind
> Никто не судит Звездные войны за звук взрывов в космосе.

более того — есть вполне научное объяснение: взрывы порождают поток энергетических частиц и/или волн, которые взаимодействуют с корпусом корабля и уже это создает звуки внутри корабля.
Ионные пушки весьма рулят в реальности «Зв. войн», насколько я помню.
Даже в этом случае должна быть задержка между вспышкой и звуком
Есть ещё проще объяснение:
Все звуки в космосе генерирует навигационная система корабля, чтобы облегчить ориентирование пилоту.
Возможно уже был такой комментарий. Сорри если повторюсь.
Главный герой не пересекал горизонт событий. Он попал в эргосферу. Это керровская чёрная дыра.
Источник — девушка, которая писала формулы к фильму.
НЛО прилетело и опубликовало эту надпись здесь
полуправды и неверных выводов. Начинается все с правильного утверждения: при приближении к горизонту событий для наблюдателей, подвешенных над черной дырой, замедление времени стремится к бесконечности. Это верно.

с правильного утверждения:

Вы серьёзно? Обсуждая вообще гипотетические штуки вроде черных дыр (неизвестно, существуют ли они, по крайней мере, с теми свойствами, какие к ним приписываются), называть что-то верным или неверным. Ну, даёте народ…
НЛО прилетело и опубликовало эту надпись здесь
НЛО прилетело и опубликовало эту надпись здесь

"Ничего не понимаю, значит это неверно".
Фанатам Эйнштейна и всевдонаучным троллям обычно не нравятся воинствующие неучи.

НЛО прилетело и опубликовало эту надпись здесь
Ну, да, Вы не одиноки.
не понравилось моё мнение
Не путайте мнение и невежество. Если я скажу, что Луна и Солнце — это огромные прожекторы, а звезды — это пробитые в покрывале дырочки, то это будет не мнение, а невежество.
НЛО прилетело и опубликовало эту надпись здесь
Это вы как назовёте?
Я назову это «высокомерием». Сравниваете себя с Эйнштейном? Серьёзно?
Я скажу, что вы не умеете читать вообще, и биографию Эйнштейна в частности. Кем-кем, а заурядным он в школе не был, и ВСЕГДА был одним из выдающихся учеников.

А то, что некоторые двоечники не удосужились прочитать про то, что в определенный период обучения в учебном заведении была «обратная» шкала, где единица считалась высшим балом, то что Эйнштейн ненавидел зубрежку из-за чего у него были конфликты с учителями — это проблема тех двоечников, которые не умеют читать.
НЛО прилетело и опубликовало эту надпись здесь
Эйнштейн родился не в средние века.

Его реальные аттестаты вполне доступны для общества, его биография подтверждена и проверена многочисленными журналистами и историками.

А вот вы почему так самоуверенно считаете, что он был двоечником?

Какая разница, кто автор теории, двоечник или отличник, святой или негодяй. Это никак не влияет на верность или ошибочность его научных результатов. Так давайте обсуждать науку отдельно, а личности (если это интересно) — отдельно.

Так этот тред и начался с обсуждения личности, мы ее и обсуждаем отдельно.
А если я скажу что Эйшнтейн в школе был заурядным троечником, потом работал клерком патентного бюро, был мужем талантливой сербки Милевы Марич, 20 раз безуспешно выдвигался на получение Нобелевской премии и получил её с 21-й попытки

Вы так самоуверенно это утверждаете ©, словно учились вместе с ним в одном классе, работали в одном патентном бюро, вместе были мужем Милевы Марич, вместе выдвигались на Нобеля и вместе её получали. Вам не кажется что такая самоуверенность это признак некого душевного дисбаланса?
НЛО прилетело и опубликовало эту надпись здесь
Вы действительно не находите в Ваших комментариях ничего ироничного?
Для искушения человека сначала Бог создал Дьявола, потом Коперника, а ещё позже — Королева с Гагариным. И ещё фон Брауна:
Эффективность боевого применения «Фау-2» была крайне невысокой: ракеты имели малую точность попадания (в круг диаметром 10 км попадало только 50 % запущенных ракет) и низкую надёжность (из 4300 запущенных ракет более 2000 взорвались на земле или в воздухе при запуске, либо вышли из строя в полёте[источник не указан]). По различным источникам, пуск 2000 ракет, направленных за семь месяцев для разрушения Лондона, привел к гибели свыше 2700 человек (от каждой ракеты погибал один или два человека).


Так что в сравнении с изобретателем газовой камеры фон Браун почти святой.
Здесь обсуждается не Эйнштейн, а ОТО. Личность Эйнштейна с истинностью ОТО вообще никак не связанна.
Достаточно добавить «в рамках данной теории». Есть математически проработанная теория, хорошо проверенная там, до куда мы смогли дотянуться, и есть вполне конкретные ее предсказания. Вот о них и речь.
Очень сильно верю, что описанные в третьей книги «Сказаний о прошлом Земли» методы ведения войны путём уменьшения скорости света и понижения размерности солнечной системы случатся не на нашем веку.
Хотя экранизацию гляну :)
Весна. У программистов баз данных началось обострение. Оказывается, в свободное время, программисты баз данных, ну чисто как хобби, изучают ОТО и физику элементарных частиц. А меня всё мучал вопрос, кто же даёт инвайты на Хабр разным фрикам?
> кто же даёт инвайты на Хабр разным фрикам

В споре рождается истина. (с)

А так в начале 20 века тоже считали что в Физике всё открыто, и если бы не фрики(как вы их называете) придумывали различные теории,(от простых до абсурдных) то долго бы топтались на месте.
Как я понимаю, Tzimie весьма глубоко изучил ОТО и физику элементарных частиц задолго до баз данных. SQL — всего лишь ремесло, дающее хлеб насущный. Квалификация автора в физике видна невооружённым глазом.
Я конечно ОТО глубоко не изучал, вывод выражения для действия грав. поля не проведу сам.
Физику эл. частиц изучали чуть лучше, но все равно не особо разбираюсь пока. Скажем в нейтроне действительно есть W-бозон массой в разницу масс нетйрона и протона.
Все верно. Я такой. И интересы даже совпали. Не ругайте нас. Нам очень нравится ОТО и физика элементарных частиц.
если Вселенная бесконечна, то она бесконечна с самого начала. Бесконечна и почти однородна.
Эти два постулата являются предположениями и потому не могут опровергнуть что бы то ни было. Тем более, что в конце статьи пришли к «невозможны статические решения Вселенной».
Приливные силы разрывают всякого еще на подлете к черной дыре… есть сверхмассивные черные дыры, массой в миллиарды солнечных масс, а радиусом больше орбиты Урана.
Это не опровержение мифа, а уточнение — есть предположения о возможности существования некоторых объектов, при падении в которые, теоретически, некоторые тела не будут разорваны.

IMHO, главный миф физики скрывается за словом «доказано». Если ознакомиться со статьями Элиезера Юдковского и других рационалистов, если освежить знания о научном методе, то реже возникает желание махать шашкой с постулированием невозможности чего бы то ни было. Любой известный закон является лишь ступенькой на пути к пониманию реальности — мы придерживаемся тех теорий, которые помогают продвинуться дальше. Любые наши знания являются временными.

А в данной статье даже не теории, а гипотезы выдаются за часть реальности, чтобы опровергнуть… сценарий кинофильма?

Я решил сформировать список наиболее часто встречающихся неправд и полуправд. Итак, самое частое это…
Самое частое в какой из выборок?
>миф физики скрывается за словом «доказано»
В физике ничего никогда не доказано, так как любой эксперимент может обрушить любую теорию, однако любое число подтверждающих экспериментов не делают теорию «доказанной». Однако, когда теория достаточно устоялась то начинает действовать правило «extraordinary claim requires extraordinary evidence».

Буквальное отношение к слову «доказано» — еще одна проблема папулярной науки

>Самое частое в какой из выборок?
Больше всего по заблуждениям новичков приходящих на physicsforums.
Буквальное отношение к слову «доказано» — еще одна проблема
Если вы понимаете проблему, то почему в вашей статье встречаются безапелляционные заявления, в которых одно из предположений выдается за часть реальности, позволяя откинуть другие предположения в список мифов? Например:
Нет никакой точки, где был взрыв, который распространялся в окружающую пустоту. Повторюсь, вселенная была почти однородной
Если ТБВ говорит лишь о том, что было после БВ, то почему отсутствие единой точки или точек до БВ, а также предположения о состоянии вселенной до БВ превратились в доказанный факт?

Замечу, что я не придерживаюсь обратной точки зрения, не пытаюсь доказать преимущество предположения «была одна точка» — оно не моё, говорю не о нём. Но для отнесения чего-либо к мифам нужно нечто большее, чем гипотеза «возможно было иначе». Для развенчания «наиболее часто встречающегося заблуждения» хотелось бы увидеть более основательное раскрытие темы.
почему в вашей статье встречаются безапелляционные заявления, в которых одно из предположений выдается за часть реальности

Чего люди часто не понимают, это то, что физика не "выдаёт нечто за реальность", а "объясняет реальность через модели". Если нечто "безапеляционно утверждается", то это всегда утверждается в рамках некоторой модели.


почему отсутствие единой точки или точек до БВ, а также предположения о состоянии вселенной до БВ превратились в доказанный факт?

ТБВ — это физическая модель (одна из многих), и конкретно в этой модели дела обстоят именно так, что позволяет модели объяснить определённые наблюдаемые факты. Вы можете предложить другую модель, где будет единая точка начала или несколько точек, и будут другие предположения. Если это будет объяснять наблюдения — это будет прекрасно. Если это будет объяснять больше наблюдений, чем все остальные модели — просто расчудесно.

Мне срочно нужно дополнить модель космологии уравнением состояния для ТМ (вместо банального P (ro) = 0) и проверить её на масштабе от конца инфляции до эпохи рекомбинации.
физика не «выдаёт нечто за реальность», а «объясняет реальность через модели»
Не спорю и вполне поддерживаю, но не сильно ли мы отклонились от обсуждаемой статьи? Я про статью говорил, в ней есть что-то вроде:
Наиболее часто встречаемая неправда или полуправда — «когда Вселенная была не больше размера атома». Правда — нет никакой точки, где был взрыв, который распространялся в окружающую пустоту.
Как это соотносится к «объясняет реальность через модели»? Я вижу утверждение, а не объяснение.

ТБВ — это физическая модель (одна из многих), и конкретно в этой модели дела обстоят именно так, что позволяет модели объяснить определённые наблюдаемые факты.
Я не спорю и вполне поддерживаю. Но в обсуждаемой статье сказано примерно так:
ТБВ это НЕ теория о большом взрыве, а о том, что случилось после большого взрыва. До БВ не было никакой точки, где был взрыв, и вселенная была бесконечной и однородной.
И меня смущает то, что для опровержения мифа упоминается только теория, которая, судя по упоминанию же, для этого не подходит. В статье не сказано ничего о том, на чем основаны постулаты.

Вы можете предложить другую модель
Нет, конечно, я лишь комментировал конкретную статью. Именно в статье есть то, что выбивается из взвешенного подхода, который озвучен в комментарии ниже:
Физики как раз довольно скромны и понимают, насколько они далеки от познания мира.
Ну, возможно, утверждения следует чаще сопровождать пометкой «в рамках данной модели», чтобы у людей не было впечатления о том, что «физики вещают истины», а не обсуждают модели. Однако как часто это делать, чтобы никто 100% не возмутился?
в рамках данной модели
Еще раз предлагаю в обсуждении вернуться к обсуждаемой статье.
О рамках какой модели идет речь в тексте после первого же подзаголовка «Большой взрыв»?
Упомянута только ТБВ, но она не подходит, поскольку говорит о времени послевзрывия, о чем сказано в статье же.

Данная статья про мифы. Самым частым назван миф о большом взрыве из одной точки.
Для развенчания мифа выдвинуты постулаты — до БВ Вселенная бесконечна и однородна, не было точки, из которой взрыв распространялся в пустоту.
Вопрос — о какой модели идет речь?
Ответ — о какой-то неназванной. Что-то неназванное опровергает «взрыв из одной точки», а также постулирует «до взрыва бесконечна и однородна».
Мне кажется, что какой бы то ни было миф нельзя опровергнуть постулатами, основанными на чем-то неназванном.
Я вот скажем скажу, что после инфляции радиус Вселенной стал 10^5584000 метров. Действительно ли меня в такой теории должен интересовать размер Вселенной до инфляции? Или мне придется назвать такую Вселенную Мультивселенной (либо наоборот — назвать «нашу Вселенную» Метагалактикой)?
Извините, но я не вижу связи вашего комментария с обсуждаемой статьей или с моим комментарием, который относился к конкретной статье.

Если я скажу, что после инфляции радиус Вселенной стал 10^5584005 метров, то это будет не теория, а высказывание. Высказывание может породить интерес к размеру до инфляции, может не породить. В любом случае, моё или ваше высказывание не позволяет назвать мифом чье-то (не моё) высказывание «Вселенная взорвалась из одной точки ничтожно малого радиуса».
Да, моя гипотеза только говорит, что Вселенная за короткий период времени (брал 10-36 секунд) стала в невероятное число раз больше, указанная величина больше подходит для гипотезы огромной мультивселенной. Может Вселенная не столь огромна, тогда она была изначально в 10^5583900 раз меньше радиуса протона.
Но это все гипотезы, кажется надежное доказательство инфляции ещё не проходит проверку с учетом всех возможных погрешностей наблюдаемых данных.
Следующий этап идеи о Мультивселенной, это сказать, что на расстоянии 10^5583995 км от наблюдаемой части вселенной постоянная тонкой структуры больше на 4% и там немного не так пошла эволюция звезд и газа в галактиках. Но проверить такую гипотезу мы не можем даже в самых «научно»-фантастических допущениях.
Упомянута только ТБВ, но она не подходит, поскольку говорит о времени послевзрывия, о чем сказано в статье же.

ТБВ принесла (вольно или невольно) сам термин "Большой Взрыв", поэтому если говорим о "Большом Взрыве" — значит говорим о ТБВ, очевидно. Да, ТБВ не объясняет, почему этот "взрыв" произошёл и объясняет лишь то, что происходило при t > 0. Но если устремить время к 0, то никакого "центра взрыва" в этой модели так и не появляется, поэтому в рамках данной модели "взрыв из точки" — это миф, порождённый неудачным термином.

никакого «центра взрыва» в этой модели так и не появляется
Hawking S. W., The occurrence of singularities in cosmology, III. Causality and singularities:
Результаты наших наблюдений подтверждают предположение о том, что Вселенная возникла в определённый момент времени. Однако сам момент начала творения, сингулярность, не подчиняется ни одному из известных законов физики.
Иначе говоря, несколько преждевременно называть любое утверждение мифом, оставаясь в рамках ТБВ и опираясь на известные законы, поскольку мы все еще не знаем, был ли взрыв из точки или из обширной области пространства, или что там в сингулярности существовало, или же всё пространство расширяется вместе с материей в чем-то, что тоже может иметь точки и области.
Можно отсеять «слово Божье», «лабиринт Дворкина» и прочие утверждения из художественной литературы ввиду их бесполезности. И сосредоточиться на формулах и числах инфляционной модели, чтобы объяснить переход от сингулярности с наблюдаемой картине. Но я бы все же смягчил формулировку от «миф» до «недоказуемое утверждение». Утверждения «центр был» и «центра не было» равны по весу — мы не наблюдаем ни подтверждения, ни опровержения каждому из них. Если устремить время к нулю в рамках ТБВ, то мы попадаем в область неизвестного.
Тут следует заметить такое свойство сингулярности. Современная Вселенная описывается определенным количеством материи (основную массу приписывают на ТМ и межгалактический газ, ЕМНИП).
На момент начала инфляции радиус Вселенной гарантированно был меньше её гравитационного радиуса, то есть она должна была быть сингулярностью. Тут может быть вопрос с тем, можно ли описать модель «горячей ЧД», в которой достаточная для образования сингулярности масса находится в состоянии движения в направлении «строго от центра масс» с релятивистской скоростью. Станет ли такой объект ЧД или нет?
Так вот, одним из решений проблемы «Вселенная была сингулярностью» является предположение о том, что все эти кварки, лептоны и бозоны (частицы ТМ ещё) возникли уже после того, как вся огромная масса Вселенной расширилось инфляцией до размера выше «гравитационного радиуса материи».
Я бы сказал так: в модели ТБВ нет пространственного центра взрыва, т.е. какой-то выделенной области с пространственными координатами (x,y,z), от которой всё разбегается. Точно как же как на поверхности надуваемого шарика (модель замкнутой Вселенной) нет никакого «центра раздувания поверхности» — все точки на поверхности равноправны, даже несмотря на конечность размеров этой поверхности. Даже устремив размер поверхности к 0, мы никак не получим на ней центра, просто математически. Покуда есть поверхность — центра на ней нет. А когда поверхность схлопнется до нуля, то говорить о пространственном центре становится просто бессмысленно.

Если поверхность бесконечна (модель открытой Вселенной), то она бесконечна даже при t=0, и центра разбегания там точно так же нет, чисто математически.

В тоже время в модели замкнутой Вселенной можно сказать, что центр БВ — это точка с временной координатой t=0 и неопределёнными пространственными координатами (в силу отсутствия пространства). По сути, Хокинг в вашей цитате пишет о ней. Но это уже как бы совсем не тот «центр», про который люди наивно думают, впервые слыша про «взрыв вселенной».
как бы совсем не тот «центр», про который люди наивно думают, впервые слыша про «взрыв вселенной»
Ну да, и Большое замерзание не сопровождается заиндевелыми сосульками, и Большой разрыв не сопровождается черными трещинами поперек галактик, и Большой хлопок не сопровождается громким звуком, как могли бы подумать люди, обладающие образным мышлением.
И лично мне трудно представить «пузырь» пространства, раздувающийся из точки в непространстве, надпространстве, четырехмерном пространстве, в абстракции небытия, или что там снаружи.

Главная мысль, с которой я начал данный тред — какое бы то ни было предположение не может являться основанием для отвержения другого предположения. Предположение должно оцениваться его полезностью, а не наличием других предположений.

Сейчас для меня диалог выглядит так:

Статья: вселенная была однородной, взрыв был не из одной точки.
Я: почему не из одной?
Вы: в нашем пространстве такой точки нет.
Я: точка могла быть, но относится не к наблюдаемому нами пространству.
Кто-то еще: и в чем польза такого предположения?
Я: в наличии простора для последующих (не моих) идей.

Предположение же «точки не было» лишено какой бы то ни было пользы, на мой взгляд. И остается лишь предположением, поэтому не позволяет отнести что бы то ни было в разряд мифов.
Раз упомянули «Большой хлопок», то скажу, как я его понимаю.
Предположим, что плотность темной энергии 5 миллиардов лет назад была обной, а через 13 миллиардов лет станет больше. Тут мы начинаем предполагать — «Насколько большой она станет?».
Локальный уровень №1 — плотность ТЭ немного вырастет, но не сможет преодолеть силу гравитации (в основном представленную ТМ) на масштабах сверхгалактик (ЕМНИП, центр нашей с.г. — это тут).
Для понимания масштабов
Скопле́ние Воло́с Верони́ки (другие названия: Скопление Кома, Abell 1656) — крупное скопление галактик, расположенное приблизительно на расстоянии 99 мегапарсек.
Объём скопления галактик в Волосах Вероники составляет приблизительно 3000 кубических мегапарсек. Состав галактик обычен для плотных сферических скоплений. Здесь нет галактик с ярко выраженной спиральной структурой.


А максимум фантазии авторов такой идеи доходит до того, что давление ТЭ растет до таких величин (гиперболически, относительно «времени разрыва»), что разрушает атомы и даже ядра.
Bronx
в модели ТБВ нет пространственного центра взрыва, т.е. какой-то выделенной области с пространственными координатами (x,y,z), от которой всё разбегается. Точно как же как на поверхности надуваемого шарика (модель замкнутой Вселенной) нет никакого «центра раздувания поверхности» — все точки на поверхности равноправны, даже несмотря на конечность размеров этой поверхности. Даже устремив размер поверхности к 0, мы никак не получим на ней центра, просто математически. Покуда есть поверхность — центра на ней нет. А когда поверхность схлопнется до нуля, то говорить о пространственном центре становится просто бессмысленно.


Давайте не математическими безразмерными точками на поверхности рассуждать, а возьмём небольшой «треугольник» (или шестиугольник, это не так важно), к примеру.
И покроем этими треугольниками поверхность шара.

image

А теперь начнём сдувать шар, выкидывая лишние треугольники, по мере сдутия шара.

Треугольников будет становится всё меньше и меньше, — на поверхности шара — пока не останется один треугольник — вот он и будет нашим центром Вселенной, его первой праматерь-поверхностью.

Это двумерная аналогия установления центра поверхности на шаре.

Но вам нужна точка? — Хорошо, точка пересечения медиан этого закрашенного треугольника и будет искомым вами центром Вселенной на плоскости.

Почему это будет центр? — Потому что с него всё началось. Этот треугольник можно закрасить и начать надувать шар по новой, добавляя уже треугольники по мере надувания, когда шар будет полностью надут, то этот закрашенный треугольник будет явно выделяться среди остальных треугольников на поверхности. — Так почему ему не быть центром поверхности? От него можно вести отсчёт расстояния до других треугольников (до их центров треугольников — точки пересечение медиан, к примеру).

Суть моего предложения — заменить точку на площадь ( а уж саму точку определить, при желании, только и только на этой, неизменной(!) площади).

А потом назвать это 3-мерной сферой — 3-мерной поверхностью, ограничивающей 4-мерный объем.
А теперь начнём сдувать шар, выкидывая лишние треугольники, по мере сдутия шара.

Какие именно треугольники вы собрались выкидывать? Кто решает, какие выкинуть, а какие оставить, по какому правилу (и по какому праву)? Как в реальности выглядит "выкидывание"? Их кто-то ножницами вырезает, а оставшееся пространство сшивает? Что происходит между вырезанием и сшиванием? Что происходит с той материей, которая была на выкинутом треугольнике?


Ничего никуда выкидывать не нужно — треугольники просто сжимаются вместе с шаром, и тогда отпадают куча проблем с нарушением целостности, непрерывности, симметрии и законов сохранения.

Bronx
Какие именно треугольники вы собрались выкидывать? Кто решает, какие выкинуть, а какие оставить, по какому правилу (и по какому праву)? Как в реальности выглядит «выкидывание»? Их кто-то ножницами вырезает, а оставшееся пространство сшивает?
Это нас не интересует, нас интересует расширение, а сдутие — это воображаемое действие.

Bronx
треугольники просто сжимаются вместе с шаром, и тогда отпадают куча проблем с нарушением целостности, непрерывности, симметрии и законов сохранения.
Нет. Это модель не та, что нам нужна и она не отображает действительность. А Законы Сохранения в ОТО не работают вовсе.

Треугольники (или иные «островки» материи — «Большие скопления галактик» — они НЕ меняются при расширении никак).

1) Вначале наш шарик — есть сдутый
футбольным мяч.
image


2) Началась стадия инфляции, — Бог начал вдувать в наш шарик «темную энергию» — наш футбольный мяч
начал распрямляться.
image

К концу инфляции наш мяч приобрёл идеально плоскую (без морщин и смятий) поверхность.
«Будем считать, что радиус Вселенной перед выходом на экспоненту всего на несколько порядков превышал планковскую длину, 10-35 м. Если в экспоненциальной фазе он вырастет, скажем, в 1050 раз, то к ее концу достигнет тысяч световых лет. Каким бы ни было отличие параметра кривизны пространства от единицы до начала расширения, к его концу оно уменьшится в 10-100 раз, то есть пространство станет»идеально плоским!


3) Когда все многоугольники («Большие скопления галактик») распрямились —
начали трещать швы между ними.
image


Тут мы добавим на «камеру» нашего футбольного мяча немного воды (или геля) — такой вязкости, чтобы оторванные куски мяча («Большие скопления галактик»)
начали скользить по нему
image


4) Произошёл Большой Взрыв
швы треснули
image
и куски мяча стали скользить по поверхности камеры отдаляясь друг от друга, приобретя ускоренное движение со временем. (Это явление реально наблюдается — расширение Вселенной не оказывает никакого действие ни на вашу комнату, ни на Солнечную систему, ни на нашу Галактику, ни на нашу местную группу галактик — «тёмная энергия» действует только и только на ускоренное (на сегодня) удаление «больших групп галактик»
друг от друга).
image

Где же мы находимся? — Наш кусок мяча («местная группа галактик») находится ровно на противоположной стороне от "пупа Вселенной" (место куда Бог вдувает «тёмную энергию»).

Почему так? — а на мячике только в этом месте (точке), при телескопировании по всем сторонам мы видим
пуп Вселенной
image
— в виде самых молодых галактик. Почему молодых, — потому что мы расширялись по крайней мере со скоростью близкой к скорости света и поэтому мы можем видеть сейчас чем «дальше» тем более молодую Вселенную.

Находясь на противоположной стороне от «пупа Вселенной» мы вполне имеем наблюдаемую нами в реальности
анизотропию Вселенной.
image


Но что делать с утверждением — что все точки во Вселенной равноправны? — Так как модель мячика — это модель плоской двумерной Вселенной — то для того, чтобы все точки в реальной Вселенной были равноправны мы можем предположить, что реальная (не плоская) Вселенная —
есть совокупность (для каждой точки) футбольных мячей,
image
где каждая точка реальной Вселенной находится (при её моделировании футбольным мячом) на противоположной стороне «пупа Вселенной»).

P.S.
Надо отметить, что это всего лишь двумерная модель расширения Вселенной, придуманная кем-то (и постоянно упоминаемая) для того, чтобы пояснить невозможность определить центр Вселенной, но модифицированная мною, с целью показать что и на данной простой модели вполне можно определить центр (от же «пуп»)
Вселенной.
image


А Законы Сохранения в ОТО не работают вовсе.
Это не так. Там плохо определена энергия-импульс гравитации, но все остальное сохраняется. Кроме того, энергию-импульс гравитации можно определить так, что-бы все сохранялось. У вас проблемы с законами сохранения серьезнее чем в ОТО.
Это явление реально наблюдается — расширение Вселенной не оказывает никакого действие ни на вашу комнату, ни на Солнечную систему, ни на нашу Галактику, ни на нашу местную группу галактик — «тёмная энергия» действует только и только на ускоренное (на сегодня) удаление «больших групп галактик»

Это совсем не верно, просто сила давления ТЭ меньше силы грав. притяжения.
2) Началась стадия инфляции
3) «Большие скопления галактик» распрямились
4) Произошёл Большой Взрыв

Т.е. у вас БВ происходит после стадии инфляции и даже после образования галактик и скоплений?

А вот у меня такой порядок (да, это все гипотезы, не имеющие достаточного основания):
1. Инфляция. Характеризуется температурой. Вроде как заканчивается на температуре kT = EPlanck.
2. Инфляция порождает все привычные частицы вещества (элементарные), включая ТМ.
2.5 Температура «плазмы» из кварков и лептонов kT >> mW, идут активно все те процессы, которые происходили в такой плазме на коллайдерах Tevatron и LHC.
3. Температура кварк-глюонной плазмы падает в несколько раз ниже величины kT = 211 МэВ и из лептонов только иногда рождаются одиночные мюоны.
Bronx
Т.е. у вас БВ происходит после стадии инфляции
Да, конечно.

Bronx
и даже после образования галактик и скоплений?
Нет, тут мой ляп. Я забыл про… тёмную материю. Но я исправлюсь.

1) Вначале наш шарик — есть сдутый футбольным мяч.

2) Началась стадия инфляции, — Бог начал вдувать в наш шарик «темную энергию» — наш футбольный мяч начал распрямляться.

К концу инфляции (очень очень быстро — в период времени с 10 в степени -42 сек до 10 в степени -36 сек от начала) наш мяч приобрёл идеально плоскую (без морщин и смятий) поверхность состоящую из… темной материи.

То есть наш футбольный мяч обшит НЕ кожей, а
ТЕМНОЙ МАТЕРИЕЙ.
image


3) Когда все области(из тёмной материи)
распрямились
«Во время инфляции любые предшествующие неоднородности скалярного поля растягиваются настолько, что практически исчезают. На завершающем этапе инфляции это поле начинает быстро осциллировать вблизи минимума своей потенциальной энергии. При этом в изобилии рождаются частицы и фотоны, которые интенсивно взаимодействуют друг с другом и достигают равновесной температуры. Так что по окончании инфляции мы имеем плоскую горячую Вселенную, которая затем расширяется уже по сценарию Большого взрыва. »


— начали трещать швы между ними. Но перед этим прошла гравитационная волна. Почему? — Потому что поверхность мяча натянулась идеально и могла позволить передавать (генерировать) гравитационный волны, как натянутая мембрана. (эти гравитационный волны от инфляции так и не были обнаружены).

И эта натянутая мембрана из темной материи резко остановила инфляционное расширение Вселенной — что было довольно неожиданно, и как всякое резкое изменение породило фазовый переход — поверхность тёмной материи "вскипела" — на ней "мелким бисером" выступили "капли барионов" —
будущае основа звёзд, галактик и нас.
image


Тут мы добавим на «камеру» нашего футбольного мяча немного воды (или геля) — такой вязкости, чтобы оторванные куски мяча (области темной материи) начали скользить по нему.

4) Произошёл Большой Взрыв — швы треснули
и куски мяча (области тёмной материи) начали отрываться друг от друга, удерживаясь,
с помощью нитей


состоящих из рвущейся же тёмной материи. Отрываться и скользить по поверхности камеры отдаляясь друг от друга.

5) Примерно 5 миллиардов лет назад эти нити, связывающие области темной материи, стали рваться и куски темной материи заскользили по мячику с ускорением.

6) Со временем гель на поверхности мяча станет… высыхать и силы трения начнут рвать уже куски тёмной материи, разлетающиеся с ускорением. Потом станут рвать галактики, звёзды, атомы, ядра атомов и… кварки. Но это будет ещё не скоро.

А где же в нашей модели звёзды и галактики? — Их очень очень мало — 5% всего от общего количества массы Вселенной. Основная масса — это темная материя и тёмная энергия.

Галактики — они как «грязь», «пыль», «прах» на поверхности нашего мяча, на поверхности кусков тёмной материи, которые и правят бал в нашей модели. — Да, они встречаются и среди кусков темной материи и среди нитей темной материи, но встречаются как незначительные вкрапления.

Неплохая модель получилась. Поди.
То есть Вы пишете «5 млрд. лет назад что-то порвалось», а не «Вот просто уравнение ОТО, уравнение состояния всех типов материи, постоянная плотность ТЭ — результаты моделирования говорят, что 5 млрд. лет назад сила давления превысила силу гравитации ТМ»?
И эта натянутая мембрана из темной материи резко остановила инфляционное расширение Вселенной

Конечно физика пока не знает, что такое ТМ. Но, ИМХО, ТМ и все прочие фермионы с бозонами были рождены в процессе инфляции. Что Вы назовете БВ не знаю, но смысл в том, что «перед ним» не было ничего, даже тех «30% ТМ» (конечно 5 млрд. лет назад доля ТМ была больше, чем сейчас).
Victor_koly, Bronx, тут такое дело… 8:15. Тот бред про футбольный мяч и гель я конечно не читал, однако вчера вот для себя открыл что БВ сегодня называют конец эпохи рекомбинации, а не t0 + th. Внезапно.
Конец эпохи рекомбинации — это реликтовое излучение. До этого мы можем «видеть» хорошо разве что грав. волны. Конкретно — рекомбинации водорода, т.к. фотон, которого хватает для ионизации атомарного водорода, не сможет ионизировать гелий.
Что понял из картинки в 8:12. Плотность бар. материи (от общей) — 0.02, плотность ТЭ — менее 68.5-69%, остальное — плотность любой материи. Возраст Вселенной — менее 13.8 млрд. лет назад от эпохи рекомбинации.
Да емое…
Суть в том что раньше была хронология БВ -> инфляция, а сейчас время инфляции входит в БВ. Видимо потому что момент БВ происходит за планковское время и нет смысла рассматривать его отдельно.

Про то что БВ считается до конца рекомбинации я затупил, конечно.
Никогда так не было, сначала инфляция была частью БВ, следуя за планковской эпохой а как они теперь планковскую эпоху и инфляцию теперь видим не считают частью БВ, а предшествующими БВ.
А вообще так называемый термин «наблюдаемая вселенная» полный бред, так как ограничение скорости распространения фотона в ничем не ограниченном пространстве это «современнонаучный» абсурд, так как пространство, которое осталось позади летящего фотона и между его источником, оно представьте себе тоже расширяется и чем дольше фотон пролетит времени тем больше будет скорость расширения этого отрезка пути, которую нужно добавить к собственной скорости фотона — скорости света, по этому в расширяющейся вселенной свет путешествует гораздо быстрее скорости света и никаких горизонтов событи нет и не может быть.
Это не верно для ускоренно расширяющейся вселенной. Действительно, скорость удаленных объектов относительно нас может превышать скорость света. Поэтому послав фотон от нас сейчас, этот фотон будет удаляться от нас все быстрее и быстрее благодаря расширению вселенной. Тем не менее, далекие объекты удаляются от нас ЕЩЕ быстрее, поэтому фотон не достигнет их НИКОГДА (обратное тоже верно)
а сейчас время инфляции входит в БВ

Для меня инфляция длиной 10-36 секунд является достаточно непознаваемой штукой, что её можно включать в БВ. Как я писал Выше, в процессе инфляции была рождена вся материя.
Хотя мои теории могут не совпадать с действительностью. Я могу взять экзотическую оценку, согласно которой инфляция длилась до конца эпохи Великого объединения (беру оценку 1015 ГэВ, может там энергия на порядок меньше).
Если кто-то хочет опровергнуть мою «теорию», то уточню. Под энергией имею в виду среднюю кинетическую энергию относительно местного центра масс, то есть эта величина не характеризует скорость «убегания частиц от центра Вселенной».
открыл что БВ сегодня называют конец эпохи рекомбинации

Это весьма странное заявление, потому что хронология Вселенной повсеместно ведётся от t=0, и определённые моменты времени существования Вселенной называют "time t since Big Bang". Никогда не встречал иного.

Victor_koly
То есть Вы пишете «5 млрд. лет назад что-то порвалось», а не «Вот просто уравнение ОТО, уравнение состояния всех типов материи, постоянная плотность ТЭ — результаты моделирования говорят, что 5 млрд. лет назад сила давления превысила силу гравитации ТМ»?
Понимаете, выражения «сила давления превысила силу гравитации» довольно странное, так как в ОТО нет сил гравитации вообще.

Понимаете, кроме того, выражения типа «что-то превысило», «нет сходимости», «так как равно нулю» и прочие — они берутся из мира математических моделей, а тут ведь реальные нити, которые реально обнаружены и которые реально рвутся.
image

Victor_koly
Но, ИМХО, ТМ и все прочие фермионы с бозонами были рождены в процессе инфляции.
Это спорно. По окончании инфляции были рождены барионы. Но про темную материю ничего не известно. В модели футбольного мяча она уже была перед инфляцией и именно она остановила инфляцию.

Кроме того, я забыл про… античастицы. Куда они делись? — В модели футбольного мяча, они возникли в момент окончания инфляции — «что было довольно неожиданно, и как всякое резкое изменение породило фазовый переход — поверхность тёмной материи „вскипела“ — на ней „мелким бисером“ выступили „капли барионов“… и антибарионов.

А дальше? — в нашем „плоском мире модели футбольного мяча“ античастицы были порождены хотя и одновременно с частицами, но на „другой стороне тёмной материи, “обтягивающей» наш футбольный мяч.

Фактически — тёмная материя стала разделителем частиц и античастиц — как обкладка конденсатора разделяет «плюс» и «минус».

А так как мы реально воспринимаем только и только плоский мир поверхности футбольного мяча (в нашей модели) — то для нас, античастицы, находящиеся на другой (внутренней) поверхности тёмной материи фактически… исчезли из нашего плоского мира вообще.

Victor_koly
Что Вы назовете БВ не знаю, но смысл в том, что «перед ним» не было ничего, даже тех «30% ТМ» (конечно 5 млрд. лет назад доля ТМ была больше, чем сейчас).
Большой Взрыв — это начало обычного, не инфляционного расширения Вселенной, которое произошло в момент окончания инфляции, которая была остановлена «натяжением» («распрямлением») тёмной материи в нашей модели футбольного мяча. Дальше мяч начал трещать.

Ни тёмная материя ни обычная материя НЕ рождались больше.
Темная материя уже была в момент начала инфляции и именно она остановила инфляцию.
Обычная (барионная) материя возникла в момент окончания инфляции из-за резкой остановки инфляции.
Всё что было потом — то это только и только за счёт постоянное пополнение Вселенной тёмной энергией — которая не устаёт «накачиваться» в наш футбольный мяч.

Модель футбольного мяча, будь она осознана лет 100 назад, описывает все (известные! на сегодня) фазы развития нашей Вселенной:
— от
компактного состояния в момент начала инфляции,
image
— стадию инфляции (быстрой, очень быстрой, — ибо ничего не мешало этому процессу) «накачки» мяча до момента распрямления поверхности мяча),
— стадию резкой остановки в момент полного распрямления поверхности мяча, остановки, вызванной «распрямившейся» темной материей, — Остановки, повлёкшую образование частиц и античастиц барионной материи, античастицы ушли под поверхность тёмной материю нашего мяча и никак не влияют на наш плоский мир барионных частиц),
— стадию начала Большого Взрыва (дальнейшего накачивания мяча и «раздирание» тёмной материи по кускам),
— стадию начала ускорения (момент когда начали рваться нити тёмной материи связывающие куски тёмной материи),
— будущую стадию "высыхания геля" на поверхности «камеры» (то есть самого пространства-времени), что повлечёт разрывание кусков тёмной материи (а также и самой барионной материи вплоть до кварков),
— стадию когда начнёт (в далёком далёком будущем) рваться уже сама «камера» нашего футбольного мяча — стадия Большого Разрыва (по научному).

image

Будь она осознана лет 100 назад, эта модель получила бы уже кучу Нобелевских, но… постфактум НЕ считается. Эх.

а тут ведь реальные нити, которые реально обнаружены и которые реально рвутся

Я не силен в моделировании космологии, но возникновение «стен» и «нитей» галактик явно следует из этих моделей.
Если что, отдельные барионы ну никак нельзя рассматривать при энергиях выше энергии великого объединения.
Если вдруг Вы не в курсе
Барионы — это частицы, состоящие из кварков (вида фермионов), а ещё в них есть глюоны (вид бозонов)

Барионы — это единственный известный вид стабильных частиц (кроме u- и d-кварков), в стабильном виде представлен протонами и нейтронами (есть нейтроны в ядрах 80 стабильных хим. элементов). Облако электронов не может быть ТМ, так что выходит, что не менее 99.945% массы обычной материи — это барионы.
IMHO, главный миф физики скрывается за словом «доказано».
У многих теорий есть границы применимости. Физики делают наблюдения, стараются их обобщить, строят гипотезы, стараются прийти к теории, понимают насколько мы слаборазвитая раса, когда смотрят на ТО и КМ и имея эту базу наблюдений строят предположения, что скорее всего будет, если залететь в черную дыру.

Конечно, никто не знает на самом деле и не узнает пока не залетит. Может вообще сервак, на котором наша вселенная запущенна упадет с NullPointerException.

Физики как раз довольно скромны и понимают, насколько они далеки от познания мира.
есть сверхмассивные черные дыры, массой в миллиарды солнечных масс, а радиусом больше орбиты Урана


Просто физика не знает сил, которые удержали бы массу в 1 миллиард масс Солнца за пределами сферы радиусом с большую полуось орбиты Урана. Но при этом видит гравитационное влияние таким массивных тел на окружающие звезды.
Конечно тут нужны уточнения, скажем возле нашей центральной ЧД нашли спустник массой где-то 1300 мас Солнца. Как минимум — это дает ограничение на радиус центральной «а вдруг не ЧД». Дает ли это ограничение сказать «совокупность нейтронных звезд массой 4 миллиона масс Солнца не может быть втиснута в сферу радиусом 11.8 миллионов км без коллапса» — это я не знаю.
Я ни разу не противник англицизмов, но вот скажите, чем «oversimplification» лучше «переупрощения»?
тем что слово «переупрощения» я никогда не слышал
думаю, правильно сказать «слишком сильное упрощение», но это длинно
Чрезмерное, излишнее.
Я люблю научно-популярные передачи, и все что вы написали говорилось в них 10-20 лет назад. Не очень понятно, что и кого вы тут «опровергаете».
Чёрные дыры — это замечательно, спасибо! Но в физике полно всяких приколов и ближе к земле. Например, контрруление на мотоцикле. Будет об этом?
напишите
Как мотоциклисту интересно, что в нём прикольного и необычного?
Как мотоциклист мотоциклисту: оно контринтуитивно, заставляет преодолевать свой «здравый смысл», поэтому может быть отнесено к бытовым парадоксам физики
Обычный обратный маятник. На нём очень наглядно видно — что почему.
Менее интуитивно для большинства мотоциклистов (рефлексы!), что для объезда мелкого препятствия оно не трубуется. И даже сохранения равновесия на время объезда не надо.
«теория Большого взрыва это НЕ теория о большом взрыве (t=0)… Она не отвечает на вопрос, в частности, почему этот большой взрыв произошел.»

Если можно, расскажите в следующий раз о таких теориях. Например, о теории вечно пульсирующей Вселенной Николая Горькавого. Правда, у его Вселенной есть центр, но нет того, что расширяется — кроме пустоты с абстрактной метрикой пространства-времени.
В других теориях наверняка есть более ощутимая среда, которая и взорвалась при своём образовании, а теперь при расширении раздвигает скопления галактик. Только среда для этого должна быть плотной…
Раздвигает скопления среда?
Мне вот нравится «абстрактная метрика» как свойство пространства, правда под «вторым горизонтом» ЧД это понятие уже будет плохо работать.
И даже непонятно, есть ли этот горизонт. В идеальной Керр метрике есть, но внутри находятся замкнутые временные циклы. А квантовая механика вроде активно сопротивляется возникновению временных циклов — при приближении геометрии к временному циклу начинается генерация ливня частиц, масса которых меняет геометрию и не дает сформироваться горизонту. В общем, тут пока все непонятно
Квантовая механика с неопределенностью — это единственный способ решения временного цикла вида «решил человек убить свою биологическую мать за год до своего рождения». Только Вам может не понравится мир в радиусе 1 световой год от точки «решения парадокса». Ведь главным единственным постоянным условием в «этом проклятом мире» (© Bethesda) будет сам факт возникновения «пути энергии и информации в прошлое» (какой бы физический принцип этого не требовал). А может возникнет несколько таких «проходов» и по сравнению с таким сюжетом «Терминатор: Генезис» покажется очень вероятным развитием событий.
Раздвигает скопления среда?

Сейчас этот товарищ начнёт про эфир вещать (это он всё раздвигает). И про то, что вакуум — неподходящее название для вакуума, и из-за этого лингвистического косяка в физике проблемы. То ли дело — его теории. Естественно, без формул, ибо не барское это дело, математикой обмазываться.

Для того, чтобы замедление времени было бы таким, планета должна была быть на расстоянии чуть более 0.003% от радиуса Шварцшильда.


У Кипа Торна есть целая книга (Интерстеллар: наука за кадром), в которой он рассказывает, как натягивал сову на глобус в разных аспектах.

Посмотрите её — там хоть и очень натянуто, но здравому смыслу ничего не противоречит. В частности, про замедление времени (дальше прямая паста почти целой главы из книги. Можете глянуть, чтобы увидеть: действительно, очень натянуто, но может.)

Когда Кристофер Нолан сказал мне, какое замедление времени на планете Миллер ему нужно – один час там на семь земных лет, – я был ошарашен. Я полагал это невозможным, о чем и сказал Крису. «Это не обсуждается», – отрезал он. Что ж, не в первый и не в последний раз я отправился в раздумьях домой, сделал кое-какие расчеты и… нашел выход.
Я обнаружил, что если планета Миллер будет настолько близко к Гаргантюа, насколько это возможно без риска упасть в черную дыру[36], и если скорость вращения Гаргантюа будет достаточно высокой, замедление «один час за семь лет» возможно. Но Гаргантюа должна вращаться чертовски быстро.
Для скорости вращения черных дыр есть предел. Если он будет превышен, горизонт событий исчезнет, оставив на виду у всей Вселенной обнаженную сингулярность. А это, по всей видимости, противоречит законам физики (см. главу 26).
Выяснилось, что для замедления, которое нужно Крису, Гаргантюа должна вращаться со скоростью, близкой к предельной, меньше ее примерно на одну стотриллионную долю[37]. В Кип-версии я по большей части исхожу из этого значения.

[...]
В 1975 году я обнаружил механизм, с помощью которого природа предохраняет черные дыры от превышения предельной скорости вращения: когда скорость близка к предельной, черной дыре сложно захватить объект, который летит по орбите в ту же сторону, что вращается она, и который, будь он захвачен, увеличил бы скорость ее вращения. Однако черная дыра с легкостью захватывает объекты, летящие в сторону, противоположную направлению ее вращения, то есть те объекты, захват которых уменьшает скорость вращения черной дыры. Поэтому черная дыра легко замедляется, как только скорость ее вращения приближается к предельной.
В моем тогдашнем исследовании я уделил особое внимание газовому диску (он напоминает кольца Сатурна), который вращается в одном направлении с черной дырой. Этот диск называется аккреционным (см. главу 9). Силы трения в диске вынуждают газ постепенно, по спирали, переходить в черную дыру, увеличивая скорость ее вращения. Кроме того, трение нагревает газ, и он излучает фотоны. Завихрение пространства вокруг дыры захватывает эти движущиеся по ходу ее вращения фотоны и отбрасывает их прочь, из-за чего они не могут попасть внутрь. И напротив, завихрение захватывает фотоны, которые движутся в сторону, противоположную вращению дыры, и засасывает их внутрь, где они замедляют ее вращение. В итоге, когда скорость вращения черной дыры достигает 0,998 от предельной, устанавливается баланс, при котором замедление за счет захваченных фотонов в точности компенсирует убыстрение за счет поступающего в дыру газа. По-видимому, этот баланс довольно устойчив, и в большинстве случаев можно ожидать, что скорость вращения черной дыры не превышает 0,998 от предельной.
Однако я могу вообразить ситуации – очень редкие, если вообще встречающиеся в реальной Вселенной, и все же возможные, – когда скорость вращения подходит к предельной гораздо ближе, и даже настолько близко, насколько это требовалось Крису, чтобы замедлить время на планете Миллер: скорость на одну стотриллионную долю меньше предельной. Маловероятно, но возможно.
В кино это не редкость: чтобы снять шедевр, режиссер часто доводит все до предела. В фэнтезийных фильмах вроде «Гарри Поттера» этот предел находится далеко за границами научной достоверности. В научной же фантастике он, как правило, остается в границах вероятного. Собственно, это главное отличие между фэнтези и научной фантастикой. «Интерстеллар» – научная фантастика, а не фэнтези. Сверхбыстрая скорость вращения Гаргантюа с научной точки зрения возможна.


Спасибо. Очень интересно. Надо почитать.

True or false?
1) Внутри горизонта ЧД есть стабильные орбиты по которым можно вращаться вокруг сингулярности довольно долгое время
2) Вселенная с positive curvature (у нашей она равна нулю насколько я знаю) эквивалентна ЧД с очень большой массой и оч большим горизонтом. Причина — «закольцованность» Вселенной и невозможность выбраться за пределы горизонта.
Вселенная могла бы быть закольцованной как минимум для массивных частиц материи, находящейся в ней (критическая плотность, кажется даже с учетом ТЭ, была весьма близка к реальной — 0.99..9 от критической). Но плотность оказалась ниже критической и через миллиарды лет Вселенная перешла (опять) на стадию расширения с ускорением.
Tzimie
Более того, они так никогда и не упадут в черную дыру, ведь для этого нужно бесконечное время — ну, вы поняли что это совсем не так

То есть — всего лишь сменой метрики, мы убеждаемся что черные дыры вполне могут образоваться? — Супер! — А нельзя ли таким «простым» манёвром доказать (или опровергнуть) почти все явления в ОТО?

То есть, если что-то где-то в ОТО запрещено или «зависает», то «меняем метрику» и «проталкиваем» это куда-нибудь?

Сменой метрики нельзя ли вообще убрать сингулярность в ОТО? Или «расширить проход» между кротовыми норами?
Я там написал, что в запале перепутал слова — не метрики а системы координат.
Бесконечность на горизонте устраняется сменой системы координат, а в центре ЧД — нет
Tzimie
Бесконечность на горизонте устраняется сменой системы координат


Раньше мы смотрели в телескоп и видели как всё вещество «зависает», бесконечно падая в ЧД, но всё упасть не может и поэтому мы решили что ЧД не существует.

Теперь, мы смотрим в телескоп и видим, что хоть и происходит замедление падения, но оно НЕ бесконечно, — а за какое время внешний наблюдатель увидит как некая глыба вещества (звезда, пыль, корабль или космонавт) упала в ЧД?
>мы смотрим в телескоп и видим, что хоть и происходит замедление падения, но оно НЕ бесконечно

Не так. Замедление времени при свободном падении вообще не определено, так как траектория не замкнута, и падающий наблюдатель никогда не сможет снова синхронизовать часы

Визуально же замедление бесконечно, но
1. Это чисто оптический эффект
2. Помимо замедления объект краснеет и блекнет, и в итоге пропадает из виду
Поэтому ответ на ваш вопрос — никогда
Tzimie
Помимо замедления объект краснеет и блекнет, и в итоге пропадает из виду
Хорошо, переформулирую свой вопрос:

1) Мы установили телескоп и наблюдаем миллионы лет за чёрной дырой.

2) В начале наблюдения мы установили массу чёрной дыры и её размер (косвенно, конечно).

3) Мы видим как на чёрную дыру валятся звёзды, как их разрывает, как они издают «предсмертные крики» и мы видим как падающий на чёрную дыру «объект краснеет и блекнет, и в итоге пропадает из виду».

4) Мы постоянно измеряем (как обычно, косвенно) массу и размер нашей наблюдаемой чёрной дыры.
Вопрос — мы можем практически установить (определить) из нашего наблюдения, что и масса и размер нашей чёрной дыры изменились (увеличились)?
Массу именно по изменению метрики (типа Шварцшильда)? Ну скажем грав. линзирование считается? Если что, как нас учил препод, в грав. линзировании с точки зрения изменения фазы фотонов есть 2 компоненты:
1. Чисто геометрический — пройденный фотоном путь длиннее, чем прямая между точкой излучения фотона и точкой приёма.
2. Чисто эффект ОТО — изменение времени пролета фотоном этого участка траектории в той условной СО, в которой считается набег фазы элементарной частицы.// ЕМНИП

И по причине линзирования ЧД будет закрывать звезды в секторе пространства меньшем, чем чисто геометрический телесный угол, перекрываемый объемом сингулярности. Но, по логике, этот угол будет расти с ростом массы ЧД.
Массу астрономических обьектов сейчас определяют тремя путями:
1)По результатам их взаимодействия на другие объекты
2)По гравитационному красному смещению ЭМ волн, излучаемых обьектов, но это имеет смысл, если точно известна скорость, относительно измеряющего, что бы вычесть эффект Доплера.
3) Ну и недавний по гравитационным волнам, хотя метод такой себе.
Да, грав. красное смещение без допплеровского нельзя оценить. А если речь конкретно о ЧД, то кроме сверхмассивных мы их видим только в двойных системах. А двойная система ещё нужна т.н. «тесная», то есть желательно с аккрецией вещества хотя бы желтого карлика на ЧД. А это уже хотя бы какая-то скорость движения ЧД по орбите, даже если это будет 0.7 от орбитальной скорости Нептуна.
Массу астрономических обьектов сейчас определяют тремя путями:
Вопрос не в этом, не в определении массы и не в способе определения массы, вопрос — в определении (и возможно в способе) ИЗМЕНЕНИЯ массы чёрной дыры.

Можем ли мы практически установить (определить) из нашего наблюдения, что и масса и размер нашей чёрной дыры изменились (увеличились)?

Дело в том, что все падающие на чёрную дыру звёзды — «краснеют и блекнеют, и в итоге пропадают из виду», а их замедление при падении есть «визуально чисто оптический эффект».

Как бы и наше измерение изменения массы чёрной дыры, не оказалась бы также «чисто оптическим эффектом», выражаясь словами Tzimie?
Некая масса падала на ЧД и объем сингулярности вырос (из общих соображений он растет всегда или хотя бы не падает, если не считать испарения).
Чисто формально у нас «объем сингулярности» определяется массой, правда желательно измерять массы не очень малыми порциями, скажем по 1 грамму. Для вращающейся ЧД конечно будут свои эффекты.
Про «краснеют и бледнеют» — если точно знать, что источник фотона движется перпендикулярно лучу зрения (он падал, но внезапно перешел на круговую орбиту «под нами»), то изменение частоты будет описано чисто ОТО.
Конечно.
Вы можете взять пробное тело вдали от ЧД и смотреть, как ЧД искривляет его траекторию. То есть измерять The Newtonian limit for nearly flat space-times

Tzimie
Конечно
Отлично.

Что имеем — Мле́чный Путь (также наша Галактика или просто Галактика) — Наблюдаем — за время наблюдения на сверхмассивную чёрную дыру (Стрелец A*) упало звёзд около 4 миллиона масс нашего Солнца.
По итогам наблюдения масса чёрной дыры (Стрелец A*) составляет уже около 4,3 миллиона масс нашего Солнца.

Вывод — начальная масса — на начало наблюдения — чёрной дыры (Стрелец A*) — 0,3 миллиона масс нашего Солнца.

Но, согласно как пишет Tzimie, мифу, ни один грамм за время нашего наблюдения не упал на чёрную дыру (Стрелец A*).

Но куда же делись все эти 4 миллиона масс нашего Солнца?
Tzimie утверждает, что они «краснеют и блекнеют, и в итоге пропадают из виду». Хорошо. Из виду то они пропали, но в чёрную дыру (Стрелец A*) они провалились или нет?

Визуально, масса чёрной дыры (Стрелец A*) возросла, это мы определили по «линзированию» и по «пробное тело вдали от ЧД и смотреть, как ЧД искривляет его траекторию».

Но теоретически(!) вся эта масса (все эти 4 миллиона масс нашего Солнца), по мнению нас, наблюдателей чёрной дыры (Стрелец A*) с планеты Земля (а не на бесконечном «далеке» и не падающими в чёрную дыру (Стрелец A*) никогда не упадёт на чёрную дыру (Стрелец A*).

Но если принять, что… типа нам всё равно, стала ли масса чёрной дыры (Стрелец A*) около 4,3 миллиона масс нашего Солнца или так и осталась около 0,3 миллиона масс нашего Солнца с "навечно зависшими недалеко, но так и не упавшими на чёрную дыру" около 4 миллиона масс нашего Солнца — то раз мы нашу теорию (ОТО) практически проверить в нашем случае (мы с Земле наблюдаем) не можем, то практически этот вопрос не имеет значения. — Так, Tzimie?

P.S. Это аналогично тому, что мы провожаем поезда в Москву из Смоленска, к примеру.
Мы видим, как народ садится в поезд, поезд трогается в направлении Москвы и исчезает из виду.
Но мы знаем, что теоретически у этого народа, севшего в этот поезд, нет денег чтобы купить квартиру в Москве и жить там.
Но по тому, сколько колбасы, вина, мяса и бананов везут в Москву товарные поезда мы понимаем, что население Москвы реально вырастает.
Проверить прямо — кто употребляет всю эту провизию мы не можем (москвичи ли или «застрявшие около МКАДа толпы народа»), но практически этот вопрос для нас не имеет значение, так как вся эта еда всё равно движется к Москве и не возвращается обратно.

P.P.S. Так что миф это (что чёрные дыры не растут) или нет — практически не проверяем нами. Теоретически (глядя с Земли) не должны расти вовсе, с момента своего рождения (коллапса звезды), а практически они не отличаются от тех чёрных дыр которые могли бы как-то и вырасти. Но как — мы не знаем. По крайней мере, глядя на чёрные дыры с Земли.

Я не в курсе идеи о том, что когда-то масса была именно 300 тысяч, но понятно, что когда-то так было. И это конечно не НЗ и не забывший провести коллапс «сверх-сверхгигант» был.
Но, Вы ни по какие эффектам гравитации не сможете понять, ближе эта масса некоего предела к центру ЧД или нет. Есть оценка радиуса:
не более 45 а. е.

Другая оценка — где-то рядом болтаются остатки звезды со скоростью 1000 км/с. В нерелятивистском приближении это может быть на расстоянии 89875 грав. радиусов, а может — и на немного другом. Указанная мною величина выходит 1.04-1.24 трлн. км, то есть явно больше указанного радиуса.
Жители третьего города уже смирились с тем фактом, что хорошая медицина, ВУЗы, театры и музеи есть только в столицах. Высокооплачиваемая или просто квалифицированная работа – тоже там. Хочешь это всё получить – вливайся! Москва нерезиновая, она, как чёрная дыра. Поглощает все ресурсы, напоминая о своём присутствии лишь фактом наличия притяжения. А что даёт взамен? Неэффективное управление, грабительскую региональную политику и сомнительную культуру. Спасибо.

Они нас читают!?
А что если представить это так: за конечное (для нас) время эти падающие остатки подойдут достаточно близко к горизонту событий, что с нашей точки зрения совокупная масса исходной ЧД и этих остатков образует новый горизонт событий — больший предыдущего — который включит упавшее и «зависшее» вещество? Считал ли кто-либо, сколько времени с т.з. удалённого наблюдателя необходимо падающему объекту, чтобы подойти к горизонту событий ЧД звёздной массы на планковское расстояние?
Есть ещё проблема. «Подходить на планковское расстояние» должны не отдельные атомы, а сгустки вещества с массой не меньше планковской (много атомов, но на порядки меньше массы Homo). Каждая такая кучка, в один момент падая на горизонт, увеличивает грав. радиус (правда в приближении сфер. симметрии массы и без вращения) на 1 длину Планка.
А так, моё мнение в том, что только слияние 2 сингулярностей можно «легко» описать с точки зрения ОТО. То есть, приближение любой массы к ЧД повышает размер сингулярности (сингулярность «выпячивается» в сторону гравитирующего «второго тела»).
А дальше уже могут быть разные теории. Например, без любых знаний по КТП я предполагаю, что «площадь горизонта» квантуется с шагом скажем 4*pi*(lPlanck)2.
По-моему, Брайан Грин об этом писал где-то, в окнтексте голографического принципа.

Наивный вопрос: тело падает под горизонт, в этот момент происходит указанное «выпячивание» сингулярности.

а) Горизонт событий — не физический объект, стало быть, может двигаться быстрее СС?
б) Может ли выйти так, что в итоге «выпячивания» уже «нырнувший» под горизонт объект снова оказывается снаружи?

Вроде бы, не должно, но я нигде не видел проверок, почему. На демках LIGO симуляциях слияния ЧД, кстати, эти «выпячивания» были очень чётко прорисованы.
a. Да
b. нет

Еще (surprise, surprise) черные дыры могут быть вложенными (но не долго).

Вообще я рекомендую почитать про разницу между абсолютным горизонтом и кажущимся (apparent)
Спасибо, не читал об этом раньше, очень интересно.
Горизонт событий — не физический объект, стало быть, может двигаться быстрее СС?

Любое изменение формы горизонта вызывается перемещением реальной материи (под ним или над ним). Движение этой материи идет не быстрее скорости света (со скоростью света, если мы учтем сверхмалое перемещение глюонов внутри каждого нуклона) — > горизонт не может расти быстрее скорости света.
В рассмотрении слияния ЧД кажется выполняется аксиома «объем пространства под горизонтом всегда растет» (очевидно для падения «пробного тела» по спирали с нулевым эксцентриситетом). Но «очевидно» тут с точки зрения СТО, как измерять «объем» или «площадь» сингулярности я по честному не знаю.
Действительно, при движении гравитирующего тела вокруг ЧД наш «бугорок» горизонта должен описывать окружность вокруг «невозмущенной сингулярности». А может ещё возникает «энтропия ГВ» (энтропия всегда не падает, но может часть энтропии системы двух ЧД унесут волны) и другие странные явления.
Не совсем
При слиянии двух дыр горизонты могут образовывать нечто вроде «гильетины» и отхватываемая горизонтом область может двигаться быстрее скорости света.
Как минимум я могу предположить, что относительно ЧД №1 горизонт событий может двигаться со скоростью сближения этих 2 ЧД. Это в любой СО меньше скорости света, но с какой-то точки зрения может «гильйотина» схлопываться в смысле 2 частей с вектором суммарной скорости достаточно большим.
Но если для наблюдателя возле горизонта в «рел. замедленном» времени — очень быстро будет «захлопываться петля».

https://images.app.goo.gl/ek2ngyEmLYUc94Yn7


Тут показан реалистичный коллапс
Как видите, в коллапсирующей звезде горизонт возникает из центра и распространяется быстрее скорости света

Честно не вижу:)
И возникновение ЧД я не рассматривал, я рассматривал наличие хотя бы одной и падение вещества на неё или массивного тела (НЗ или ЧД).
А вообще, Вы возможно правы. Образование ЧД — это именно падение «большого числа материи», правда большое — это «планковская мощность», в терминах массы ЧД M будет

dM/dt = c3/G,
если не забыл размерности величин.
Там в самом начале горизонт расширяется так быстро, что линия близка к горизонтальной (а свет наклонен под 45 градусов).
Я вижу, что угол достигает 45 градусов, но это уже «процесс движения вещества внутри сингулярности». В рассмотрении такого явления я точно не специалист.
Но если Вы про эту часть:
Под выделенным красным
image

То там в начале процесса действительно быстрый процесс в терминах dr/dt.
Не сингулярность а горизонт
Я совсем не специалист в ОТО. Некоторые понятия постулирую вообще из ньютоновской механики, в которой изначальное определение ГС — сфера радиуса rg, на которой «Пот. энергия + Кин. энергия» = 0 даже при v = c. Очевидно, что такое представление не работает даже в СТО.
В частности, из такого представления выходит, что прямо при слиянии сфер радиусом rg (сближения центров масс на r = rg(m1) + rg(m2)) возникает сингулярность в форме лемнискаты, но, скорее всего — даже большего объема.
Pavel_L
Считал ли кто-либо, сколько времени с т.з. удалённого наблюдателя необходимо падающему объекту, чтобы подойти к горизонту событий ЧД звёздной массы на планковское расстояние?
Планковское расстояние — это уже вне теории ОТО.
Все же не факт. На таком расстоянии от горизонта конечно будут большие проблемы с принципом неопределенности — получаем неопределенность по импульсу (радиальная компонента) delta_p = h/(4pi)/LPlanck
Это импульс либо не дает частице упасть на ЧД, либо наоборот — толкает ещё глубже.
Ещё нашел замечание, что элементарную частицу не хорошо определят по координате с такой точностью, что неопределенность по импульсу достигнет mc.
Приблизительное объяснение, ИМХО, выглядит в такой формуле.
Пускай p1 = mc, p0 = 0. Разница кинетической энергии этих 2 «неопределенных состояний» будет
T1T0 = ((mc^2)+(mc^2))^(1/2) — mc^2 = (21/2-1)mc^2
В принципе это не очень много, на рождение пары «электрон-позитрон» в случае электрона (m = me) не должно хватить.
Но выходит, что поведение одного электрона сложно описать на расстоянии r таком, что
((mc^2)^2 + (hc/(4pi)/r)^2)^(1/2) — mc^2 = 2mc^2 ->
(mc^2)^2 + (hc/(4pi)/r)^2 = 9(mc^2)^2 ->
r = hc/(4pi)/(23/2*mc2)1/2
Victor_koly
Приблизительное объяснение, ИМХО, выглядит в такой формуле.
Пускай p1 = mc, p0 = 0. Разница кинетической энергии этих ...


Энергия — это уже вне теории ОТО.
Как-то локально энергия должна сохранятся. Действительно, если мы дойдем до релятивистских скоростей и разности грав. потенциалов между точками траектории c^2 — будет сложно применять подобные теории.
Но я в курсе того, что с объединением ОТО с КМ есть много проблем (даже если речь не про квантовую теорию поля). И я даже не знаю, выйдет ли доказать, что обнаруженные грав. волны требуют наличия переносчика взаимодействия со спином 2.
Вопрос не в этом, не в определении массы и не в способе определения массы, вопрос — в определении (и возможно в способе) ИЗМЕНЕНИЯ массы чёрной дыры.

Вообще так и происходит, масса ЧД для наблюдателя удаленного на бесконечность и для наблюдателя находящегося в окрестностях ЧД не одна и та же величина и это касается не только массы, но и многих других параметров связанных с метрикой пространства-времени.
Можем ли мы практически установить (определить) из нашего наблюдения, что и масса и размер нашей чёрной дыры изменились (увеличились)?

Если считать, что мы система удаленная на бесконечность от какого либо релятивистского объекта(ЧД, НЗ), то для нашей системы никаких изменений не будет, пока мы не начнем приближаться слишком близко к релятивистскому объекту.
Как бы и наше измерение изменения массы чёрной дыры, не оказалась бы также «чисто оптическим эффектом», выражаясь словами

Нет это не оптический эффект, и вы сможете это на себе ощутить опять же приближаясь к РО, где на вас со стороны РО будет действовать силы значительно превышающие GM/R^2.
Можем. По изменению картины линзирования как минимум.
Скажем если была ЧД массой 3 массы Солнца и «скушала» за период измерений 0.03 массы С. — нужны весьма точные измерения, чтобы заметить это (по картине линзирования или чему либо другому).
Я понял так что речь о принципиальной возможности.
en.wikipedia.org/wiki/Schwarzschild_metric#Alternative_coordinates

The singularity at r = rs is an illusion however; it is an instance of what is called a coordinate singularity. As the name implies, the singularity arises from a bad choice of coordinates or coordinate conditions. When changing to a different coordinate system (for example Lemaitre coordinates, Eddington–Finkelstein coordinates, Kruskal–Szekeres coordinates, Novikov coordinates, or Gullstrand–Painlevé coordinates) the metric becomes regular at r = rs and can extend the external patch to values of r smaller than rs
Кажется, это тот самый случай, когда заметка написана ради неё самой. Ничего экстраординарного я из неё не узнал (и про Интерстеллар тоже). Про черные дыры со всеми параметрами было написано еще в какой-то переводной книге (лень сейчас искать точное название, но в принципе могу), изданной еще в середине 1980-х (когда даже Хокинг еще не начал писать научпоп).
Новиков Игорь Дмитриевич. Чёрные дыры и Вселенная. — М.: Молодая гвардия,
1985. — 190 с., ил. — Серия «Эврика». Была такая дома в детстве)
Помню ее)
Раскопал свой школьный реферат по астрономии, который потом пересдал как допуск к кандидатскому минимуму :) и нашел там такие ссылки:
И.Д. Новиков — в журнале «Земля и Вселенная» в 1970-80х. Наверное, то что было издано в серии «Эврика» — это компиляция из статей.
Книга, про которую я говорю, это Иан Николсон «Тяготение, черные дыры и Вселенная». М., 1983.
Вот что давно интересует: в дыру падает материя и свет, и на горизонте свет отскакивает от материи обратно и застревает на горизонте. Получается, чёрная дыра — «белый шар из света»? Но такое не происходит, потому что дыры не идеально стабильны, так? Да и материя не перестаёт ведь падать, постоянно снося этот слой из фотонов.
Фотоны могут рассеиваться на плотной плазме вокруг горизонта. Если это плазма имеет элетронные свойства, близкие к металлу, то существенная часть фотонов опредленного диапазона длин волн может рассеяться наружу. Если эта плотность электронов высокая (скажем как плотность 1s-электронов в железе при н.у.) — может быдет сильно рассеиваться даже фотоны энергией до 1 МэВ.
А при большей энергии фотоны начнут рождать электрон-позитронные пары, 1 из частиц пары упадет за горизонт (не путать с «правильным» и «неправильным» объяснением излучения Хокинга).
Если брать идеальное статичное решение (где размеры ЧД не меняются никогда), то да, формально горизонт событий это null surface, так же как границы световых конусов обычных. Свет, излученный на горизонте в сторону «наружу», остается на горизонте. На диаграмме Крускала это особенно хорошо видно.

Там есть правда еще одна хитрость: поверхность эта не трехмерна (2д сфера х 1д время), а двухмерна, т.к. время на горизонте не тикает совсем. Что это значит физически — хороший вопрос. Но геометрически по ОТО выходит так.
Как провести светоподобную траектории фотона по предельно измененной метрике на горизонте — такой вопрос у Вас получается?
У меня остался один вопрос: с точки зрения внешнего наблюдателя, падающий объект застревает где-то на горизонте событий. Однако, в результате испарения ЧД, когда-нибудь горизонт событий начнёт уменьшаться, соответственно, наблюдатель будет видеть, что падающий объект приближается к центру ЧД, пока она не испарится полностью? То есть, так никогда и не пересечёт горизонт?
Свет от падающего в ЧД тела будет испытывать красное смещение. Поэтому сначала в видимом диапазоне он будет краснеть, затем перейдет в инфракрасный, затем в радиодиапазон. Так красным и растворится.
Тут интересно рассмотреть такую границу «глубины источника фотонов». Пускай у нас на ЧД падают электрон и протон, которые в любой момент могут рекомбинировать и выдать фотон с энергией не ниже 13.5 эВ (то есть скажем если они и связаны изначально, то находятся на уровне не ниже n = 12).
В какой-то момент испускается этот УФ фотон и пробует покинуть грав. колодец ЧД. Вопрос: «Начиная с какого растояния до горизонта (в какой бы метрике мы его не смогли измерить) за время отлета фотона на расстояние скажем 1.5 грав. радиусов от центра ЧД длина волны фотона превысит пройденный путь?».
Или не может быть такого сильного красного смещения? Я конечно подозреваю, что такой «путь фотона» займет не один миллиард лет. Или можно заменить на условие «достигнет длины волны 1.5 грав. радиуса ЧД».
Или не может быть такого сильного красного смещения? Я конечно подозреваю, что такой «путь фотона» займет не один миллиард лет. Или можно заменить на условие «достигнет длины волны 1.5 грав. радиуса ЧД».
Теоретически нет предела для длины волны при красном смещении, практически же фотон растворится в реликтовом. Это в идеализированной модели. На практике многое зависит от конкретной ситуации. Есть комптоновское рассеяние, которое ускорит снижение частоты фотона, есть обратный эффект Сюняева — Зельдовича (увеличение частоты фотонов за счет рассеяния на горячих электронах падающего в ЧД газа).
По поводу горячих электронов и газа. Если фотон потеряет энергию до скажем 5 кэВ (в СО ЦМ с налетающим электроном), то очень слабо сможет изменить свою энергию за каждый акт комптоновского рассеяния.
А если Вы говорите про реликтовое — фотонов конечно во Вселенной много (в миллиард раз больше, чем электронов, протонов и нейтронов?), но сколько из них долетят из сферы радиусом 15 Мпк до конкретной ЧД, расположенной в конкретной точке скажем галактического диска спиральной галактики?
По поводу горячих электронов и газа. Если фотон потеряет энергию до скажем 5 кэВ (в СО ЦМ с налетающим электроном), то очень слабо сможет изменить свою энергию за каждый акт комптоновского рассеяния.
Рассеяние на горячих электронах это обратное комптоновское — фотон приобретает дополнительную энергию и частота растет. То что слабо (потеряет при комптоновском)… лень считать цифры, честно говоря, но не спорю. И тем не менее это вносит вклад в смещение длины волны.

но сколько из них долетят из сферы радиусом 15 Мпк до конкретной ЧД
Ну сколько долетело до WMAP и Планк? ) Почему в окрестностях нашего наблюдателя их должно быть недостаточно?
Если мы говорим о потери энергии фотоном, значит фотон энергию теряет. Я честно не помню, может формула эффекта Комптона совсем не для релятивистской скорости э-на, но скорость ведь нужна в системе ЦМ, чтобы быть релятивистской? Так вот формула там говорит, что изменение длины волны фотона
d_lambda = (0..2)*lambdaCompton
То есть, если фотон имеет длину волны lambda0, равную 1/200 комптоновской длины, то за каждый отдельный акт он будет рассеивать не более 1% энергии (ну или считайте точнее, lambda <= 1.01*lambda0 становится).
Но рассеивание такое будет много раз, все же вокруг ЧД должно быть много свободных электрона.

По поводу «долетело» — согласен. А если то 1 небольшой телескоп на орбите вокруг Земли, то в грав. поле ЧД «окончательно линзированных» фотонов попадет куда больше.
Возможно, я неудачно выразился, меня интересует не столько изображение, сколько то, что, насколько я понимаю, по часам внешнего наблюдателя падающий объект никогда не пересечёт горизонт событий, в то же время, как по этим же часам ЧД испарится за большое, но конечное время.
Верно ли я это понимаю, или тут тоже не все так просто, может, всё-таки, Tzimie тоже глянет вопрос?
Тут надо рисовать диаграммы (одна с испарением где то уже проскакивала, столько комментов что уже не найду) а не оперировать понятиями «по часам внешнего наблюдателя» — такие понятия просто притягивают к ошибкам.
Интересно, практически случайно сегодня наткнулся на это обсуждение:
Кроме того ОТО утверждает, что процесс образования черной дыры неосуществим за конечное время, с точки зрения наблюдателя, все время находящегося снаружи черной дыры. Специальная и общая теории относительности утверждают, что у каждого наблюдателя есть свое собственное время. По часам наблюдателя, падающего в черную дыру, он пересечет горизонт за конечное время. Если же наблюдатель находится все время снаружи черной дыры, то с его точки зрения процесс падения в черную дыру занимает бесконечное время. То есть с точки зрения любого стороннего наблюдателя формирование черной дыры — это бесконечный асимптотический процесс падения материи, который никогда не кончается. Иными словами, вся материя, которая составляет черную дыру, с точки зрения наблюдателя снаружи, вечно находится над горизонтом. Тогда как же вообще сторонний наблюдатель что-то теряет из виду? Данную проблему, однако, можно обойти и объяснить, как черная дыра формируется за конечное время с точки зрения стороннего наблюдателя, но там много чего все еще остается неясным.
Кстати, чтобы полностью «раствориться», если под этим вы подразумеваете увеличение длины волны света в бесконечность, потребуется бесконечное же и время, в то время, как испарение ЧД, опять же, произойдёт несколько раньше.
увеличение длины волны света в бесконечность, потребуется бесконечное же и время
Почему? Красное смещение зависит только от скорости (ускорения?) удаляющегося объекта. А так как на подлете к горизонту она околосветовая, то и растворится объект где-то в окресностях горизонта полне себе быстро. А пока гипотетическое хоккинговское испарение для ЧД около 3х солнечных масс ~60млрд. лет. Если я ничего не напутал.
Нужно глянуть, на Вики были формулы для ЧД нормальной массы (much larger than 1017 grams).
Для массы 1,9885*1033 грамм выходит 6.8*1073 секунд. Короче, Вы немного ошиблись:)
Красивая цыфра 10100 секунд выйдет для массы свыше 1.029*1042 грамм или 517.5 миллионов масс Солнца. Интерестно — как сильно разлетится Вселенная с гугол секунд от того, как потухнут все звезды и смогут ли в такой «неплотной» Вселенной собраться много ЧД массой свыше полумиллиарда солнечных?
Но она ведь «около», а не световая? Я не имею в виду что именно в видимом диапазоне его не будет видно, это понятно, но не очень интересно, но хоть какой-то очень большой длины радиоволны же от него все ещё будут идти?
Будут. Ровно до момента как мы не сможем детектировать эти волны (например с длиной волны 100 000км). Тогда все, потеря информации, считай что объект перестал существовать и ушел под горизонт (к слову в этот момент он по своим часам уже давно где-то в сингулярности). Если вы спросите «А что если построить гипотетический приемник для любых длин волн» — я не знаю )
Гипотетический сигнал с длиной волны 0.5 а.е. на расстоянии 5 а.е. будет сильно расширен дифракцией. И да, каждый фотон, испущенный с энергией 1 эВ может уменьшить свою энергию до 1 мэВ (почти реликтовый фон по порядку величины). Но, скажем это был источник мощностью 1.6 Вт (возьмем монохроматичность лазера как d_lambda 0.1 нм, энергия фотона 1 эВ) — тогда он испускал 1019 фотонов, превратятся они на небольшом расстоянии от горизонта в тоже число фотонов и мощность 1.6 мВт, размазанную по некому характерному углу (порядка 1.22 lambda/D).
Да, указанная мною мощность говорит, что точка испускания фотонов уже имеет замедление времени 1000 от точки регистрации, за время пролета скажем 5 а.е. «лазер» упал ещё немного глубже. Если он заодно разогнался хотя бы до gamma = 500 — ещё поправки на обычный эффект Доплера учесть нужно?
Скорее, пересечет и исчезнет одновременно с исчезновением самой ЧД в ее финальной вспышке. См. диаграммы Пенроуза для испарающихся дыр. Например fias.uni-frankfurt.de/~hossi/Bilder/BR/bhevap.jpg
(если забить на красное смещение)
А никакого поясняющего текста к этой картинке нет?
Много их. В частности, backreaction.blogspot.com/2009/11/causal-diagram-of-black-hole.html
При этом неплохо бы преставлять что такое диаграммы Пенроуза и Крускала.
en.wikipedia.org/wiki/Penrose_diagram
en.wikipedia.org/wiki/Kruskal%E2%80%93Szekeres_coordinates
И всё-таки, можно поподробнее рассмотреть теорию космологии чёрной дыры, по которой наша 4-х мерная вселенная — это ЧД в 5-ти мерной вселенной. Она отлично согласуется со свёрнутыми измерениями в Теории Струн и объясняет множество парадоксов, но её почему-то пытаются проработать лишь несколько человек. Почему так?
Почему так?

Скорее всего потому, что ее никак не возможно проверить эксперементально и вряд ли будет когда-либо возможным. Ну сделали модель такой космологии, а дальше что? Все равно это никак не проверяется и не отвечает на вопрос, как 5-ти мерная вселенная выглядит.

Потенциально она никак не отличается от теории, что наша Вселенная это шар в космическом бильярде. Таким теорий может быть сотни и все они одинаково неопровергаемые и непроверяемые.
А может не шар в бильярде, а фермион (кварк или лептон, желательно заряженный и стабильный) в каком-то атоме «вселенной верхнего уровня». Очень старая теория, если что.
Не обязательно верхнего, забавней если симметрично, наша вселенная у них частица, а их вселенная — у нас частица. Топология не запрещает :)
На самом деле, это не я придумал. И вверх и вниз движение такого масштаба читал в этой книге:
Древо жизни
Где-то там была идея, конечно сюжет уже толком не помню. Раньше казалось, что книга из 3 частей, где 2я описывает жизнь в аналоге советского концлегеря.
Забавное совпадение. Первое решение уравнений ОТО описывающее черные дыры нашел человек с фамилией Шварцшильд (Schwarzschild), а «schwarz schild» с немецкого переводится как «черный щит».
Такие себе «мифы»…
Большой взрыв

Это всего лишь концепция того, что наша вселенная однажды появилась, а не существовала вечно, но это уже даже не теория а факт, следует так объяснять теорию БВ для простого народа не больше не меньше.
Замедление времени в черной дыре бесконечно

Нет, оно просто равно 0, никакого времени никакого пространства, никакой энергии и попасть туда нельзя самим не став ЧД.
Приливные силы разрывают всякого еще на подлете к черной дыре

Это не миф, а вот это миф:
Однако, есть сверхмассивные черные дыры, массой в миллиарды солнечных масс, а радиусом больше орбиты Урана. В таких черных дырах лететь от горизонта к сингулярности — часы! именно в такие черные дыры можно провалиться, не заметив этого.

На самом деле сила у ГС возрастает до бесконечности не зависимо от того какая это ЧД, потому что сила по ньютоновской формуле умножается на кофициент замедления времени.
Вот со слиянием ЧД кажется очевидным, что в какой-то момент 2 сферы Шварцшильда объединятся в одну поверхность (которая конечно при наличии любого 2го, 3го, 4го гравитирующего тела не будет сферой), при этом объем сферы постоянно растет. В некоторой гипотезе не может падать площадь поверхности сингулярности, т. к. площадь пропорциональна энтропии.
Но есть небольшой вопрос с ГВ. При слиянии 2 ЧД они уносят энергию, это только кинетическая энергия пары ЧД (огромная величина на расстоянии нескольких грав. радиусов до точки слияния)? Сама масса ЧД на самом деле никуда не девается и не может «уносить энтропию, в не зависимости от того как считать энтропию „искажения метрики“?

По поводу „очевидно“. По имеющимся моделям когда-то (5 миллиардов лет назад) Вселенная точно была меньше, то есть была больше сила грав. взаимодействия между её отдаленными частями. При этом из астрофизики следует, что 5 миллиардов лет назад рождались желтые карлики из того же поколения, к которому принадлежит Солнце. То есть, как минимум — галактики и/или звездные скопления тогда уже существовали.
Но есть небольшой вопрос с ГВ. При слиянии 2 ЧД они уносят энергию, это только кинетическая энергия пары ЧД (огромная величина на расстоянии нескольких грав. радиусов до точки слияния)? Сама масса ЧД на самом деле никуда не девается и не может «уносить энтропию, в не зависимости от того как считать энтропию „искажения метрики“?

Если вы меня спрашиваете, то у меня на это есть несколько вариантов и выбрать какой из них правильный могут помочь только эксперименты и наблюдения, а варианты следующие:
1)Современное понимание массы не верно, а принцип эквивалентности массы-энергии справедлив только для пространства с однородным временем, или в «плоским» пространстве-времени, которого в реальном мире не существует. Как это описать на практике. Представим куб с размером ребра в 10^12 километров и на каждый кубический километр в этом кубе есть 1 атом железа-56 или что бы уже наверняка никеля-62 дабы исключить любые варианты возможной потенциальной энергии вне гравитационных сил.
И так мы имеем N=10^36 атомов Ni-62 каждый с энергией покоя(в собственной локальной системе отсчета) Eо, а так как вещество в этом кубе чрезвычайно разрежено, то можно сказать этот куб находится в практически «плоском» времени-пространстве и относительно любой другой системы отсчета удаленной от этого куба его масса равна M=Eо*N/c^2=1.
Теперь представим, что этот куб сожмется до размеров сферы скажем диаметром в 1 км и на этом остановиться, а энергия этого сжатия постепенно улетучится и никелевый шар остынет практически до абсолютного нуля. С точки зрения любой удаленной системы отсчета энергия покоя каждого из атомов уменьшится, а следовательно и масса всего куба, будет уже равна меньше единицы, но так гласит современная физика в которой масса НЕ АДДИТИВНАЯ величина во всех 4 случаях фундаментальных взаимодействий, а что если это не так и в случае гравитацией масса как раз аддитивная величина и гравитационное сжатие не превращает массу в энергию излучения, а уменьшает энергию покоя массы, относительно СО удаленной на бесконечность которая при этом является функцией количества вещества определенного вида, в данном случае атомов никеля-62. При этом давление создаваемое гравитационным полем в сжатой никелевой сфере, которое имеет отрицательную энергию как бы замещает собой часть той массы улетевшей в виде фотонов остывающего тела, а так как фотоны не имеют массы покоя, то при этом масса не изменится, а останется постоянной относительно удаленной системы отсчета до тех пор, пока давление гравитационного поля не начнет превращать одни частицы на другие, как это происходит в недрах звезд, но в этот процесс уже вовлекаются все остальные фундаментальные взаимодействия в которых масса НЕ аддитивна. И того массу обьекта можно описать как сумму положительной энергии его частиц +модуль отрицательной энергии связи гравитационного поля. При этом если продолжаться сжимать вещество увеличивая его количество то на определенном этапе мы получим сначала нейтронную звезду, а потом и черную дыру. Но при этом масса ЧД уже будет состоять из энергии покоя равной 0, а энергия гравитационной связи = -1 и того масса ЧД будет равна M= 0+|-1|=1
А энергия массы ЧД относительно СО отличной от нее равне Не E=mc^2 а E=-mc^2
При этом энергия гравитационных волн является энергией чисто кинетеческой энергии масс сливающихся ЧД. либо что более вероятно энергия гравитационных волн это и вовсе фикция, которую вообще нельзя употреблять.
2) Второй вариант допускает, что масса все же не аддитивна во всех 4 ФВ, а если это так, то ЧД вовсе не должно существовать, потому что ни энергии ни массы не может быть там где нет времени — за ГС, а объекты, которые мы называем ЧД нечто другое, при слиянии коих масса превращается в энергию гравитационных волн.
Действительно есть какая-то фишка с тем фактом, что энергия потенциальная меньше 0, если система частиц стабильна. То есть, этот случай не описывает сильное взаимодействие скажем в распаде тяжелого сигма-гиперона (1382-1385 МэВ) на пион и более легкий гиперон (без изменения кваркового состава). Тут получается, что кин. энергия этой системы кварков (реальных и виртуальных пар) + глюонов изначально была выше (по модулю), чем потенциальная энергия связи того, что стало гипероном и мезоном.
Гравитация в центре нейтронных звезд тоже вроде как делает энергетически невыгодным процесс распада сильно насыщенных нейтронами ядер. Предположу тут, что внутри НЗ может существовать изотоп железа-59, не производя распад в кобальт за 44.5 суток. Просто потому, что отлет электрона даже на 1s-орбиталь уже не будет давать энергетического эффекта в грав. поле звезды.
На самом деле сила у ГС возрастает до бесконечности не зависимо от того какая это ЧД, потому что сила по ньютоновской формуле умножается на кофициент замедления времени.

Сила чего действующая на что и в какой системе отсчёта?

Обычно гравитацией называем силу, действующую на тело массой m, деленную на m. Да, в ОТО кроме обычной записи уравнения геодезической для частицы с m > 0 ещё нужно добавить прочие, негравитационные силы.
Какие именно силы, действующие на тело, свободно-падающее в ЧД, обращаются в бесконечность на горизонте событий?
Да я не знаю, куда обращаются эти силы на горизонте. Есть обычные силы, а из обычных сил остается (в фундаментальнос смысле) только ЭМ поле. Так что не забудьте в уравнение Эйнштейна кроме массы ЧД добавить тензор энергии-импульса падющей на ЧД плазмы.
Конечно в какой-то момент столкновение скажем протонов и ядер гелия начнет рождать антипротоны и пионы, но плазму с такой температурой уже сложно будет описать.
Если не знаете, зачем пишете? И к чему этот загруз про плазму, антипротоны, пионы, если вопрос был о совсем другом?
Формально ускорение (И «сила») на горизонте становятся бесконечными (я давал ссылку) — все это относительно рядом висящего наблюдателя. Относительно далекого наблюдателя эта бесконечность умножается на ноль (замедление времени). Забавно что это произведение сходится к вполне конечной величине, хотя она не имеет особого смысла
В ОТО гравитация — это НЕ СИЛА а следствие искривление пространства времени
Она не сила, но в слабых грав. потенциалах работает приближение эффектов ОТО теорией грав. Ньютона (ЕМНИП следует из логики вывода уравнений ОТО).
В каком-то приближении можно описать движение релятивистских частиц под действием такой обычной гравитационной силы. Правда понятно, что будут погрешности. Скажем если какие-то негравитационные силы болтают пробную частицу в области на расстоянии 46-70 млн. км от Солнца 100 лет, а мы можем рассчитать (с учетом СТО. у нас ведь релятивстская частица) её положение по «широте и долготе» с точностью до 40 секунд дуги.
Речь в ветке была о горизонте событий. Возле него нельзя использовать приближения.
Нельзя, Вы правы. Ещё и эффекты СТО будут странно работать для такой «релятивистской пробной частицы».
Это всего лишь концепция того, что наша вселенная однажды появилась, а не существовала вечно, но это уже даже не теория а факт, следует так объяснять теорию БВ для простого народа не больше не меньше.

С чего это вдруг факт?
А взорвалось что? Строить теории что было до БВ можно до бесконечности. Включая то, что взрывов может быть множество, просто в видимой вселенной мы пока что видим последствия одного.

Замедление времени в черной дыре бесконечно

Или до следующего большого взрыва, например?

Нет, оно просто равно 0, никакого времени никакого пространства, никакой энергии и попасть туда нельзя самим не став ЧД.

Хокинг с вами не согласен. А также зафиксированное слияния двух черных дыр тоже с вами не согласно. Что-то там происходит, что-то туда попадает.
Что-то там происходит, что-то туда попадает.


Происходит слияние 2 сингулярностий а так же — переход во что-то энергии и спина кругового движения. ДВижение именно круговое, см. задачку.
А вам не кажется, что у ЧД просто нечему вращаться так как ГС это не твердая поверхность да и времени там нет что бы можно было констатировать сам факт вращательного движения.
Загуглите Эргосферу
Загуглите «предел статичности», без него нет Эргосферы, а предела статичности нет у не вращающихся ЧД, и того мы вернулись к тому с чего начинали — что можно считать вращением ЧД, может горизонта событий, который типа условность, или может сингулярность, которая в принципе не имеет определенных размеров.
Вот такое загуглил:
В отличие от тела, оказавшегося за горизонтом событий, вырваться из-под поверхности предела статичности ещё возможно — при определенном угле и скорости вхождения в эргосферу можно сбросить часть массы в чёрную дыру, зато оставшаяся масса, получив мощное ускорение, будет выброшена за предел статичности.
Какая ж это условность если frame dragging вас увлечет в сторону вращения ЧД с бесконечной силой (грубо говоря)?
Так это мне часто говорят, особенно здесь и вы в их числе:
Как я уже писал, свободно падающий наблюдатель может даже не заметить пересечения горизонта

И как не странно я согласен с вами, он не заметит так, как уже будет мертв(если конечно не уверовать в загробную жизнь за горизонтом событий) и не просто мертв, а расщеплен на самые мелкие элементарные частицы.
Сила взаимодействия ЧД и падающего на нее тело будет равна
GM/R^2*E/c^2*1/(1-Rs/R)^1/2 Где «Е» это полная энергия(кинетическая+покоя) в системе отсчета координат на расстоянии «R» от центра ЧД, а Rs — гравитационный радиус но так как E=mc^2*1/(1-Rs/R)^1/2 то
F=GM/R^2 * (mc^2*1/(1-Rs/R)^1/2)/c^2 * 1/(1-Rs/R)^1/2
Ну и что что сила? Сила делает ускорение, а ускорение локально (в одной точке) не чувствуется, каким бы огромным не было. Чувствуется на телах конечного размера (если разное в разных точках), но когда размер самой ЧД очень большой, то все точки тела человека летят одинаково и тело ничего не чувствует.
Есть некоторая граница r = rs, на которой метрика теряет смысл в том смысле, что координата r не может уменьшаться меньше. Координата r не является длиной радиус-вектора.
Можете строить любые траектории частиц по координатам «радиус-вектор, широта, долгота», лишь бы они не достигали этой границы. Под границей тоже «сложно жить» частице будет, вернуться на границу она конечно не сможет.
метрика теряет смысл
Нет, не теряет. Система координат там теряет смысл, нужно выбрать другую. Метрика — инвариант, она не зависит от координат (в отличии от метрического тензора). На северном или южном полюсе полярные координаты плохо себя ведут, но человек этого не чувствует, когда там находится. Частице нет дела до координат. «Координат не существует на самом деле.»
Под границей тоже «сложно жить» частице будет, вернуться на границу она конечно не сможет.
Понятно что вернуться не сможет. Мы обсуждали что тело почувствует проходя горизонт.
Система координат там теряет смысл, нужно выбрать другую.

Да, где-то мне уже упоминали, что есть даже вариант метрики, которая плавно проходит через горизонт. Просто на бесконечности её нельзя использовать.
Не то, что сила, а градиент силы, а растет он еще быстрее чем сама сила и все по той причине, что в реальностм гравитационное поле само по себе не однородно и быть таким может лишь в самых грубых приближениях. Формула у вас есть можете сами посчитать.
Градиент растет, по этому все же «не заметить пересечение горизонта» — это бред. В любой ЧД Вы за время, вроде как конечное в Вашей СО, достигнете состояния «Я не чувствую своих ног, так как иону кальция не хватит 100 лет на преодоление пути по нервам на 1 метр вверх от центра ЧД».
Локально все ему хватает. Потому что все тело приблизительно с одинаковой скоростью и ускорением летит. Ион кальция по своим часам будет неподвижным относительно всего тела и время там будет одинаково идти. Это удаленный наблюдатель будет стареть очень быстро.
Неоднородность конечная. Значит ее можно свести к пренебрежительно малой увеличивая ЧД, все еще оставаясь конечной очень большая черная дыра может создавать слабо неоднородное гравитационное поле, прямо как земля, например.
Горизонт — это не просто поверхность сферы, она может «вращаться». Но объем под этой «сферой» точно растет. Можно сказать, что после каждого оборота ЧД №1 вокруг ЧД №2 горизонт вокруг ЧД №2 займет тот же объем плюс какой-то дополнительный. Дополнительный будет потому, что тесные системы теряют какую-то часть кин. энергии на грав. волны, даже если это будет 1% кин. энергии за 800 тысяч лет для очень тесных систем.
С таким успехом я могу заявить, что лист бумаги это пространство время, вырезать в нем дырку и начать вращать вокруг дырки лист бумаги, так что по вашему здесь вращается лист, или дырка в нем?
В моей идее форма «дырки» зависит от положения других тел относительно «дырки».
А вам не кажется, что у ЧД просто нечему вращаться так как ГС это не твердая поверхность да и времени там нет что бы можно было констатировать сам факт вращательного движения.

Но излучение Хокинга же есть? Значит движение там есть, а есть движение — есть и время.
Что-то уже на GT обсуждали по поводу «как правильно объяснять излучение Хокинга». Кажется пары «частица-античастица» размазываются (неопределенность в смысле плотности вероятности) как раз по объему радиусом с черную дыру.

И да, можно предположить, что в паре близких ЧД кроме испарения будет туннелирование частиц из одного горизонта (условно назову «глубина 0.01 грав. радиуса от края сферы Шварцшильда») в другую сингулярность (на аналогичную глубину).
«Когда видимая Вселенная была не больше размера атома...». Видимая Вселенная, грубо говоря, это область, где свет мог пройти от одного места до другого с момента t=0. То есть небольшой шарик, который раздувается (примерно!) со скоростью света (на самом деле быстрее). Таких шариков может быть бесконечно много.

Да нет, пока известен только один шарик, и расширяется он как раз со скоростью света, только расширяется во времени, а так называемая «наблюдаемая вселенная», это вселенная начиная с окончания эпохи рекомбинации не более того.
ibb.co/tBXp74K
И на вскидку:
самая далекая наблюдаемая галактика
Красное смещение 11.9 — вполне нормально для галактик.
Ну и цитату приведу из одно науч. поп. статьи:
Согласно стандартной космологической модели, первые галактики, еще очень небольшие, без выраженной структуры, образовались спустя 400 тысяч лет после Большого взрыва.

Вполне возможно, что галактики до эпохи вторичной ионизации имеют такое свойство:
Подобное поведение объясняется «обрывом Бальмера» (Balmer jump), то есть поглощением УФ-фотонов невозбужденными атомами водорода. Благодаря эффекту Доплера излучение далеких галактик сдвигается в область больших длин волн. Поэтому если телескопы регистрируют этот обрыв не в ультрафиолете, а в ИК-диапазоне, то это явный признак того, что галактика от нас очень далека.

Данное явление заметили для галактики с красным смещением 3.717 (соотносят с возрастом 1.65 млрд. лет из 13.7 млрд. текущего возраста).
Источник.
Это понятно, я для примера привел то, что наблюдать объекты с красным смещением z+1=e^пи мы еще долго не сможем, а скорее всего там вообще нет никаких объектов, в это время(596 миллионов лет после БВ) «темные века» могут быть в самом разгаре. Так что подтвердить, или опровергнуть замкнутость вселенной практически нет возможности, есть еще один способ — с помощью РИ, но там куча своих трудностей.
Мы прекрасно наблюдаем области с z=1000 (CMB).
Это куда раньше темных веков
Речь идет только о РИ, но дело в том. что РИ не представляет какой либо конкретный объект.
z = 22.14 — это действительно много. Волна 121.6 нм (водород, переход 2->1) перейдет в волну 2.692 мкм (и т.д. для серии Лаймана до длины 2.018 мкм).
Выводим на высокую орбиту ИК-телескоп (с оптикой и сенсорами под диапазон 2.5-5 мкм) и пробуем найти этот диапазон, идущий от галактик.
Собственно, РИ в значительной мере и есть тот самый диапазон, когда фотоны с длиной волны чуть больше 2.695 мкм перестали поглощаться горячим газом.

Я понимаю, что Вы пытаетесь подвести наблюдаемые факты под какую-то свою «альтернативную» теорию, но я не считаю разумным описывать красное смещение (z-1) как какую-то «экспоненту от отношения окружности к диаметру, равному ct».

Далее. Упомянутая Вами галактика. результат сеанса наблюдения HUDF. Он собрал информацию в таких длинах волн:
Для наблюдений при помощи ACS использовалось 3 широкополосных фильтра: 435 нм, 606 нм, 775 нм, и один низкочастотный — 850 нм

Условно говоря, чтобы вместо 850 нм от источника с z = 11.9 получить сигнал от источника с z = 22.14 нужно суметь набрать много данных на длине волны 1481.5 нм.
Да, это может быть очень сложная задача. Вот нашел «спектрометр»:
www.czl.ru/pr/spectr/sol-1-7
z = 22.14 — это действительно много. Волна 121.6 нм (водород, переход 2->1) перейдет в волну 2.692 мкм (и т.д. для серии Лаймана до длины 2.018 мкм).

Наблюдать серию Лаймана стает не эффективным мероприятием так как УФ активно поглощался более густыми водородными облаками в раннюю эпоху, нужно наблюдать те серии, которые уже в то время излучались в ИК спектре.
Я понимаю, что Вы пытаетесь подвести наблюдаемые факты под какую-то свою «альтернативную» теорию, но я не считаю разумным описывать красное смещение (z-1) как какую-то «экспоненту от отношения окружности к диаметру, равному ct».

Скорее наблюдаемые факты стали причиной теории… И то что z+1=a/a' это не я придумал это сейчас общепризнанный факт в космологии, только в отличии от современной космологии в моей теории масштабный фактор полностью соответствует временному(так как вселенная расширяется с постоянной скоростью): z+1=Aо/Az=Tо/Tz. Да и вообще еще никто не придумал лучшей симуляции Хабловского закона чем тот же Шарик, который еще в придачу однороден изотропен и безграничен. Но некоторые будут возражать мол «она ускоряется», но так называемое ускорение это лишь вольная интерпретация не стыковок в измерениях сверхновых и теоретических моделей на основе той или инной предполагаемой постоянной Хаббла, но возьмите к примеру постоянную Хаббла не 70 а 64 и все сойдется как надо — вселенная станет больше и старше. Еще могут быть некоторые не учтенные не космологические факторы, например разный уровень поглощения излучения в процессе изменения его длины волны на пути к источнику, так называемая проблема К- поправки и еще многое чего могло быть не учтено.
И то что z+1=a/a'

Масштабный фактор — это не величина, ограниченная Вашим epi, а член метрического тензора (некая глобальная метрика пространства).
Что у Вас сойдется не знаю, современное значение около 67-68.
Проблема с оценкой поглощения в межгалактических масштабах — это может быть. Соответственно оценка расстояния до звезды по диаграмме «мощность излучения»(«какой-то независящий от расстояния параметр»), вроде оценки по цефеидам в ближайших галактиках, мог дать погрешности.
Масштабный фактор — это не величина, ограниченная Вашим epi, а член метрического тензора (некая глобальная метрика пространства

Я знаю, что такое масштабный фактор, но раз называть вещи простыми словами для вас не камильфо — ваше право, но сути это не меняет и как оказалось вы до сих пор не поняли, что число «е» это всего лишь показатель степени изменения «глобальной метрики пространства» за время необходимое для того, что бы фотон преодолел отрезок на поверхности сферы длина дуги, которого равна 1 радиану, отрезок который все время растет с одной и той же скоростю — с, и так как между двумя противоположными полюсами всего π таких отрезков, то пока фотон долетит из одного полюса в другой, то сфера, образующий ее шар и любой отрезок взятый на ее поверхности увеличится π раз в е раз.
И конечно же длина волны фотона также увеличится в е^π раз или z+1=е^π. Но он может зайти на следуйщий круг по этому никакого ограничения нет.
Я не силен в теории замкнутой Вселенной. Действительно, «края Вселенной» мы не видим, но это не означает, что Вселенная подчиняется хитрому закону типа
dr/dt = int{дуга радиуса r} c*dt.
И да, может я не в курсе точных выводов теории, но что-то мне кажется, что начиная с «даты старта» около 5 млрд. лет назад уже невозможен путь фотона, дастигающего из «1 конца Вселенной» в «противоположный» за скажем эти 5 млрд. лет или какое либо другое время.
что Вселенная подчиняется хитрому закону типа

Скорее вы хотели сказать НЕхитрому закону, но на самом деле она только НЕхитрым законам и подчиняется, хитрые законы придумывают себе люди и сами в них барахтаются.
А теперь я придумаю свой миф про черные дыры. Закон физики, принят в 3 чтениях с поправками.
1. Объем сингулярности никогда не падает.
Легко доказать для факта слияния 2 ЧД. Не очень легко — для движения любых гравитирующих масс вокруг общего с ЧД центра масс.
2. Объем сингулярности никогда не падает при движении 2 тел (1 или 2 из которых ЧД) по круговой орбите.
Это тоже кажется легко доказать.
3. Эта поправка была принята конституционных большинством обеих палат ЗС.
Даже при движении по эллиптической орбите эта пара ЧД так быстро теряет кин. энергию и угловой момент (за счет излучения ГВ), что закон выполняется.
Предельный случай понятия «быстро теряет угловой момент» — хотя орбита кажется «эллиптической спиралью», но, на самом деле, за любую четверть периода обращения пары тел вокруг ЦМ «афелий» орбиты становится меньше, чем был «перигелий» до начала этой четверти периода. Если что, идея о быстрой потере орб. момента (эксцентриситета) парой ЧД подтверждается наблюдениями слияний ЧД по спектрам ГВ.

Закон, финальная версия, принятая народом на референдуме (да, это иногда бывает?). У нас растет не объем сингулярности, а площадь. Как я писал в пункт №1, тот пункт доказать легко. А вот эта версия — более сильное утверждение. По некоторым теориям, которые имеют проблемы с фальсифицируемостью, энтропия ЧД пропорциональна площади горизонта. По этому даже не объем, а площадь не может падать. Но есть 1 процесс — теоретическое испарение ЧД. В случае, если испарение ЧД уменьшает площадь горизонта (они должны светиться как АЧТ с температурой выше темп. реликтового излучения), то прирост энтропии обеспечивается за счет энтропии вылетевших из энергии ЧД частиц.
Объем сингулярности

Вам не кажется, что слово «объем» к сингулярности в принципе не применим?
Ну это скажем так объем, который стал «недоступен» для остальной Вселенной для «возвратного» посещения. И да, площадь по Вашему применима?
В простейшем «доказательстве» объем сферической ЧД — это 4pi/3*rg3.
который стал «недоступен» для остальной Вселенной для «возвратного» посещения

Скорее наоборот, это объем с точки зрения удаленного наблюдателя, который вырезан из нашей вселенной и не доступен для посещения.
В простейшем «доказательстве» объем сферической ЧД — это 4pi/3*rg3.

То есть вы тоже меня убеждаете(и так убежден), что сингулярность=объем за горизонтом. Так как наблюдатель, который пересекает ГС со скоростью света(иначе и быть и не может, если вы конечно верите в ЧД) стыкается с обычным Лоренц сокращением пространства в направлении к центру ЧД, по этому с его точки зрения никакого объема за ГС нет, а вот на счет площади вопрос интересен, я над этим поудамаю.
На счет площади, это из такой теории (по Вики):
Затем они сравнили результат с площадью горизонта событий чёрной дыры — эта площадь пропорциональна энтропии чёрной дыры, как предсказано Бекенштейном и Хокингом на основе классического понимания[2], — и получили идеальное согласие

Я согласен с тем, что понятие «площадь горизонта» — это где-то из области КТГ (причем именно как теории квантования величины «кривизна Римана-Эйнштейна», ИМХО), а с последовательной теорией КГ пока у ученых кажется проблемы.
Ага, значит «площадь пропорциональна энтропии» — это и без теории струн предсказывают. Но, это условие «площадь растет» явно выполняется, так как при росте массы в 2 раза (слияние 2 ЧД равной массы) радиус растет в 2 раза, а площадь — в 4/2 = 2 раза. Желающие могут прикинуть для малого возмущения, вроде падения ЧД массой 5000 масс Планка на ЧД массой 5000 масс Солнца.
Падающий наблюдатель видит оставленную им вселенную немного замедленной во времени и покрасневшей.

А почему не посиневшей, или скажем фиолетовой? Ведь энергия фотонов по пути в черную дыру растет. Или в системе падающего наблюдателя она падает?
Именно так. Если бы наблюдлатель бы не падал сам< то он бы видел, разумеется, посиневшие фотоны

Публикации

Изменить настройки темы

Истории